Uworld 69%

Ace your homework & exams now with Quizwiz!

How do beta blockers impact RAAS system?

Bblockers inhibit renin release by blocking beta1 receptor mediated regulation of RAAS. This reduces plasma renin activity, with a resulting reduction in angiotensin I, II and aldosterone levels.

t(14;18) translocation what is overexpressed?

Bcl2 overexpression

Potassium concentration in bowman's space compared to glomerular capillaries?

Because K+ is freely filtered across the glomerular membrane, K+ concentration within Bowman's space is equal to that in the glomerular capillaries & represents 100% of the filtered load

Thiamine deficiency

Beriberi(b1b1) & Wernicke-Korsicoff

Relationship between beta-blockers and thyroid hormone

Beta-blockers ameliorate the symptoms of thyrotoxicosis by decreasing the effect of sympathetic adrenergic impulses on target organs. THEY ALSO DECREASE THE RATE OF PERIPHERAL CONVERSION OF T4 TO T3.

Patient with post-operative voiding issues (not peeing out everything in bladder) [decreased contraction of bladder/decreased mictrition reflex]...what is a potential treatment?

Bethanolocol...need to contract detrusor muscles and in order to do this you need a muscarinic agonist. (or an a1 blocking drug)

Jaundice, dark urine, acholic stools in first two months of life due to hyperbilirubinemia...what is pathology?

Biliary atresia...biopsy reveales intraheptic bile duct proliferation, portal tract edema & fibrsosi.

How do penicillins and cephalosporins work?

Binding irreversibly to PBPs like transpeptidases

Mechanism of action of Heparin?

Binds Anithrombin 3 ...unfractionated and low molecular weight heparins are commonly used for DVT prophylaxis. HEPARIN INCREASES THE EFFECT OF THE NATURALLY OCCURING ANTICOAGULANT ANTITHROMBIN III

What vitamin deficiency causes mental status change, myalgias, anorexia & chronic dermatologic changes. Deficiency due to poor diet + excessive egg white consumption?

Biotin (Vitamin B7) needed as a cofactor: pyruvate carboxylase, Acetyl-CoA carboxylase, Propionyl-CoA carbooxylase

Patients who experience major depressive & hypomanic episodes are diagnosed with what?

Bipolar 2 (hypomanic episodes are less severe and do not involve psychosis)

Patients experiencing a major depressive episode should be screened for what before SSRis are given?

Bipolar disorder (HISTORY OF MANIA)...can result in treatment-emergent mania. Anti-depressant monotherapy should be avoided in bipolar patients

How are ERYTHROCYTES able to conduct glycolysis and yield no ATP?

Bisphosphoglycerate mutase allows erythrocytes to bypass the step between 1,3BPG and 3-phosphoglycerate. Erythrocytes convert 1,3-BPG to 2,3-BPG producing NO ATP. 2,3BPG decreases hemoglobins affinity for oxygen and this allows blood cells to deliver oxygen (when the blood oxygen concentration is low) to peripheral tissues.

What happens with NE extravasation? What drug can reverse/prevent?

Blanching of a vein into which NE is being infused + induration and palor of the tissues surrounding the IV site are signs of NE extravasation (resulting in vasoconstriction) Tissue necrosis is prevented by a1-blocking drug like PHENTOLAMINE

50 year old man 4 months of cough and weight loss, patient is a farmer in mississippi. Has pulmonary inflitrates in right upper lobe +granulomatous inflammation. What bug?

Blastomyces Dermatidis...in immunocompetent will manifest as pulmonary disease. In immunocompromised will manifest as dissemination to skin & bones.

RivaroXaban & ApiXaban (both oral) work how and where?

Block factor Xa it is even in the name. RivaroXaban & ApiXaban. They can be given as monotherapy and are used for VENOUS THROMBOEMBOLISM (DVT) & AFIB

Patients with parkinsons is in hospital and on IV dopamine for hemodynamic support, what structure (and what is it made of) is stopping the dopamine from impacting parkinson's?

Blood Brain barrier = TIGHT JUNCTIONS = ZONA OCCLUDENS (made of claudins and occludins). Tight junctions between NONFENESTRATED CAPILLARY ENDOTHELIAL CELLS

How does O2 and CO2 in blood change during exercise?

Blood CO2 goes up which means pH goes down. Exercising muscles need additional O2 so venous O2 blood is decreased...venous blood CO2 is increased due to increased CO2 production and venous pH is decreased.

Potential side effects of motion sickness medicine?

Blurry vision, dry mouth, urinary retention, constipation

Difference between myasthenia gravis and lambert-eaton syndrome?

Both are caused by poor signal transmission at neuromuscular junction...lambert-eaton has an underlying malignancy (ANTIBODIES TO VOLTAGE-GATED PRESYNAPTIC CALCIUM CHANNELS ARE FOUND IN THESE PATIENTS)

Where does the highest osmolarity in the nephron occur?

Bottom of the loop of Henle.

Where is the tubular concentration of PAH the lowest?

Bowman's space. (it is freely filtered by glomerulus into bowman's space)

Under starvation situations when ketone bodies are being use, what tissue uses them? What tissue cannot use them?

Brain (prefers glucose but will use ketones under starvation. Erythrocytes CANNOT use ketones since erythrocytes do not have mitochondria

Patient understands language but is not able to properly formulate motor commands to write or form words? Where is lesion?

Broca (motor, nonfluent) aphasia results from damage to the INFERIOR FRONTAL GYRUS of the dominant hemisphere. This aphasia may be associated with right upper limb and face weakness due to extension of the lesion into the primary motor cortex.

The respiratory tract lining changes in structure and function as it progresses distally....explain these changes

Bronchi has pseudostratified columnar ciliated epithelium with goblet cells and submucosal mucoserous glands & cartilage...Bronchioles, terminal bronchioles & respiratory bronchioles lack goblet cells, glands and cartilage (by terminal bronchioles the epithelium is ciliated simple cuboidal.

In patients with long-term opiod therapy what drug can precipitate withdrawal?

Buprenorphine is a partial opiod receptor agonist that binds with high affinity but has low intrinsic activity...it can displace other opiods and precipitate withdrawal.

7 year old, recurrent granulomatous skin infections & prior episode of aspergillus pneumonia. Genetic testing shows inactivated component of a neutrophil oxidase enzyme. What bug/disease?

Burkholderia Cepacia & CGD...chronic granulomatous disease: recurrent bacterial & fungus diseases due to impaired intracellular killing by phagocytes. Caused by genetic defect in NADPH oxidase complex (normally NADPH oxidase participates in the killing of microbes within neutrophil & macrophage phagolysosomes. [Staph, Burkholderia, Serratia, Nocardia, Aspergillus]

What is the responsibility of the protein translocated in Burkitt lymphoma

Burkitt Lymphoma (EBV association) t(8;14) . The product of C-MYC functions as a TRANSCRIPTION ACTIVATION. Burkitt Lymphoma has a STARRY SKY APPEARANCE. Translocation of C-MYC on long arm of chromosome 8 with Ig heavy chain region on chromosome 14 produces a nuclear phosphoprotein(c-myc) functions as TRANSCRIPTION ACTIVATOR

For class I antiarrhythmics, what is the sodium-channel-binding strength from most to least?

C > A > B = Class 1C antiarrhytmics demonstrate the most use dependence and class 1B drus have the least

What is the formula for calculating Clearance of any substance, how do you get the filtration fraction?

C(s) = ((Urine conc of S) x (Urine flow rate))/(Plasma S0....FF = GFR/RPF (use creatinine for GFR and use PAH for RPF)

A new drug has an impact on QRS prolongation, but does not effect QT interval what part of cardiac action potential will it effect?

"B" the initial upstroke

In intimate partner violence, what is the priority of the doctor? What terminology should be used?

"Do you have a safe place to go in an emergency"...ensure patients safety and develop a emergency plan

Woman has genital lesions and her husband calls dr office two weeks later asking about wife's current condition, what is the appropriate response?

"IF YOUR SPOUSE IS A PATIENT HERE, SHE WOULD HAVE TO PROVIDE A RELEASE OF INFORMATION" HIPPA protects health info by requiring a VERBAL or WRITTEN authorization for release of information. Fam members get nothing without patient's consent.

12-24hrs after ischemic stroke what cells are expected on histology?

"Red Neurons" (eosinophils & loss of nissl substance)

Antiretroviral Medications: gp41 inhibitor

"fusion inhibitor" Enfurvirtide binds to heptad repeat 1(HR1) of gp41. It INHIBITS PENETRATION INTO THE TARGET CELLS

Most common pathogen cystitis & pyelonephritis? Second most common?

#1 E. coli!!! #2 Staph Saphrolyticus

False positive

(1-Specificity)*(number of patients actually without the disease)

Calculate RBF

(PAH clearance)/ (1- hematocrit)

What is the net filtration equation?

(Pc-Pi) - (pic - pii)

What are T-tubules and what is their job?

(if messed up/absent: UNCOORDINATED CONTRACTION OF MYOFIBRILS) T-tubules are invaginations of the sarcolemma that extend into each muscle fiber...they TRANSMIT DEPOLARIZATION signals to the SR & trigger release of calcium

When does a left shift of the oxygen-dissociation curve occur?

(increased pH(basic), decreased 2,3BPG, decreased temperature(HYPOTHERMIA)

Baby has cystic mass with a tuft of hair, what actions by mother during pregnancy could have prevented this?

(meningocele & myelomeningocele) VITAMIN SUPPLEMENTATION FOLATE B9. Failure of neural tube closure at 4 weeks gestation results in neural tube defects [pericontraceptal use of folate(b9) reduces risk][first trimester use of valprorate is a signficant risk factor for neural tube defect]

What is a kinesin, what process would be interrupted if there were none?

(needs ATP) Kinesin +anterograde transport of intracellular vesicles and organelles towards the +(POSITIVE/GROWING END). Kinesin use energy from ATP hydrolysis

What is the 4th heart sound indicative of?

(s4)s1s2 (buh-dUp-dup) Low frequency heart sound at the end of diastole just before S1...it is due to decreased left ventricular compliance [often associated with restrictive cardiomyopathy & left ventricular hypertrophy]

What do aldosterone receptor antagonists do?

(spironolactone & eplerenone) inhibit effects of aldosterone and reduce secetion of K+ & H+ by the collecting tububle.

There is a 20% probability of concluding that there is no difference in blood cholesterol when there is one. What is the power of the study?

.80 ~ POWER = 1 - BETA = the probability of seeing a difference when there is one

Blotchy red muscle fibers on Gomori trichrome stain are characteristic of what? What percent change does the man in the vignette have of passing to kid?

0%!!! Blotchy red muscle fibers on GOMORI TRICHROME stain are characteristic of MITOCHONDRIAL MYOPATHY (abnormal mitochondria accumulate under the sarcolemma of muscle fibers

CN nerve foramina:

1(cribiform plate) 2(optic canal) 3,4,V1,6(SOF), V2(foramen rotundum), V3(foramen ovale), [foramen spinosum - middle meningeal artery &vein] 7,8(IAM) 9,10,11,jugularvein(jugular foramen), 12(hypoglossal canal) vertebral arteries, brain stem and SPINAL ROOTS of cn11(foramen magnum)

How does the histological picture of post-strep glomerulonephritis look?

1) ENLARGED hypercellular glomeruli on light microscopy 2) LUMPY-BUMPY granular deposits of IgG & C3 on immunoflourescent (not linear not linear not linear) 3) electron dense deposits on epithelial side of the basal membrane on electron microscopy.

Given phenotypucally normal parents, the probability of a female sibling of a male affected by an X-linked recessive disease will give birth to an affected child is what?

1/8 (hemophilia A & B are both X-linked recessive)

t(8;14) translocation what is overexpressed?

C-myc

What deficiency impairs synthesis of androgens, estrogens & cortisol but does NOT inhibit mineralocorticoid production?

17a-hydroxylase deficiency HYPOGONADISM, HYPERTENSION, HYPOKALEMIA

1st pharyngeal arch

1st aortic arch & trigeminal nerve [1st arch regresses almost completely...gives rise to part of maxillary artery]

Abdominal discomfort, flatulence, bloating, occasional diarrhea...epigastric pain. Hyperemic mucosa hyperplasia of tubular submucosal glands. Where is the problem?

1st part of duodenum Brunner's Glands (submucosal glands). Brunner glands secrete copious amounts of alkaline mucous into duodenum

In normal vitamin D metabolism what step does UVB(sun light) get involved?

1st step UVb rays from sun takes from 7-dehydrocholesterol -> Vitamin D2/D3

What are the Vitamin K dependent coagulation factors, which has the shortest half life?

2,7,9,10...7 has shortest half-life. Failure of prothrombin time to correct with vitamin K supplementation indicates factor 7 deficiency (often due to underlying liver disease).

2nd pharyngeal arch

2nd aortic arch & facial nerve...pharyngeal arch gives rise to the muscles of facial expression + ear and hyoid associated structures

Cystic Fibrosis, what is It most commonly due to?

3 base-pair deletion in the cystic fibrosis transmembrane conductance regulator (CFTR) gene at amino acid position deltaF508. The mutation impairs POST-TRANSLATIONAL PROCESSING OF CFTR resulting in shunting of CFTR toward the proteosome = complete absense of protein on cell surface

Niacin deficiency

3 d's of pellagra: dementia, dermatitis, diarrhea

Genetic instability in hepatitis C is due to what?

3' -> 5' exonuclease activity of its RNA polymerase [enveloped glycoprotein sequences contain a hypervariable region prone to frequent genetic mutation]

Patient is started on zidovudine, what component of the viral genome replication is mostly likely inhibited by zidovudine

3'->5' PHOSPHODIESTER BOND FORMATION ~ Zidovudine (AZT) is a nucleoside reverse transcriptase inhibitor used to tx HIV infection. It competitively binds to reverse transcriptase and is incorportated into the viral genome as a thymidine analo. AZT does not have a 3'-OH group so making a 3' -> 5' phosphodiester bond is impossible

1 week old poor feeding, unusual movements involving left thumb and hand, HYPOPLASTIC MANDIBLE, LOW SET EARS, BIFID UVULA, CLEFT PALATE. What embryological structure?

3rd PHARYNGEAL POUCH - 22q11.2 microdeletion ~ DiGeorge Syndrome. [3rd & 4th pharyngeal pouches]. +hypocalcemia & t-cell deficiency. [twitching of nose and lips from touching facial nerve(chvostek); inflation of blood pressure cuff = carpal spasm(troussaue sign)

Drug is eliminated via 1st order kinetics, has a half-life of 10 hours...if given by continuous infusion how long would it take to achieve 95% plasma steady state?

40 hours...during continuous infusion of a drug metabolized by first-order kinetics, the steady state concentration is reached in 4 to 5 half-lives.

6 year old girl, very short for stature and has 45, XO karyotype...given a drug. What disease? What drug/pathway?

45XO = Turner Syndrome. Drug = Growth Hormone. Pathway = JAK/STAT pathway...1 Growth hormone molecule binds 2 receptors and activates JAK(nonreceptor tyrosine kinase) phosphorylates several tyrosine kinase residues which are the docking sites for STAT(signal transducer and activator of transcription). Once STAT is recruited it is also phosphorylated by JAK. STAT dimerizes and goes to nucleus where it induces IGF-1 gene trascription by binding to the PROMOTER

Unbalanced robertsonian translocation in baby with protruding tongue, excessive skin at nape of the neck, upslanting palpebral fissures?

46, XX, t(14;21)

Mild intellectual disability with 47 chromosomes, what disease +gynecomastia. What disease? Most common process impaired

47 XXY Klienfelters, Meiotic Non-disjunction = Tall stature, small, firm testes, azoospermia & gynecomastia. (mild intellectual disability increases with each additional x chromosome)

Clenched hands with overlapping fingers, heart defect, low set ears, small mandible....what chromosomal abnormality?

47, XX, +18 trisomy 18, Edwards syndrome is most commonly the result of meiotic nondisjunction due to advanced maternal age.

Woman gives birth to a phenotypical male with 46XY...but he has hypospadias & a small phalus, testes are normal but are inside inguinal area and testosterone levels are normal. What is deficient?

5-alpha-reductase [5-a-reductase is needed to convert testosterone to dihydrotestosetone] this is needed to male external genitalia

What does finasteride inhibit and what is it used for?

5-alpha-reductase inhibitor...suppresses peripheral conversion of testosterone to dihydrotestosterone ...used to treat benign prostatic hyperplasia and androgenic alopecia

Patient is looking for medication that would best reduce votiming, what is the critical site for achieving that function?

5HT3 receptor. 5HT-3 receptor ANTAGONISTS are useful for tx of visceral nausea due to GI insults (like gastroenteritis, chemotherapy & general anesthesia)

What percentage obstruction of a fixed atherosclerotic plaque is associated with stable angina?

75%+ [ATHEROSCLEROTIC PLAQUE OBSTRUCTING 80% OF THE CORONARY ARTERY LUMEN, NO THROMBUS...question about a man chest pain walking up hill, especially against wind] {less than 75% = assymptomatic}

A couple is trying to get pregnant, how long after sex would the hCG levels be detectable on a pregnancy test?

8 days after fertilization (6 days at the earliest in serum). B-hCG is produced by the syncytiotrophoblast after implantation (6-7 days after fertilization). Serum 7 days, Urine 14 days.

90% of follicular lymphoma patients have what translocation? What is overly expressed?

90% of follicular (non-hodgkin) lymphoma have the t(14;18) = overexpression of antiapoptotic BCL-2 protein.

Urine has frothy appearance, easy fatiguability, and anorexia...+pitting edema, what is physiologic pressure problem?

A DECREASE IN PLASMA ONCOTIC PRESSURE ...frothy urine ~ proteinuria (nephrotic syndrome) regional or generalized interstitial edema because of a decrease in serum albumin [total protein concentration lowers the plasma oncotic pressure and increases the plasma net filtration in capillary beds]

In a patient with mitral regurgitation, the most reliable auscultatory finding indicating severe MR LV overload is what?

A LEFT-SIDED S3 GALLOP. [PRESENCE OF AN AUDIBLE S3]

What is strong evidence of malignancy in lymph node enlargement?

A MONOCLONAL LYMPHOCYTIC PROLIFERATION IS STRONG EVIDENCE OF MALIGNANCY

Man undergoes total gastrectomy, physicians counsels him saying he must now take a life-long administration of what?

A WATER-SOLUBLE VITAMIN = Vitamin B12....Intrinsic Factor is a glycoprotein that is normally secreted by parietal cells in the stomach and is necessary for the absorption of Vitamin B12 in the ileum. Patients who have undergone a total gastrectomy require lifelong B12 supplementation due to inability to produce IF.

What is a cholesteatoma?

A collection of squamous cell debris that form a mass behind the tympanic membrane...they can be congenital or may occur as a primary lesion following infection/trauma/surgery. They can cause hearing loss due to erosion of auditory ossicles.

What is a communicating hydrocoele?

A communicating hydrocele results when serous fluid accumulates within the tunica vaginalis in the setting of a patent processus vaginalis [it presents as painless swelling that transilluminates on examination]

What is wilson's disease?

A progressive neurological disease likely to be seen on the USMLE that causes cystic degradation of the PUTAMEN. The putamen shows up as medial to the insula and lateral to the globus pallidus & internal capsule.

A study's power increases as what increases? Formula?

A study's power increases as its sample size increases (power = 1 - B)

What type of produce is released by merocrine glands?

A watery secretory product is released via exocytosis with no loss of cell membrane

On auscultation what is the best indicator of mitral stenosis?

A2-to-opening snap time interval...the best and most reliable auscultatory indicator of the degree of mitral stenosis is the A2-OS interval. A shorter interval indicates more severe stenosis.

Pulsating central abdominal mass?

AAA = CHRONIC TRANSMURAL INFLAMMATION

On histology periodic acid-schiff postive granules + wheezing and poor air movement?

AAT deficiency. Alpha-1-antitrypsin deficiency -> panacinar emphysema.

Explain the toxin in diptheria?

AB toxin, B = binding, does action via ENDOCYTOSIS [HEPARIN-BINDING EPIDERMAL GROWTH FACTOR RECEPTOR ON CARDIAC & NEURAL CELLS] A = active, INHIBITS HOST CELL PROTEIN SYNTHESIS BY CATALYZING ADP-RIBOSYLATION OF PROTEIN ELONGATION FACTOR 2 (EF-2) ...ef-2 is necessary for tRNA to insert new amino acids into growing protein chain during translation. TOXIN CAUSES DEATH BY INHIBITING CELL PROTEIN SYNTHESIS

Man with Pneumocystic pneumonia and HIV history with lung crackles and CD4+ count of 122, positive HLA-B*57:01....finding is relevant if patient is considered for what medication?

ABACAVIR ~ nrti used in hiv/aids...allergic rxn occurs in 2-8% of patients taking abacavir and is highly associated with HLA-B*57:01

Cells ID'd by immunohistological staining are showing an overexpressed 185kD glycoprotein that spans the cell membrane and has tyrosine kinase...what is the function of the protein?

ACCELERATES CELL PROLIFERATION. The HER2 oncogene encodes for a transmembrane glycoprotein with intrinsic tyrosine kinase activity and is a member of the epidermal growth factor receptors

Relationship between ace inhibitors and potassium?

ACE INHIBITORS ARE LIKELY TO INCREASE POTASSIUM ~ HYPERKALEMIA & DRY COUGH

What drugs can cause significant first-dose hypotension in paitents with volume depletion?

ACE inhibitors (act on efferent arteriole dilating it) to reduce this ACE inhibitor therapy should be started a low dose.

How can ACE inhibitors cause an acute rise in serum creatinine?

ACE inhibitors can cause an acute rise in serum creatinine by REDUCING GFR, which REDUCES FF...so more creatinine gets thru...which can precipitate renal failure.

Question is talking about gluconeogenesis and asking about the conversation of pyruvate to glucose...what substance directly STIMULATES the first enzyme involved in the process/

ACETYL-COA ~ Acetyl-CoA stimulates gluconeogenesis by increasing the activity of pyrvuate carboxylase when acetyl-CoA is abundant. This regulatory step allows pyruvate to be shunted toward acetyl-CoA production when Acetyl-CoA levels are low

How is isoniazid metabolized?

ACETYLATION ~ isoniazid is metabolized by acetylation, the speed with which a patient is able to ACETYLATE drugs depends on whether they are FAST or SLOW ACETYLATORS

Early infant with cardiomegaly, sever hypotonia, macroglossia, hepatomegaly....muscle biopsy shows enlarged lysosomes containing PAS positive material, what enzyme is deficient?

ACID A-GLUCOSIDASE ~ glycogen storage disease type II (POMPE DISEASE)...acid maltase=a-glucosidase; deficiency leads to abnormal glycogen accumulation within lysosomal vesicles is seen on muscle biopsy

Young girl incidental finding of ectopic tissue, EM shows cells have a highly developed smooth ER...what do the cells get stimulated by?

ACTH. Smooth ER contains enzymes for steroid and phospholipid biosynthesis.[all steroid-producing cells contain a well-developed SER]

Patient has a slowly growing mass that began in the setting of oral trauma and has recently been draining yellow pus through the skin...what bug?

ACTINOMYCES ISRAELII (gram possitive organism best known for causing cervicofacial actinomycosis in patients following dental manipulation or other oral trauma. The disease is characterized by a slowly growing and firm-feeling abscess in the face or neck region that eventually forms cutaneous sinus tracts (penicillin + debridement)

What are the atypical lymphocytes in blood smear of a teenage girl with persistent fever and sore throat with cervical lymphadenopathy and splenomegaly?

ACTIVATED CD8+ cytotoxic lymphocytes...the activated t-lymphocytes destroy virally-infected B lymphocytes

Woman with faituge, exertional dyspnea and cough has noncaseating granulomas on lymph node biopsy, what is the most likely cause of elevated calcium levels?

ACTIVATED MACROPHAGES ~ hypercalcemia in SARCOIDOSIS is caused by PTH-independent formation of 1,25-dihydroxyvitamin D by activated MACROPHAGES. This leads to increased absorption of calcium

Further evaluation shows that the patient's lymphocytes contain a mutated and functionally defective Fas gene product. What immunologic mechanisms is most likely impaired in this patient as a result of this molecular defect?

ACTIVATION-INDUCED T LYMPHOCYTE DEATH = EXTRINSIC PATHWAY OF APOPTOSIS = The Fas receptor acts to initiate the extrinsic pathway of apoptosis. Mutations involving the Fas receptor or Fas ligand can prevent apoptosis of autoreactive lymphocytes, thereby increasing the risk of autoimmune disorders such as SLE

Girl OD'd on something: abdominal pain, vomting, diarrhea, hypotension, GARLIC ODOR what is tx?

ACUTE ARSENIC POISONIN (insecticides & contaminated water) DIMERCAPROL is the chelating agent of choice.

Man with month long history of fever, fatigue, joint pain, urticarial skin rash...has elevated ALT & AST, what is likely diagnosis?

ACUTE HEPATITIS B can cuase a serum sickness-like syndrome with joint pain, lymphadenopathy & pruritic urticarial rash. Other features include right upper quadrant pain, heptomegaly & elevated hepatic transaminaselevels

Image shows blast cells with Auer rods, what kind of cancer?

ACUTE MYELOID LEUKEMIA ~ AUER RODS

35 year old man chalky white lesions in the mesentary, histo: fat cell destruction and calcium deposition?

ACUTE PANCREATITIS ~ pancreas is grossly edematous, focal areas of fat necrosis, calcium deposition and interstiial edema.

Woman has primary genital herpes (HSV) takes an antiviral drug, how does it work?

ACYCLOVIR ~ incorporated into newly replicated viral DNA and ultimately terminate viral DNA chain synthesis.

5 month old chronic diarrhea & failure to thrive, has a low blood T-cell count and super low Ig level. Experiemental treatment suggested where cells are inefected with retroviral vector coding the gene for his deficient proteion. The gene most likely codes for:

ADENOSINE DEAMINASE ~ SCID.The second most common cause of SCID is autosomal recessive deficiency of adenosine deaminiase, an enzyme necessary for the elimination of excess adenosine within cells. TOXIC levels of adenosine accumulate within lymphocytes in this condition, leading to lymphocyte death & resultant cellular and humoral immune deficiency.

24 year old male presents to ED with sudden onset of palpitations, he had an episode similar a year ago. Rapid IV injection of a drug into the patient results in instantaneous resolution of the arrythmia but he experienced flushing, burning of chest and shortness of breath. what drug?

ADENOSINE ~ adenosine is a rapidly acting antiarrhythmic used to quickly convert people out of PSVT(paroxysmal supraventricular tachycardia). It is also rapidly cleared and has a half-life of less than 10 seconds. [can cause chest burning, flushing, THIS DRUG IS USED IN CHEMICAL STRESS TESTS]

Where does ADH act in kidney and what is its purpose?

ADH acts on medullary segment of collecting duct to increase urea & water reabsorption, allowing for production of maximally concentrated urine. Acts on V1(vasoconstriction & increased prostaglandin release) & V2(antidiuretic response)

Man in accident has leg cast, his leg circumference shrinks two cm, what other process is similar?

ADRENAL GLANDS DURING PROLONGED GLUCOCORTICOID THERAPY ~ diffuse atrophy ~ loss due to cellular or reduced cell size.

3 year old, high-grade fever, vomiting, neck stiffness altered mental status...hypotension, tachycardia, petechial rash....autopsy would likely show what?

ADRENAL HEMORRHAGE ~ bacterial meningitidis ~ NEISSERIA MENINGITIDIS = DIC from sepsis, shock, onset of ADRENAL CRISIS = WATERHOUSE-FRIDERICHSEN SYNDROME

Patient is experiencing Acyclovir nephrotoxicity (serum creatinine has gone from .9 to 3.4) the observed finding could have been prevented by what?

AGGRESSIVE INTRAVENOUS HYDRATION ~ acyclovir can cause crystalline nephropathy if adequate hydration is not also provided

18 month old with language regression, was saying several words before and is now no longer able to speak any words at all. Family comes from poor part of town. What ENZYME is most likely inhibited in this patient?

ALA-AMINOLEVULINATE DEHYDROGENASE ~ LEAD TOXICITY. Cannot go from ALA to PBG(porphobilinogen). Young children in homes built before 1978 are at high risk for lead toxicity. Lead directly inhibits ferrochelatase and ALA(aminolevulinic acid dehydratase) resulting in anemia, ALA accumulation & elevated zinc protoporphyrin levels. NEUROTOXICITY is also a significant long-term complication

Question stem talks about kid with perianal itching especially at night, child is otherwise healthy scotch tape test is positive (presence of eggs asymmetrically flatterend on one side) what is the most appropriate tx/

ALBENDAZOLE ~ ENTEROBIUS VERMICULARIS (enterobiasis) occurs most frequently in school-age children and presents with perianal pruritus. Diagnosis is made via scotch tape test. Albendazole is FIRST-LINE tx, with PYRANTEL PAMOATE as an alternate preferred in PREGNANT patients

What should you be thinking when a patient has an acute onset of tremulousness, agitation and elevated pulse 48 hours after being admitted to the hospital? What should you prescribe?

ALCOHOL WITHDRAWAL ~ insomnia, tremulousness, anxiety, autonomic hyperactivity BENZODIAZEPINES ARE FIRST LINE (substitute action of alcohol on GABA receptors). DIAZEPAM OR CHLORDIAZEPOXIDE

What type of leukemia are down's patients likely to experience?

ALL (acute lymphoblastic leukemia)

Man takes a bunch of meds, BP is 120/80 in supine and 90/60 when standing....blockade of what receptors is likely contributing to the patient's current condition?

ALPHA 1 ADRENERGIC ~ orthostatic hypotension, meds like a1-adrenergic antagonists, diuretics...or volume depletion & autonomic dysfunction are common causes

26 year old man with rapidly progressive glomerulonephritis & alveolar hemorrhage (SOB, hemoptysis) what antiobodies?

ALPHA 3 CHAIN OF TYPE 4 COLLAGEN ~ goodpasture syndrome

What induces resistance during mycobacterium tuberculosis?

ALTERED STRUCTURE OF ENZYMES INVOLVED IN BACTERIAL RNA SYNTHESIS = structural alteration of DNA-dependent RNA polymerase induces bacterial resistance.

Special cells found that do have an altered Fas w/no transmembrane domain...what is responsible for formation of altered Fas proteins in cancer cells?

ALTERNATIVE SPLICING (process by which a single gene can code for various unique proteins by selectively including/excluding different DNA coding regions (exons) into mature mRNA

Northern blot image with different types of tissue and different markers for mRNA, why does the gel run differently for different tissues?

ALTERNATIVE SPLICING (process by which a single gene can code for various unique proteins by selectively including/excluding different DNA coding regions (exons) into mature mRNA. Normalprocess where exons of a gene are reconnected in multiple ways during post-transcriptional processing. This creates different mRNA sequences and subsequently, different protein isoforms. ~BIODIVERSITY

Man prescribed HTN medication and develops bilateral swelling, what drug class?

AMLODIPINE (calcium channel blockers cause PERIPHERAL EDEMA & dizziness)

Man with previous urosepsis develops fever & right jaw pain, PE: firm swelling of preauricular area on right side extending to the angle of the mandible...what serum marker would be most helpful?

AMYLASE ~ Suppurative Parotitis in adults (staph aureus...there will be ELEVATED SERUM AMYLASE WITHOUT PANCREATITIS & lipase will be normal)

Skin rash, photosensitivity, arthralgias & renal disease in a young woman...what do you expect and what are the markers?

ANA (found in virtually ALL SLE patients) Anti-dsDNA (highly SPECIFIC for SLE). Anti-smith = anti-snRNPs

Region is composed of ~20 amino acid residues consisting primarily of valine, alanine, isoleucine...this particular region of protein most likely performs what functions?

ANCHORING TO THE CELL MEMBRANE ...integral membrane proteins contain transmembrane domains composed of alpha helices with hydrophobic amino acid residues...these transmembrane domains help anchor the protein to the phospholipid bilayer of the cell membrane.

Woman has gestational ultrasound showing markedly elevated amniotic fluid levels, feeling SOB when laying supine +hx of epilepsy(controlled with meds); PE:abdominal circumference larger than expected for gestational age...what fetal anomalies would account for polyhydramnios?

ANENCEPHALY ~ polyhydramnios (too much amnionic fluid) presents with increased abdominal circumference out of proportion to gestational age. It is either from decreased fetal swallowing or increased fetal urination...can lead to GI obstruction or anencephaly

22 year old immigrant man, routine care no symptoms has Anti-HepA IgG(positive)...Anti-HepA IgM(negative) how sway?

ANICTERIC VIRAL INFECTION AS A TODDLER ...it is most commonly clinically silent in children.

3 year old refuses to move his right upper extremity, mom says he was fine this morning....he was holding hands with his sister earlier and now he's holding his right upper arm at his side with his elbow extended and forearm pronated. Any motion of the right elbow produces pain,,,what structure is injured?

ANNULAR LIGAMENT ~ Radial head subluxation (nursemaid's elbow) radial head subluxation (nursemaid's elbow) results from sudden traction on the outstretched and pronated arm of a child. Affected children are usually in distress in attempt to move the elbow (ANNULAR LIGAMENT IS TORN/DISPLACED]

What test is used to determine whether there are any significant difference ebtween the means of 2+ independent groups?

ANOVA = Analysis of Variance

Suprapubic cystostomy is performed and the trocar and cannula pierces what (besides the bladder)?

ANTERIOR ABDOMINAL APONEUROSIS ~ bladder is EXTRAPERITONEAL and the trocar and cannula wil pierce the layers of the adbominal wall but NOT enter the peritoneum

Flattening of the deltoid and loss of sensation over the deltoid implies what kind of injury?

ANTERIOR DISLOCATION OF THE HUMERUS (axillary nerve is the nerve most commonly injured by anterior shoulder dislocations)

MRSA outbreak in a hospital, where on the body would you expect to be the most commonly MRSA-populated site?

ANTERIOR NARES

12 year old boy evaluated for chronic headaches and visual changes, headaches have gotten worse now there is nausea...intracranial calcified mass (mass removed and has thick, brownish-yellow fluid rich in cholesterol) what cells is the mass derived from?

ANTERIOR PITUITARY ~ craniopharyngiomas ~ suprasellar tumors found in children & composed of calcified cysts containing cholesterol crystals. They arise from remnants of Rathke's pouch, an embryonic precursor of the anterior pituitary.

Cancer immunotherapy what to t-cell specific cell surface receptor targets are there?

ANTI-PD-1 & ANTI-CLTA-4 ~ Programmed death receptor 1 (PD-1) is expressed on the surface of activated T cells. PD-1 and PD-L1 binding downregulates the immune response against tumor cells by inhibiting cytotoxic t cells.

Major adaptive immune mechanisms preventing reinfection of influenza virus is anti-____?

ANTIBODIES AGAINST HEMAGGLUTININ

Acute hemolytic transfusion reaction w/fever, chills,hypotension, chest/back pain, hemoglobinuria what kind of reaction is this?

ANTIBODY-MEDIATED HYPERSENSITIVITY [preformed anti-ABO antibodies against BLOOD in recipient's circulation]

Patient has a hyperacute transplant rejection as soon as graft is given, what kind of reaction?

ANTIBODY-MEDIATED HYPERSENSITIVITY [preformed antibodies against graft in recipient's circulation]

Woman from pakistant works in goat wool processing center and has widened mediastinum on CXR. Sputum and blood cultures reveal large gram-positive rods. What do the bacteria likely produce?

ANTIPHAGOCYTIC D-GLUTAMTE CAPSULE ~ B anthracis. Inhalation of spores "wool sorter's disease" long chains of serpentine/medusa head. D-GLUTAMATE CAPSULE

Man with 5 year HIV history comes to clinic and has had a significant increase in pol gene mutation. What is the observed finding likely related to?

ANTIRETROVIRAL CHEMOTHERAPY ~ the high mutability of HIV-1 allows for the evasion of host humoraland cellular immune responses and the development of ressitance to anti-retroviral drugs. Pol gene mutations are responsible for acquired resistance to HIV reverse transcriptase inhibitors and HIV protease inhibitors. ENV gene mutations enable escape from host neutralizing antibodies

Patient comes in and says her legs are getting smaller while her torso/stomach is getting fatter (lypodystrophy) what kind of medicine is she taking?

ANTIRETROVIRAL MEDICATION (NUCLEOSIDE REVERSE TRANSCRIPTASE INHIBITORS (STAVUDINE & ZIDOVUDINE)

Patient with high BP and chest pain given ENOXAPARIN this drug is expected to bind to which substance in the patient's blood?

ANTITHROMBIN III ...enoxaparin is a LMWH functions like heparin and binds/activated ATIII which will bind to factor Xa and stop the conversion of promthrombin to thrombin...

19 year old with blurred vision has worstened and is a swimmer, he's 6'5" and has pectus excavitum what is he most likely to die from?

AORTIC DISEASE ~ marfan syndrome fibrillin-1 ///mitral valve prolapse & cystic medial degeneration of the aortic -> aortic dissection

What endometrial processes causes the bleeding during menstrual cycle?

APOPTOSIS

What should you expect when a patient with acute pancreatitis has subsequent respiratory failure?

ARDS ~ ALVEOLAR HYALINE MEMBRANES ON AUTOPSY ~ pancreatitis is a major risk factor for acute respiratory distress syndrome as it results in the release of large amounts of inflammtory cytokines and pancreatic enzymes which activates neutrophils in the alveolar tissues

Process takes place that results in vasodilation, a reaction involving what amino acid is responsible for observed dilation?

ARGININE (nitric oxide is derived from Arginine via nitric oxide synthase) As a precursor to nitric oxide, ARGININE supplementation may play an adjunct role in tx of conditions that improve with vasodilation, such as stable angina.

What is the calculation for the attributable risk percentage? (ARP)

ARP(exposed) = 100 x [(RR-1)/RR]

Man with acute MI is admitted and has chest pain but is given medicine that fixes it....the medicine inhibits COX-1 & COX-2 enzymes via irreversible acetylation...resulting in significant relief, what medicine?

ASPIRIN

What autosomal recessive condition occurs due to mutation of the ATM gene and how does it present?

ATM = Ataxia, Telegectasia, Mutated. ATM gene is responsible for DNA BREAK REPAIR. The classic triad is: Cerebellar Ataxia, Telangiectasias & increased risk of sinopulmonary infections. [AR; immune deficiency manifests as an IgA deficiency]

In vitro studies demonstrate that tumor cell lines can become resistant after exposure to various anticancer agents, these cells express a specific cell surface glycoprotein that has what functions?

ATP-DEPENDENT TRANSPORTER ~ the human multidrug resistance (MDR1) gene codes for P-glycoprotein, a transmembrane ATP-dependent efflux pump protein that has a broad specificity for hydrophobic compounds. (this protein can both reduce the influx of drugs into the cytosol and can increase the efflux from the cytoso, thereby preventing the action of chemotherapeutic agents

Kid with multiple pneumonias, weight in 3rd percentile & high sweat chloride content, describe the dysfunctional transmembrane protein?

ATP-GATED the mutated CFTR channel with impaired ATP binding blocks chloride ions creating a sticky mucus buildup = ELEVATED sodium and chloride in the sweat. TRANSMEMBRANE ATP-GATED CHLORIDE CHANNEL.

What substance binds directly to Potassium channels expressed in pancreatic Beta cells?

ATP...glucose is the most important stimulator of insulin release and it enters the beta cells via glucose transporter 2 (GLUT2) and undergoes oxidative metabolism through glycolysis and the citric acid cycle generating ATP...ATP then binds to the regulatory subunit of the ATP-sensitive K+ channel (leading to membrane depolarization)

Man started on a neutral endopeptidase inhibitor, works via increasing urinary output and decreasing total peripheral vascular resistnace...what tissue produces the hormone?

ATRIAL CARDIOMYOCYTES = Atrial Natriuretic Peptide (ANP) inhibits neprilysin and responds to atrial stretch ultimately lowering blood pressure thru peripheral vasodilation, natriuresis and diuresis.

Patient presents with palpitations, tachycardia, IRREGULARY IRREGULAR rhythm...after excessive alcohol consumption

ATRIAL FIBRILLATION = ABSENT P WAVES = HOLIDAY HEART

A 'wide, fixed splitting of the second heart sound' is characteristic of what? What surgical repair to prevent irreversible changes might be necessary?

ATRIAL SEPTAL DEFECT ~ PULMONARY VESSELS

Patient is incompacitated and a friend mentions a sister, but says the patient said he would rather "go naturally" what should physician do?

ATTEMPT TO CONTACT THE PATIENT'S SISTER FOR CONSENT TO INTUBATE...in an emergency, the physician can treat an incapacitated patient without obtaining consent.

What are ateriovenous (AV) shunts?

AV shunts normally result from an AV fistula, which is an abnormal communication between an artery and a vein that BYPASSES ARTERIOLES (the major source of RESISTANCE). So Av shunts allow blood under arterial pressure to enter the venous system. Can be congenital or acquired (trauma). INCREASE PRE-LOAD, DECREASE AFTERLOAD. More volume on LV, less pressure on LV...can lead to high-output cardiac failure.

Woman with social inhibition and feeling like people don't like her or they will judge her, fear of embarassement & rejection

AVOIDANT PERSONALITY DISORDER

Where does the gonadal artery come from?

Abdominal Aorta

How does abetalipoproteinemia show up on histology?

Abetalipoproteinemia is an inherited INABILITY to synthesize APOLIPOPROTEIN-B (in chylomicrons and vldl). ENTEROCYTES WITH CLEAR OF FOAMY CYTOPLASM + BULKY/GREASY STOOLS.

Embryologically, what causes annular pancreas?

Abnormal migration of the VENTRAL pancreatic bud...it encirculates the descending duodenum...7-8th week of development, can present as duodenal obstruction or pancreatitis

Difference in lymphatic draining above and below pectinate line?

Above pectinate line (proximal): Inferior Mesenteric & Internal Illiac Lymph Nodes. Below pectinate line (distal): Inguinal lymph nodes

Normal contraction of the upper esophageal sphincter, decreased amplitude of peristalsis in the mid esophagus, increased tone and incomplete relaxation at LES?

Achalasia = motility disorder caused by reduced numbers of inhibitory ganglion cells in the esophageal wall, this creates an imbalance favoring excitatory ganglion cells.

Macrocephaly, Frontal Bossing, Rhizomelia, Trident Hand, Genu Varum?

Achrondroplasia. Patient will present with normal sized torse and very small arms/legs with depressed nasal bridge & bulging forehead. [50% chance of passing condition to fetus]

Lesions consist of erythematous papules with a central scale and a rough "sandpaper like" texture...develops on chronically sun exposure area of skin?

Actinic Keratosis (considered premalignant lesions and have the potential to progress to squamous cell carcinoma)

What is acute cholecystitis what diagnostic test result is pretty specific for it?

Acute Cholecystitis is caused by gallstone obstruction of the cystic duct [ingestion of fatty foods stimulates the contraction of the gallbladder against the impacted stone causing collick pain]. Failed Gallbladder visualization on radionuclide biliary scan is specific for acute cholecystitis.

how can functional heart murmurs occur?

Acute hemodynamic changes can produce them in the absense of a fixed valve lesion.

What is Gerstman syndrome?

Acute ischemic stroke effecting the ANGULAR GYRUS of the DOMINANT PARIETAL LOBE [a region supplied by the MCA] = Agraphia (cannot write); Acalculia (cannot carryout mathematical calculations); Finger agnosia ( inability to ID individual fingers on hand); Left-Right disorientation

Preinvasive lesion characterized by growth along intact alveolar septa without vascular or stromal invasion?

Adenocarcinoma in situ (MALGINANT NEOPLASM) = dysplastic columnar cells that line the alveolar septa without vascular/stromal invasion

Uniformely enlarged uterus in middle aged female w/heavy menstrual bleeding. Disease and histological finding?

Adenomyosis = endometrial glandular tissue in the myometrium. Patient will have normal appearing endometrial tissue on biopsy

What drugs effect phase 4 of the cardiac action potential? What happens when these drugs exert their effect?

Adenosine & Acetylcholine prolong phase 4 of the action potential, reducing the rate of spontaneous depolarization in cardiac pacemaker cells.

Which group of women statistically speaking have a higher bone density (and therefor lower fracture risk)

African-American women have a higher bone density and a lower fracture risk (lifetime) compared to other women.

What causes hereditary fructose intolerance?

Aldolase B deficiency...disease manifests after introduction of fructose into diet with vomitting and hypoglycemia 20-30 minutes after ingestion of fructose (will have hepatomegaly & abnormal liver enzymes)

What does aldolase reductase do and what is the final product?

Aldolase reductase converts glucose into sorbitol...which is further metabolized and eventually you end up with FRUCTOSE.

What is aldosterone and what does it do?

Aldosterone is a mineralocorticoid hormone synthesized and released by the zona glomerulosa cells of the adrenal cortex. Aldosterone INCREASES the number of basolateral Na+/K+-ATPase pumps and apical sodium channels found on principal cells in the cortical collecting duct [increasing sodium and water reabsoprtion]. Aldosterone promotes potassium and hydrogen ion secretion.

What is a serum marker for Paget Disease of Bone?

Alkaline phosphatase due to osteoCLAST metastasis & paget disease of bone. Biopsy showing MOSAIC PATTERN OF LAMELLAR BONE is diagnostic for Paget Disease of Bone [initially there is an increase in osteoclastic activity]

During adult-hood BLUE-BLACK deposits on sclera & ear cartilage + arthritis. Disease and reason

Alkaptonuria = deficiency of homogenistic acid dioxygenase [Tyrosine is unable to be converted to Fumarate) TYROSINE METABOLISM PROBLEM this is the direct result of homogenistic acid dioxygenase = ACCUMULATION OF HOMOGENISTIC ACID (black urine when exposed to air; blue-black pigment on face; ochronot

Seemingly healthy kid pees, pee turns black. What disease? What reaction is impaired?

Alkaptonuria [Tyrosine is unable to be converted to Fumarate) TYROSINE METABOLISM PROBLEM this is the direct result of homogenistic acid dioxygenase = ACCUMULATION OF HOMOGENISTIC ACID (black urine when exposed to air; blue-black pigment on face; ochronotic arthropathy

Effect of volitale anesthetics on cerebral blood flow?

Almost all volitale anesthetics INCREASE cerebral blood flow...undersirable because it increases ICP...these anesthetics also: myocaridal depression, hypotension, respiratory depression and decreased renal function

Patient with straining during urination, hesitancy & intermittent urinary flow + high BP...what medicine can be given to address both?

Alpha-1 blocker (DOXAZOSIN) are useful for tx of BENIGN PROSTATIC HYPERPLASIA and hypertension

Patient presents with yellowing of the skin and shortness of breath, had prolonged jaundice as a kid but no episodes until now + liver disease. What is deficient?

Alpha-1-antitrypsin deficiency

Relationship between pure alpha-adrenergic agonists, HR & BP>

Alpha-adrenergic agonists increase systolic and diastolic blood pressure by stimulating alpha-1-adrenoreceptors in the vascular wallas, causing vasoconstriction. The elevated systemic blood pressure then causes a reflexive increase in vagal tone, resulting in decreased HR and slowed AV conduction.

Patient with acute onset of a dyspnea, hypoxemia, hypocapnia

Alveolar Hyperventillation

What is ARTERIAL PaCO2 a direct indicator of?

Alveolar Ventillation status...hypocapnia implies ongoing alveolar hyperventillation

In pulmonary secretions which cell is most likely to contain ELASTASE

Alveolar macrophages. (also infiltrating neutrophils) the proteases can cause destruction of terminal lung parenchyma when secreted in excess

23 yo asymptomatic male in research study is homozygous for apolipoprotein E-4 allele, what will he likely suffer from in the future?

Alzheimer dementia. [late onset familiar alzheimer is associated with apolipoprotein E4 genotype, Early-onset familiar alzheimer disease is associated with: APP(chrom21), presenilin 1 & presenilin 2

Chronically progressive pre-senile dementia with cortical atrophy but no other radiological/lab abnormalities permits a clinical diagnosis of what? What drug can be used?

Alzheimer's disease (diagnosis of exclusion) Current AD-specific tx: cholinesterase inhibitors (donepezil), Vitamin E (antioxidant) & NMDA receptor antagonists (memantine)

65 yo woman brought in with worstening memory, cog impairment died 2 years later from MI. Congo red staining of brain samples reveals patchy red deposits that turn yellow-green under polarized light...what was she suffering from?

Alzheimer's disease ~ AMYLOID(CONGO RED) [Beta-amyloid deposits in brain parenchyma (neuritic plaques) and cerebral vessels (amyloid angiopathy) normally folded and is associated with congo red (diagnosis of exclusion) Current AD-specific tx: cholinesterase inhibitors (donepezil), Vitamin E (antioxidant) & NMDA receptor antagonists (memantine)

Patient comes back from camping trip eating poisonous mushroom, name of mushroom and what is inhibited by the toxin?

Amatoxin (Amanita Phalloides) = potente inhibitors of RNA Pol II (halting mRNA synthesis)

What two drugs selectively bind to inactivated sodium channels and then rapid dissociate in order to suppress ventricular tachyarrthymias induced by rapidly depolarizing & ischemic myocardium

Amiodarone & lidocaine...with amiodarone being the drug of choice in acute management of ventricular arrthymias during myocardial infarction.

What is the common site of an ectopic pregnancy? What kind of tissue would uterine curettage show?

Ampulla of the fallopian tube. DILATED, COILED ENDOMETRIAL & EDEMATOUS STROMA Uterine curettage would reveal decidual changes in the endometrium due to progesterone secretion but no embryonic or trophoblastic tissue (no villi)

What accounts for the difference in minute ventilation and alveolar ventillation?

Anatomical DEAD SPACE. Minute ventillation = tidal volume x breaths per minut [product of tidal volume and RR] Alveolar ventillation = (tidal volume - dead space volume) x breaths per minute [product of RR and difference between tidal volume and deadspace volume].

What are the seronegative spondyloarthropathies?

Ankylosing Spondylitis, reactive arthritis, psoriatic arthritis & arthritis associated with inflammatory bowel disease. [individuals expressing HLA-B27 are at an increased risk for seronegative spondyloarhtopathies.

Last menstrual period was 3 months ago, vertebral compression fracture and small heart

Anorexia Nervosa

Lesion of femoral nerve would result in sensory loss where?

Anterior & medial thigh and medial leg. Weakness of quad muscles,loss of paterllat reflex & loss of sensation over anterioer and medial thigh/medial leg

Lateral ankle sprain due to inversion of the foot...what ligament?

Anterior talofibular ligament!

Patient presents with fever, cutaneous flushing, dry oral mucosa, dilated, poorly reactive pupils & confusion...what kind of overdose?

Anticholinergic toxicity...tricyclic antidepressants like amitriptyline have strong adverse effects

Early diastolic murmur best heard at left sternal border?

Aortic Regugitation

Man was told he had a murmur so he came to clinic, headbobbing and bounding peripheral pulses...what is pathology?

Aortic Regurgication = increase in total stroke volume with abrupt distension and rapid falloff of peripheral arterial pulses resulting in a wide pulse pressue. This leads to HEADBOBBING and BOUNDING PERIPHERAL PRESSURES.

Nocturnal palpitations and head bobbing, what heart pathology?

Aortic Regurgitation as a result of ABNORMAL LARGE (WIDE) PULSE PRESSURE

Patient presents with fatigue, progressive dyspnea, presyncope with exertion and cardiac murmur...what's wrong?

Aortic Valve Stenosis...EXTENSIVE VALVE CALCIFICATION WITH IMPAIRED LEAFLET MOBILITY...calcific degenration of the trileaflet valve is the most common cause of AS in developed nations. AS murmur = harsh ejection-type systolic murmur best heard at the base of the heart in the "aortic area"

Progressive fatigue, exertional dyspnea auscultation reveals a murmur best heard when patient sits up and leans forward whats up?

Aortic regugitation, high-pitched blowing descrendo diastolic murmur begins immediately after closure of aortic valve and is best heard along the left sternal border (3rd/4th intercostal space) while patient is sitting up and leaning forward with breath held in end expiration.

How can you reduce the progression of diabetic neuropathy in patients with proteinuria?

Appropriate glycemic and blood pressure control [ACE inhibitors & Ang IIReceptorBlockers are preferred due to their pressure independent anti-proteinuric effects

Fever, Meningitis signs(headache, neck stiffness), CSF normal except elevated protein

Aseptic meningitis ~ likely enterovirus (most common cause of aseptic meningitis)

This fungus produces thin septate hyphae with acute v-shaped branching.

Aspergillus Fumigatis (especially in immunosuppressed patients) - invasive aspergillosis, aspergillomas, allergic bronchopulmonary aspergillosis. MONOMORPHIC (ONLY IN MOLD FORM)

Patients with prolonged neutropenia are at risk for developing fungal infections, what are the two most common?

Aspergillus Fumigatis (especially in immunosuppressed patients) - invasive aspergillosis, aspergillomas, allergic bronchopulmonary aspergillosis. MONOMORPHIC (ONLY IN MOLD FORM) ...CANDIDA IS ALSO COMMON (WHITE STUFF THAT COMES OFF)

Two most common reasons for pulsus paradoxus in absence of signifcant pericardial disease?

Asthma & COPD [use B-adrenergics, to stimulate AC and increase cAMP]

At age 2 what should kids be able to do?

At age 2 should have a vocabulary of 200 words, be using 2 word phrases. "Your child may have a language disorder and could benefit from further testing"

Picture of a big tRNA cartoon, where does the residue most likely attach tRNA (UUU)?

At the top at the 3' end near the 'OH

Patient with cerebellar ataxia, oculocutaneous telangiectasias, repeated sinopulomnary infections & increased incidence of malignancy?

Ataxia-telangiectasia; CEREBELLAR ATROPHY...high risk of cancer due to inefficient DNA repair

What set of arteries develop the highest burden of atherosclerosis?

Atherosclerosis is a pathological process involving arterial walls and affects ALL major vascular beds including coronary, cerebral and peripheral arteries. THE LOWER ABDOMINAL AORTA AND CORONARY ARTERIES develop the highest burden of atherosclerosis.

What is the attack rate? (question possibly about a bbq and a certain # of people get sick)

Attack rate is the ratio of the number of people who contract an illness divided by the number of people who are at risk of contracting that illness.

In prospective studies a DISPROPORTIONATE loss to follow-up between exposed & unexposed groups creates potential for what?

Attrition bias (a type of selection bias)

Impaired social communication/interaction and has restricted repetitive interests or behaviors...can occur with or without language/intellectual impairement?

Autism Spectrum Disorder

What inheritance pattern does classic galactosemia have?

Autosomal recessive (25% of offspring) Classical galactosemia [deficiency of galactose-1-phosphate uridyl transferase] is the most common and most severe galactosemic disorder (days after birth: jaundice, vomitting & hepatomegaly)

Shoulder trauma, what nerve are we concerned about (won't be able to Abduct the arm)

Axillary nerve (C5, C6). SENSORY loss over lateral shoulder & weakness on shoulder Abduction...deltoid is denervated

Phase trial looking at an anti-fungal medicine that inhibits fungal colony growth by changing composition of the fungal cell membrane ...new agent inhibits liver cytP450 metabolism, what class?

Azoles(like itraconazole). Azoles inhibit the synthesis of ergosterol by the fungal cytP450 enzymes ...they also suppress human P450 system resulting in many drug-drug interactions.

At low doses atenolol is a B-1 selective antagonist, what cell type(s) effected? What is not?

B-1 antagonist (a-m) is going to effect the Heart and Renal [decreased cAMP} but will NOT effect vascular smooth muscle

What should be monitored closely after evacuation of a hydatidiform mole?

B-hCG...persistenly elevated or rising levels may signify the development of an invasive mole or choriocarcinoma

Intraabdominal infections (like appendicitis) are often polymicrobial with what 2 being the most common?

B.Fragilis & E. Coli

Kid presents with shortness of breath and palpitations +fever, +poor apetite, +fatigue...cardiac biopsy shows big purple area in the heart (Aschoff body) whats up?

BACTERIAL INFECTION - Acute Rheumatic fever, untreated group a strep (pyogenes) Mitral valve regurge, Aschoff body(purple instead of red on biopsy)~interstitial myocardial granuloma

What bug is diagnosed via KOH prep and "whiff" test? How can you treat it?

BACTERIAL vaginosis from Gardenerella Vaginalis (dx: KOH/Whiff. Tx: Metronidazole or CLINDAMYCIN

Man has a diasbling fear of evelators, new promotion requires him to work on 48th floor (once he gets on elevator he sweats profusely, feels panicky and faint) what is the most effective treatment?

BEHAVIORAL THERAPY (he has a specific phobia and CBT w/exposure is treatment of choice)

26 year old man comes to ED with chest pain, palpitations, shortness of breath & sweating. He's been there twice for samesymptoms. BP: 140/90, pulse 96/min, resp 20/min. ECG is normal patient says "I feel like I'm going to die" what is the next appropriate step?

BENZODIAZEPINE ADMINISTRATION ~ panic disorder is characterized by reccurent, unexpected panic attacks and should be considered in young, healthy adults who come to the emergency department with unexplained chest pain.

What does maternal hyperglycemia do to the beta cells of the newborn?

BETA CELL HYPERPLASIA...maternal hyperglycemia causes increased transplacental glucose delivery to the infant, fetal hyperglycemia, and BETA CELL HYPERPLASIA = fetal macrosomia & hypoglycemia after delivery.

16 year old boy dies suddenly while jogging, significant left ventricular hypertrophy...what inherited protein abnormality is responsible?

BETA-MYOSIN HEAVY CHAIN. Autosomal dominant mutations effecting cardiac sarcomere genes (beta-myosin heavy chain & myosin binding protein C gene)

Man with type diabetes has a lab analysis showing Hemoglobin A2 level of 7.5% (normal is 1.5%-3.5%) what is the likely explanation?

BETA-THALASSEMIA TRAIT Hemoglobin A2 is elevated in beta-thalassemia as beta-globin chain underproduction leads to decreased hemoglobin A synthesis.

What is the most common congenital cardiac malformation in patients with Turner Syndrome?

BICUSPID AORTIC VALVE. [early systolic high frequency click]

Autopsy on 17 year old girl: underdeveloped ovaries (composed of CT and no follicles) what else is likely to be found on exam?

BICUSPID AORTIC VALVE. [early systolic high frequency click]. Turner syndrome 45, X = Streak ovaries, Amenorrhea & infertility...Short stature, webbed neck, shield chest and low posteruir hairline =. BICUSPID AORTIC VALVE ~ MOST COMMON CARDIAC COMORBITY

45 year old lupus woman found dead, she had progressive truncal obesity, facialplethora, proximal muscle weakness, mild hyperglycemia. Autopsy: left coronary artery mainstem thrombus how will her adrenal glands likely look?

BILATERAL CORTICAL ATROPHY (longterm use of supraphysiologic doses of glucocorticoids causes suppression of the HPA axis --> bilateral adrenocortical atrophy)

Woman has multiple burns on her hands she cannot feel when things are "really hot" doesn't notice she has burned herself until she looks at her hands, PE: diminished pinprick and temp sensation across back/shoulders/arms [light touch, position, vibration all preserved] where is spinal cord damage?

BILATERAL INTERRUPTION OF PAIN AND TEMPERATURE FIBERS (lesion will be marked in the middle of the spinal cord on image) = SYRINGOMYELIA = formation of a cavity in the cervial region of the spinal cord. Damaging ventral white commissure

What is HIGHLY suggestive of Abusive head trauma in a infant?

BILATERAL RETINAL HEMORRHAGES abusive head traume can be caused by vigorous shaking of an infant and results in SUBDURAL HEMORRHAGE(due to tearing of bridging veins) and RETINAL HEMORRHAGES

Man cuts his leg, it drains purulent exudate (cellulitis) gram-positive cocci in clusters...the organism responsible synthesizes a protein as part of its peptidoglycan wall that does what?

BINDS THE FC PORTION OF IgG [protein A = virulence factor from Staph Aureus; this leads to impaired complement activation, opsonization and phagocytosis.

What is the mechanism of action of ARGATROBAN?

BINDS TO THROMBIN ACTIVE SITE

Man arrested for touching people saying he can "heal their pain" and has been up all night all week trying to end world hunger, his mood is very irritable, speech loud and rapid...what diagnosis?

BIPOLAR I DISORDER WITH PSYCHOTIC FEATURES ~ bipolarI disorder is diagonsed in patients with a lifetime history of +1 episodes of mania. Manic episodes are characterized by elevated/irritable mood, hyperactivity, decreased need for sleep, pressured speech and grandiosity and may occur with psychotic features.

Man with low grade fever/cough/malaise had outdoor adventure in great lakes. Microscope exam of KOH prep shows large yeast cell with a SINGLE BUD

BLASTOMYCES DERMATITIDIS

Patient given morphine, aspirin and alteplase...what is most likely complication?

BLEED Intracerebral or GI

Patient with a several week history of depressed mood, weight loss, death thoughts...a drug with what primary mechanism is suggested?

BLOCKADE OF SEROTONIN TRANSPORTER = mechanism of action of SSRIs

Constipation, abdominal pain, irritability, pallor. Family moved into old house 4 months ago, 3 months ago kid started having less frequent and more ball-shaped poops. What should we do?

BLOOD LEAD LEVEL. Lead poisoning can have neuro, GI, renal, hematological complications...lead poisoning should be suspected in homes built before 1978.

Hypochromic, microcytic anemia is most commonly due to an iron deficiency, what must be ruled out?

BLOOD LOSS, especialy occult loss from the GI tract must be ruled out in a patient with iron-deficiency anemia

Man being started on CANAGLIFLOZIN, what test should be ordered before the patient begins canagliflozin therapy?

BLOOD UREA NITROGEN AND CREATININE (SGLT2 inhibitors are oral anti-diabetic agents that work by decreasing proximal tubular reabsorption of glucose and thereby promoting urinary glucose loss)

Image of child with blue sclera and history of multiple fractures after only minimal trauma, what is the primary impairment of the disease?

BONE MATRIX FORMATION ~ Osteogenesis Imperfecta ~ results from defective synthesis of type 1 collagen.

What is BOSENTAN and what is it used for?

BOSENTAN is a competitive antagonist of endothelin receptors used for tx of idopathyic pulmonary arterial hypertension

Which group of bacteria is the peptidoglycan wall found in?

BOTH gram + and gram -...therefore a trial drug that inhibits peptidoglycan wall synthesis would not be resistant to gram + or gram -...only bactieria like mycoplasma [since they lack peptidoglycan walls; gotta use a anti-ribosomal agent]

Patient coughing up phlegm/blood. Has right-sided face & arm swelling...engorgement of subcutaneous veins on same side...what vein is obstructed?

BRACHIOCEPHALIC VEIN (drains ipsilateral subclavian & jugular veins ~ similar to SVC syndrome but only on one side of the body

Woman with facial swelling and difficulty breathing, takes lansoprazole and linopril. PE: swelling of lips and tongue with audible stridor without wheezing...what is the mechanism?

BRADYKININ ACCUMULATION ~ causes of angioedema include: mast cell activation and (les commonly) excess bradykinin which can come from ACE inhibitors or C1 inhibitor deficiency. Angioedema is a rare/serious adverse effect of ACE INHIBITOR THERAPY (ACE inhibition INCREASES BRADYKININ)

Woman with pain and mass in right flank, mass removed and is composed of fat, smooth muscle and blood vessels..what else is likely to be seen?

BRAIN HAMARTOMAS & ASH-LEAF SKIN PATCHES ~ Renal angiomyolipoma is a benign tumor composed of blood vessels/smooth muscle/fat...tumors can be diagnosed with abdominal CT and are associated with TUBEROUS SCLEROSIS(hamartoms in brain and seizures)

Man is started on SIDENAFIL...the intracellular signaling of that drug is similar to that of what substance?

BRAIN NATRIURETIC PEPTIDE....ANP and BNP have transmembrane receptors linked to GUANYLYL CYCLASE...cGMP..Phosphodiesterase inhibitors (sidenafil) decrease degradation of cGMP = smooth muscle relaxation and vasodilation

Man got letter from his job saying he has significant exposure to asbestos....what type of malignancy is he likely to develop?

BRONCHOGENIC CARCINOMA...bronchogenic is more common than mesothelioma

Man with one week history of headaches and progressive confusion, had viral esophagitis, PNEUMOcystic Pneumonia, latex agglutination test is positive for soluble polysaccharide antigen. LM of CSF is most likely to reveal:

BUDDING YEAST ~ cryptococcus neoformans (causes meningoencephalitis in HIV(+) patients. Latex agglutination test detects the polysaccharide capsule antigen of Cryptococcus and is used for diagnosis (INDIA INK SHOWS ROUND BUDDING YEASTS)

Woman comes to office for health maintenance exam: irregular menses, smoker, enlarged parotid glands bilaterally, teeth have eroded enamel....abnormal electrolytes. What is going on?

BULLEMIA NERVOSA an eating disorder that can have patients develop bilateral parotid gland enlargement, erosion of tooth enamel, irregular menses & abnormal electrolytes.

old man with skin blisters on trunk and groin, image shows entire cellular layer & basement membrane pulled up, what is diagnosis?

BULLOUS PEMPHIGOID ~ characterized by autoantibodies against HEMIDESMOSOMES along the basement membrane of the dermal-epidermal junction. This causes entire epidermis to separate from the dermis, forming tense, subepidermal blisters

Man w/previous splenectomy, presents with fever, chills, fatigue, dyspnea. CXR: bilateral infiltrates, vector responsible can also transmit what?

Babesiosus via Ixodes tick can also transmit Borrelia Burgdorferi [co-infection is common]

What does the concentration of Fructose 2,6 Bisphosphate levels control?

Balance between gluconeogenesis & glycolysis. This is done through inverse regulation of PFK-1 & Fructose 1,6 bisphosphatase. F2,6BP activates PFK-1[increasing glycolysis] and inhibits fructose 1,6 bisphosphatase [decreasing gluconeogenesis]. INSULIN ALSO ACTIVATES PFK-2 -->INCREASED FRUCTOSE 2,6 BISPHOSPHATE LEVELS AND DECREASED CONVERSION OF ALANINE (AND OTHERS) TO GLUCOSE.

Squamous metaplasia is reversible and an adaptive response to irritants, for barret's esophagus what is the change?

Barret's esophagus = esophageal sqaumous epithelium is replaced by columnar epithelium in response to chornic acid exposure. [although reversible, persistent irritant exposure can lead to dysplasia or squamous cell carcinoma]

What is the CAAT box and where is it?

CAAT (-75, it Is first) It is a transCRIPTION promotor (crip cat!)

What psoriasis drug activates a nuclear transcription factor?

CALCIPOTRIENE = Vitamin D analog binds to Vitamin D reveptor and INHIBITS KERATINOCYTE PROLIFERATION and stimulates differentiation

Signal transduction study applies a substance to human Gprotein dependent phospholipase C...what intracellular substance is likely to increase as a result?

CALCIUM (IP3 DAG pathway)

What substance is most likely responsible for increasing the reaction rate of glycogenolysis within myocytes during active skeletal muscle contraction?

CALCIUM ...synchronization of glycogen degradation with skeletal muscle contraction occurs due to release of sarcomplasmic calcium following neuromuscular stimulation. Increased intracellular calcium causes activation of PHOSPHORYLASE KINASES, stimulating glycogen phosphorylase to increase GLYCOGENOLYSIS

Patient with significant blood loss from injuries gets fresh packed blood and has a reaction where his fingers and toes are tingling, what is happening?

CALCIUM CHELATION BY A SUBSTANCE IN THE TRANSFUSED BLOOD. The patient's paresthesias are due to low calcium. An entire body worth of blood within 24 hours may result in increased levels of CITRATE. CITRATE CHELATES CALCIUM & MAGNESIUM and may reduce their plasma levels, causing paresthesias.

In order for presynaptic vesicles to release their contents into the synpatic cleft, what substance is required?

CALCIUM...acetycholine release from presynaptic terminal vesciles at the neuromuscular junction depends upon the influx of extracellular calcium into the presynaptic terminal. Calcium influx into the nerve terminal occurs following neuronal depolarization and opening of voltage-gated calcium channels

8 year old with two day history of fever, abdominal pain and diarrhea kid's dog had diarrhea a week earlier and kid is UTD on vaccinations...no ova or parasites in stool...what is cause of diarrhea?

CAMPYLOBACTER infection is a common cause of inflamamtory gastroenteritis and can be acquired from domestic animals or contaminated food....inflammatory diarrhea accompanied by fever, abdominal pain and tenesmus (GBS)

3 week old baby brought to ED: fever, irritable, poor feeding. CSF: white cells, protein, glucose. Culture: Gram- rods pink colonies on MacConkey agar...what is the most important virulence factor?

CAPSULE ~ E Coli...frequent cause of neonatal meningitis via K1 capsular antigen. (K1 allows bacteria to survive in bloodstream and establish meningeal infection)

Combination of: JVP, hypotension, muffled heart sounds is suggestive of what?

CARDIAC TAMPONADE ~ tachycardia and pulsus paradozus are frequently seen with tamponade.

man comes to office with chronic cough fro smoking and has to begin oxygen therapy, upon beginning a nasal cannular oxygen supplementation the patient's RR reduced, this is due to decreased stimulation by what receptors?

CAROTID BODIES ~ PaCO2 is the major stimulator of respiration in healthy people, even a slight increase in PaCO2 results in increased pulmonary ventillation. In people with COPD the response to PaCO2 is blunted and hypoexmia becomes an important contirbutor to the respiratory drive. Peripheral chemoreceptors are primarily responsible for sensing arterial paO2 and can be suppressed via oxygen

Lesion found on non-smoking woman's lung, what would favor diagnosis of hamartoma?

CARTILAGE TISSUE IN THE BIOPSY SAMPLE...solitary lung nodule (coin lesion). Benign lung tumor is usually a hamartoma (CARTILAGE, adipose, fibrous tissues)

tRNA is non-coding, and a specific form was isolated for further analysis what is likely to be found on the 3' end of the molecule?

CCA ~ tRNA is small/noncoding form of RNA that contains chemically modified bases. tRNA has a CCA sequence at its 3' end that is used as recognition sequence by proteins. The 3' terminal hydroxyl group of the CCA tail serves as the amino acid binding site.

Leukocyte adhesion deficiency is due to the absense of what? How does it present?

CD18 antigens necessary for the formation of integrins. Leukocyte Adhesion Deficiency is caused by failure of leukocyte chemotaxis and include reccurrent skin & mucosal infections without purulence, delayed separation of the umbilical cord & persistant leukocytosis.

2 year old boy with recurrent sinopulmonary infections, giardia lamblia gastroenteritis, failure to thryive and low IG are suggestive of what? What will a blood smear show a deficiency of?

CD19; X-LINKED AGAMMGLOBULINEMIA (XLA) mutation in BRUTON TYROSINE KINASE failure of bone marrow pre-b cells (CD19+, CD20+) to develop into mature B lymphocytes

Known host receptor and viron binding specficities include:

CD4 ----HIV gp120; CD21 ---- EBV gp350; erythrocyte P antigen ---- parvovirus b19

Woman works as a bird keeper has dyspnea & dry cough, her serum ACE and Calcium are elevated...what type of cells will be found in bronchoalveolar lavage and what pathology?

CD4+ LYMPHOCYTES ~ Sarcoidosis. Sarcoid granulomas produce ACE and active vitamin D (1,25) so patients often have ELEVATED ACE & HYPERCALCEMIA

What cells controll the pulmonary tuberculosis infection by containing the pathogen?

CD4+ TH1 lymphocytes & macrophages...the cells work together to contain M. Tuberculosis within a caseous granuloma (which offers macrophages inside an opportunity to kill the remaining organisms if necrotic area is small enough)

What is the mechanism of damge done on liver by Hep B>

CD8+ T LYMPHOCYTE RESPONSE TO VIRAL ANTIGENS ON THE CELL SURFACE RESULTING IN HEPATOCYTE DAMAGE~ hep b virus does not have cytotoxic effect itself, howeverm the presence of viral HBsAg & HBcAg on the cell surface stimulate the host's cytotoxic CD8+ T lymphocytes to destroy infected hepatocytes

Burn victim infected with gram-neative rods that are oxidase positive and non-lactose fermenting...what is the tx to get started?

CEFEPIME ~ Pseudomonas should come to top of differential (major pathogen in burn victims)

Man with 2 days of left knee swelling & pain, had facial palsy 3 months ago...6 months ago went hiking in new hampshire, what drug could have prevented the patient's knee condition?

CEFTRIAXONE ~ he has Lyme disease = borrelia burgdorferi = ixodes tick..there is also ECM = target lesion = erythema chronnicum migrans

Amphotericin toxicities include nephrotoxicity, HYPOkalemia, HYPOmagnesia...what mechanism does it exert damage on kidney>

CELL MEMBRANE CHOLESTEROL DAMAGE

Thrombus is extracted in a DVT patient, then there is an increase in Serum Creatine Kinase...best explanation for CK increase?

CELL MEMBRANE DAMAGE

Pale retina + Cherry Red macula + ACUTE presentation in one eye

CENTRAL ARTERY OCCLUSION

65 year old male, bilateral visual difficulty...fundoscopy shows small yellow retinal lesions clustered in MACULA, what part of his visual field is messed up?

CENTRAL SCOTOMAS (scotoma is a visual field defect that occurs due to a pathologic process that involves parts of the retina or the optic nerve resulting in a discrete area of altered vision surrounded by zones of normal vision. LESIONS OF THE MACULA CAUSE CENTAL SCOTOMAS.

If there is a dilated third ventricle and nromal sized fourth ventricle where is the blockage?

CEREBRAL AQUADUCT OF SYLVIUS ~ obstruction at this level can cause dilated lateral and third ventricles with a normal-sized fourth ventricle. (magendie & luschka causes enlargement of all 4 ventricles)

What can cystic fibrosis impact as far as reproduction?

CFTR MUTATIONS ARE THE MOST COMMON CAUSE OF CONGENITAL BILATERAL ABSENCE OF THE VAS DEFERENS...patients with CBAVD have AZOOSPERMIA & infertility but normal levels of FSH, LH & testosterone. [elevated sweat chloride = cystic fibrosis]

Woman comes back from brazil to hospital with fever and skin rash...has headache, retro-orbital pain, high-grade fever & joint & muscle pains +epistaxis her boyfriend is fine. What disease has a similar mode of transmission?

CHIKUNGUNYA FEVER & DENGUE FEVER = AEDES AEGYPTI mosquito

What medications should be avoided in someone taking a CNS depressant like diazepam/benzodiazepine?

CHLORPHENIRAMINE ~ 1st gen H1-histamine receptor antagonists, including diphenhydramine and chlorpheniramine, can cause signficant sedation, especially when used with other medications that cause CNS depression (such as benzos)

What is the only thiazide diuretic that doesn't share the suffix and how would this relate to increased muscle cramping?

CHLORTHALIDONE is a thiazide and thiazide diuretics decrease intravascular fluid volume, which stimulates aldosteroner secretion and leads to excretion of K+ & H+ in urine. This HYPOKALEMIA can cause cramping and metabolic alkalosis

Fat woman has sever epigastric pain after heavy meal, related to fatty food consumption...what hormone likely provoked the current attack?

CHOLECYSTOKININ (CCK) is the hormone responsible for gallbladder contraction. It is made in the duodenum and jejunum in response to fatty acids and amino acids.

Somatostatin (secreted by pancreatic delta cells) decrease secretion of(secretin, cholecystokinin, glucagon, insulin & gastrin) suppression of what hormone is likely responsible for biliary stones?

CHOLECYSTOKININ (CCK) is the hormone responsible for gallbladder contraction. It is made in the duodenum and jejunum in response to fatty acids and amino acids.

Blue toe, livedo reticularis w/normal peripheral pulses following an invasive proceduse are signs of what? What histopathological finding would be seen on biopsy of kidney?

CHOLESTEROL CLEFTS IN ARTERIAL LUMEN, Atheroembolic disease where cholesterol-containing debris get dislodged from larger arteries

Increased cardiac output (increased height of cardiac function curve); decreased total peripheral resistance; increased mean systemic pressure...what could causes these things?

CHRONIC ARTERIOVENOUS FISTULA (this shunt would increase cardiac output because of increased sympathetic stimulation to the heart, decreased TPR and increased venous return)

32 year old woman with worsening shortness of breath, arterial blood shows: PaO2 normal. %saturation normal, Oxygen content: low...what is the likely cause?

CHRONIC BLOOD LOSS ~ anemia is characterized by decreased hemoglobin concentration in the setting of normal SaO2 & PaO2

Black gallstones in a woman from Nepal who recently immigrated to the US, what condition predisposed her?

CHRONIC HEMOLYSIS...black pigment stones arise from conditions that increase the amount of unconjugated bilirubin in bile, which promotes CALCIUM BILIRUBINATE production. Can occur in chronic hemolysis: Sickle Cell, B-thalassemia, hereditary spherocytosis. and increased enterohepatic cycling of bilirubin

The presence of erythroid precursors in liver/spleen means what?

CHRONIC HEMOLYSIS...erythroid precursors is indicative of extramedullary hematopoiesis, a condition characterized by erythropoietin-stimulated, hyperplastic marrow cell invasion of extramedullary organs. (b-thalassemia can be a cause)

What is on the differential in a man with previous EBV infection who now has purulent nasal discahrge and lymphadenopathy with a low leukocyte alkaline phosphate score?

CHRONIC MYELOGENOUS LEUKEMIA definitive diagnosis of CML requires demonstration of the philadelphia chromosome t(9;22) = BCR-ABL fusion gene or mRNA

Woman 8 months out from lung transplant presents with a reduced FEv1 and normal FVC, biopsy shows areas of total fibrotic obstruction in terminal bronchioles....what caused patient's condition?

CHRONIC TRANSPLANT REJECTION...major problem in lung transplant patients; it effects SMALL airways, causing bronchiolitis obliterans. Characterized by lymphocytic inflammation, fibrosis and destruction of the bronchioles

Woman with recent falls/accidents seems to have glaucoma and is started on timolol, what structure is the target of that medication?

CILIARY EPITHELIUM

7 year old injures himself while playing in the yard, wound cultures grow Clostridium Tetani, patients never gets tetanus why?

CIRCULATING ANTIBODIES THAT NEUTRALIZE BACTERIAL PRODUCTS. C tetani produces the protein exotoxin tetanospasmin that blocks release of inhibitory neurotransmitters from inhibitor motor interneurons in CNS...tetanus is prevented by immunization with toxoid that triggers production of antitoxin anitbodies (ACTIVE IMMUNITY)

Man comes to office due to 3 weeks of malaise & fatigue, cardiac sounds: apical holosystolic murmur radiating to the axillas (not heard on previous visits) +proteinuria/hematuria w/red cell casts...what is most likely pathogenesis?

CIRCULATING IMMUNE COMPLEX-MEDIATED INJURY. The most likely cause of fever/fatigue with new-onset murmur is infectious endocarditis...diffuse, proliferative glomerulonephritis secondary to circulating immune complex deposition may complicate IE and result in renal insufficiency

girl scared of doctors, now sweats when she sees her nursing roomate's stethascope whats up?

CLASSICAL CONDITIONING ~ neutral stimulus being repeatedly paired with a non-neutral stimulius that elicits a reflexive, unconditioned response.

Kid has decreased appetite, fever and ear discharge....he's had otitis media 3x this year, you prescribe amoxicillin and add clavulanic acid, why?

CLAVULANIC ACID DECREASES AMOXICILLIN CLEAVAGE BY BACTERIAL CELLS

41 year old man comes to ED with fever, weight loss & productive cough...alcoholic, foul-smelling sputum sample is sent for gram stain and culture. What antibiotic would be the most appropriate ?

CLINDAMYCIN ~ best coverage against ANAEROBIC ORAL FLORA that ALCOHOLICs are susepctible (BACTEROIDES, PREVOTELLA, FUSOBACTERIUM, PEPTOSTREPTOCOCCUS) these are antibiotics of choice for tx lung abscesses

Transplant patient with worstening shortness of breath, non-productive cough, low-grade fevers...rna or dva virus? Enveloped or nah?

CMV = ENVELOPED, DOUBLE-STRANDED DNA virus (herpesviridae family) +Owl's Eye inclusion

What is the most common cause of retinitis in HIV+ patients? And what drug should be started?

CMV ~ Gancyclovir

Two formulas for cardiac output:

CO = SV x HR ||| Fick CO = rate of O2 consumption / arteriovenous O2 content difference

52 year old male with low-grade fevers after replacement of his aortic valve, blood cultures grow gram-positive cocci (Staph Epidermidis) what are the characteristics?

COAGULASE NEGATIVE catalase positive, novobiocin SENSITIVE

Woman is treated with high-dose folic acid for anemia, she returns to the clinic complaining of bilateral foot numbness and difficulty walking...why are those symptoms present?

COBALAMIN DEFICIENCY = Vitamin b12 deficiency....the patient taking folate would only improve megaloblastic anemia. Would need Vitamin B12 if she has megaloblastic anemia and neurological dysfunction [treatment of a B12 deficiency with folate instead can actually worsten neurological dysfunction]

28 year old man with severe depression and suicidal ideation, had vivid disturbing dreams and is irritable, withdrawn, hypersomnolent and hyperphagic...extensive history of substance abuse and is lethargic and dysphoric....symptoms like due to withdrawl from what?

COCAINE WITHDRAWAL ~ acute depression accompanied by fatigue, vivid dreams,hypersomnia and hyperphagia

Woman with skin easily brusied, members of her family have a very flexible body.....inherited defect in what protein?

COLLAGEN = a major component of connective tissue; collagen = 3 polypeptide alpha chains held together by hydrogen bonds. Lysyl oxidase forms covalent bonds between individual tropocollagen molecules. EDS Ehlers Danlos Syndrome...don't forget hemarthrosis

Woman comes in because she is on a drug that treats heart failure, her potassiumlevel is low so she is given a medication that works on kidneys...what part of kidneys?

COLLECTING DUCT - Potassium sparing diuretics works on collecting duct (only diuretics that DO NOT cause potassium loss)

Man with previous TB now has blood-tinged sputum, CT scan shows an old upper lobe cavity with a new round mass, what is best way to describe patient's condition?

COLONIZING ~ Aspergillus fumigatus (mold) opportunistic in immunosuppressed and neutropenic patients. COLONIZING when it forms a fungus ball within a preexcisting lung cavity

What is symmetrical enlargment of the ventriculi characteristic for? And what does this occur secondary to?

COMMUNICATING HYDROCEPHALUS ~ARACHNOID GRANULATIONS .....hydrocephalus (communicating) usually occurs secondary to dysfunction or obliteration of subarachnoid villi...dysfunction is usually a sequelae of meningeal infection

Most likely outcome for seurm positive HBsAg patient?

COMPLETE RECOVERY

Skin biopsy reveals, uniform round cells at the basal portion of the epidermis that extend into the dermis?

COMPOUND MELANOCYTIC NEVUS

Hypotonic baby with "floppiness" and poor feeding has Large anterior fontanelle, large tongue and reducible umbilical hernia + CONSTIPATION?

CONGENITAL HYPOTHYROIDISM ~ asymptomatic at birth but once moms t4 runs out infant develops constipation, lethargy, hypotonia, maccroglossia, large anterior fontanelle

Marijuanna stimulates CB1 & CB2 receptors, what are the characteristic signs of marijuanna use?

CONJUNCTIVAL INJECTION(RED EYES), tachycardia, increased apetite & dry mouth

woman comes in with severe pain and swelling in her right knee & left albow/wrist....labs/PE normal. Joint aspiration reveals opaque exudate...what could have prevented symptoms?

CONSISTENT CONDOM USE...asymmetric poly arthritis, Neisseria Gonorrhoeae are faculative intracellular organisms seen within neutrophils...DIG = disseminated gonococcal infection

What is most likely to decrease renal plasma flow & increase filtration fraction?

CONSTRICTION OF THE EFFERENT ARTERIOLE will impede blood flow through the kidney, this will decrease RPF but increase glomerular hydrostatic pressure as the fluid "backs up" in the glomerulus. FF = GFR/RPF

Previously depressed patient presents with difficulty swallowing, dry mouth, blurred vision, mydriasis&poorly reactive pupils...whats up?

CONSUMPTION OF HOME-CANNED FOODS...clostridium botulinum toxin, nicotinic(diplopia/dysphagia)&muscarininic(dry mouth)

What is the required course of action of you suspect a physican is drunk (even taking orders on call)?

CONTACT THE COLLEAGUE'S ON CALL SUPERVISOR TO REPORT THE PROBLEM NOW ...ethically and legally required to report impaired colleagues in a timely manner

Woman thinks she has UC, what finding(s) would most suggestion UC in this patient?

CONTINUOUS MUCOSAL INVOLVEMENT ~ Rectum is ALWAYS involved in US, Inflammation is limited MUCOSA & SUBMUCOSA ONLY, MUCOSAL DAMAGE IS CONTINUOUS, no areas of normal mucosa between affected segments. TOXIC MEGACOLON is a feared complication...sever dilation of the bowel (UC increases risk of adenocarcinoma of the colon)

Man cuts his hand, 3 weeks later actin-containing fibroblasts & increased metalloproteinases are at site of injury...why?

CONTRACTURE

How does rasburicase work to protect normal organs during chemo?

CONVERTING URIC ACID INTO MORE SOLUBLE METABOLITES

Smoking patient with: dyspnea, tachypnea, prolonged expiration and bilateral wheezing what is going on? What is immediate tx?

COPD exacerabtion; tx: B-adrenergic agonist...produce relaxation of bronchial smooth muscle by stimulating beta-2 adrenergic receptor (GsPCR activates AC increasing cAMP concentration

4 year old kid...fever that has persisted for 5 days, vomiting, bilateral conjunctival injection, posibly asian, tongue is bright red, nonpitting edem on hands/feet...what complications Is patient at risk for?

CORONARY ARTERY ANEURYSM ~ KAWASAKI DISEASE = vasculitis of medium-sized arteries that presents with persistent fever for >5 days, bilateral conjunctivitis, cervical lymphadenopathy & mucocutaneous involvement. CORONARY ARTERY ANEURYSMS ARE SERIOUS COMPLICATIONS OF KAWASAKI DISEASE.

woman gets chest pain during moving, has history of hypertension and MI in family, blood pressure in right arm is 146/85...left arm is 142/80..no hear murmur, what's cause?

COSTOCHONDRITIS due to costosternal syndrome (anterior chest wall syndrome) occurs after repetitive activity and is characterized by pain that is reproducible with palpatation and worstened with movement or changes in position.

Fishbone lodged in the piriform recess, what reflex can be damaged by removing the fishbone?

COUGH reflex (vagus - internal branch of superior laryngeal) [remember gag reflex is 9]

What anti-inflammatory agent does NOT impair platelet aggregation and can be given

COX-2 inhibitors (CELECOXIB) have anti-inflammatory effects without the side effects of bleeding and gastrointestinal ulceration associated with non-selective COX inhibitors. Platelets selectively express COX-1, so COX-2 specific is fine!

Increased activity of what enzyme has been found to increase many forms of colon adenocarcinoma?

COX-2, so COX-2 inhibitors (like regular aspirin) have been shown to lower rates of colonic adenoma and adenocarcinoma.

What is formed on light microscopy when anti-glomerular basement membrane antibodies react with collagen type 4?

CRESCENT FORMATION...rapidly progressive glomerulonephritis + GLOMERULAR CRESCENT FORMATION on LM...immunoflourescense showing linear deposits of IgG & C3 along glomerular basement membrane is characteristic.

Zenker diverticulum shown on image, what is the cause of the chocking spells dysphagia cough and bad breath?

CRICOPHARYNGEAL MOTOR DYSFUNCTION...diminished relaxation of cricopharyngeal muscles during swallowing results in increased intraluminal pressure in the oropharynx

26 year old woman, intermittent abdominal pain occuring over last several years, pain crampy without radiation +fluctuating diarhea; there is a draining FISTULA near her coccyx...what is the likely diagnosis?

CROHN DISEASE ~ typically presents with insidious onset of abdominal pain, diarrhea and constitutional symptoms (weight loss, fever)...lesions affect the entire thickness of the bowel wall...perianal disease is also common

What is the term for people who have chronic excess glucocorticoid?

CUSHING SYNDROME ~HYPERPLASIA OF FASCIULATE LAYER OF THE CORTEX. ACTH is the major trophic hormone of the zona fasciulata and reticularis ...prolonged ACTH stim causes HYPERPLASIA OF ZONA FASCIULATA AND RETICULARIS...resulting in excessive cortisol production.

Industry worker presents with rapid-onset shortness of breath, dizziness, palpitations and flushed skin after accidental exposure to chemical fume, reddish skin discoloration, patient instructred to inhale amyl nitrate...the success of this antidote depends on ability to convert hemoglobin into a form with increased affinity for what?

CYANIDE...nitrates are oxidizing agents that are effective in treating cyanide poisoning due to their ability to induce methemoglobinemia. Methemoglobin contains ferric (Fe3+) rather than ferrous iron (Fe2+) ...cyanide binds to ferric iron more avidly than to mitochondrial cytochrome enzymes, diminishing cyanide's toxic effect.

Man comes to ER with severe dizzziness and confusion had chest pain took nitroglycerin. Med: aspirin, acetominophen(sometimes), tadalafil...BP 50/20 HR 120 BPM...what cellular changes are responsible for patient's symptoms?

CYCLIC GMP ACCUMULATION interaction between nitrates and phosphodiesterase inhibitors used in ED is important to understand. Double whammy of created cGMP and stopping the stoppers (PDEinhibitors) = profound system HYPOtension [contraindication alert]

Linear ulcerations on esophagogastroduodenoscopy in someone with frequent pneuomcystis jirovecii pneumonia?

CYTOMEGALOVIRUS ~ infectious esophagitis is common in patients with HIV. The most common cause is candida albicans, although cytomegalocirus and herpes simplex virus are also frequently implicated. Diagnosis is based on endoscopic and microscopic findings.

Patient with polycythemia vera, what kind of receptor is messed up?

CYTOPLASMIC TYROSINE KINASE RECEPTOR ~ polycythemia vera is a myeloproliferative disorder characterized by uncontrolled erythrocyte production. Virtually all patients with polycythemia vera have a mutation in JAK2, a non-receptor (cytoplasmic) tyrosine kinase associated with the erythropoietin receptor

Patient with gout and history of peptic ulcer disease (so NO NSAIDs) is taking a medicine that causes diarhea with persistent nausea...the drug used most likely affects which of the following cell structures?

CYTOSKELETON ~ Colchicine binds to intracellular tubulin and inhibits its polymerization into microtubules.

Adherens junctions & Desmosomes use what kind of protein?

Cadherins

Woman has complete thyroidectomy, no immediate complications, what medication should be started now?

Calcitriol (post-operative hypoparathyroidism can occur following thyroidectomy and cause significant hypocalcemia). Oral calicum and vitamin D can help treat/prevent postoperative hypocalcemia.

What can be used to assist in the prevention of cerebral vascular spasm following subarachnoid hemorrhage?

Calcium channel blockers (specifically Nimodipine) can be used to assist in the prevention of cerebral vascular spasm following SAH

What class are calcium-sensing receptors? What disorder can result from them being messed up?

Calcium sensing receptors are transmembrane G-Protein coupled (metabotropic) receptors. Binding of calcium to CSR will inhibit release of PTH...low calcium leads to increased PTH. FAMILIAL HYPOCALCURIC HYPERCALCEMIA is an autosome dominant disorder caused by defective calcium-sensing receptors PT gland & kidneys

Bilateral loss of muscle in lower extermities w/absent deep tendon reflex + bout of diarrhea, what bug?

Campylobacter Jejuni can lead to Guillan Barre Syndrome (demyelination of the peripheral nerves ASCENDING MUSCLE WEAKNESS AND PARALYSIS)

What is the most common cause of opportunistic mycosis?

Candida Albicans (yeasts and pseudohyphae on light microscopy & Positive GERM TUBE TEST)

What does capacity to refuse treatment require? (seemingly crazy patient is somehow still involved in making decision)

Capacity to refuse treatment requires the ability to express a choice, understand relevant medical information, understand consequences and offer a rationale for decision. Patients who have this capacity have the right to refuse any medical treatment,

What is the name for an arrangement in which a payor (individual/employer/government) pays a fixed, predetermined fee to cover all the medical services required by a patient?

Capitation (CAPITATION is the payment structure in HMOs)...the fee is FIXED

Patient presents with headache & her husband has a headache. They were burning fireplace, what is responsible for symptoms and how does it work?

Carbon monoxide, COMPETITIVELY & reversibly binds to heme [creating carboxyhemoglobin] and can present as headache/dizziness. The competitive/reversible binding shifts oxygen dissociation curve to the LEFT.

Subungal splinter hemorrhages should give you a sign that what's going on?

Cardiac Pathology. Vegetations of bacterial endocarditis...possible new onset murmur

Carotid Sinus Afferent & Efferent limbs (man buttoning his shirt keeps passing out)

Carotid Sinus Hypersensitivity (afferent=CN9(Herring Nerve); efferent=CNX(Parasympathetic))

Patient presents with discomfor & swelling in right axilla, on PE there is enlarged tender axillary lymph node + several scratch marks on right arm. Pathogen and other associated condition

Cat Scratch(self-limited) ~ Bartonella Henslae ~ Bacillary Angiomatosis (BA)

Decreased sensation in right upper face, ptosis, mydriasis, loss of corneal reflex...where is the lesion?

Cavernous Sinus thrombosis (sometimes caused by Staph Aureus), CN: III, IV, VI, V(1), V(2)

What is the most common benign liver tumor?

Cavernous hemangioma [cavernous blood-filled vascular spaces; lined by a single epithelial layer]

Relationship between celiac disease and vitamin D?

Celiac disease, small intestine becomes inflammed and atrophies in response to gluten = malabsorption of vitamin D. Serum Ca(decreased). Serum phosphorus(decreased). Serum PTH(increased)

What is the primary mechanism underlying giant cell arteritis (big ass temporal artery on forehead)/

Cell-mediated immunity is the primary mechanism underlying giant cell arteritis. IL-6 is an important driver of this process and correlates with the severity of the symptoms.

Chest x-ray in acute decompensated heart failure will show what?

Cephalization of pulmonary vessels, perihilar alveolar edema (batwing distribution) and blunting of costophrenic angles due to pleural effusions

How do chancroids present, are they painful?

Chancroid due to Haemophilus ducreyi [dx via gram stain] presents as deep/painful MULTIPLE ulcers with ragged borders that are associated with a grey exudate and inguinal lymphadenopathy

Only a few bands are detected using radioautography (down from many)...resistance is occuring between strep pneumonia and ceftriaxone, how is this possible?

Change in protein structure. A change in protein structure of PBPs can prevent cephalosporin binding and is a mechanism of bacterial resistance to cephalosporin

4 year old kid, reccurent skin & respiratory infections +light skin, +silver hair +horizontal nystagmus....what's up?

Chediak-Higashi syndrome = albinism, immunodeficiency, neuro defects. CHEDIAK HIGASHI SYNDROME IS AN AUTOSOMAL RECESSIVE DISORDER OF NEUTROPHIL PHAGOSOME LYSOSOME FUSION

Old man having breathing problems at night, image shows airflow & abdominal/thoracic movements lined up. Type of breathing and pathology?

Cheyne-Stokes breathing, The most likely cause is Congestive heart failure. [CSB is commonly seen with CHF] --> Cyclic breathing where apnea is followed by gradually increasing then decreasing tidal volumes until the next apneic period

2 groups divided by exposure, 2x2 table...what statistical method is best?

Chi-square test for independence is used to test the association between 2 categorical variables.

5 year old Patient's family moved from Nepal, PE: grayish pharyngeal exudate partial soft palate paralysis. How does the exotoxin work?

Child has DIPTHERIA(corynebacterium diptheriae) the A(active) half of AB toxin INTRACELLULAR PROTEIN RIBOSYLATION = INACTIVATES ELONGATION FACTOR-2 (this inhibits protein synthesis and leads to cell death)

STD characterized by initially painless ulcers with later progression to painful inguinal lymphadenopathy & ulceration

Chlamydia Trachomatis (serotype L1-L3 lymphogranuloma venereum painless ulcer --> painful inguinal lymphadenopathy). Chlamydial inclusion bodies are seen in host cell cytoplasm.

What makes up the most common type of gallstone? What is the best non-operative treatment?

Cholesterol gallstones are #1 (can be composed of calcium salts, bilirubin & mucin) TX is hYDROPHILLIC Bile Acids (uresodeoxycholic acid) decrease biliary cholesterol secretion and increase biliary bile acid concentration...improving cholesterol solubility.

Woman has dull abdominal pain that occurs mainly after eating, pain is localized to upper right quadrant (especially after fatty meals) how will levels of: Cholesterol, Bile acids & Phosphatidylcholine look?

Cholesterol(UP), Bile Acids(DOWN), Phosphatidylcholine(DOWN)...elevated cholesterol concentrations increase the likelihood of cholesterol precipitation and gallstone formation. (BILE ACIDS AND PHOSPHATIDYLCHOLINE MUST BE DOWN FOR GALLSTONES TO FORM)

What disease is classically seen in those who are "fat, female, forty, fertile" (brought on by the fertile one)

Cholesthiasis (gallstone disease): ESTROGEN-induced cholesterol hypersecretion and PROGESTERONE-induced gallbladder hypermotility

What drug can be given as monotherapy to help a person INCREASE TRIGLYCERIDE LEVELS

Cholestyramine, bile acid-binding resins inhibit the enterohepatic circulation of bile acids. INCREASE triglycerides.

What is the most common eye related problem that can be passed to a child during pregnancy by a primary CMV infection?

Chorioretinitis (most common eye problem), sensorineural deafness, seizures, jaundice, hepatomegaly, splenomegaly, microcephaly

What are the cells released as a result of pheochromocytoma and what other cells do they share an embryologic origin with?

Chromaffin cells of adrenal medulla & Parafollicular cells are both derived from neural crest cells along with: MOTEL PASS (melanocytes, odontoblasts, trachael cartilage, enterochromaffin cells, laryngeal cartilage, PARAFOLLICULAR cells of thyroid, adrenal medulla & ganglia, schwann cells & spiral membrane.)

Relationship between Chron's disease and easy bruising?

Chron's typically involves the terminal ileum where bile acids are reabsorbed, if the terminal ileum is inflamed or removed (chron's) then bile acids are lost with the feces and fat-solble vitamins (KEDA) are lost leading to deficiency. Vit K deficiency can result in prolonged bleeding and easy brusing

Thickened bronchial walls, lymphocytic infltration, mucous gland enlargement & patchy squamous metaplasia of bronchial mucosa...

Chronic Bronchitis, cigarette smoking is #1 cause of chronic bronchitis. [BEHAVIORAL]

Dihydrorhodamine flow cytometry reveals absence of green flourescence characteristic of normal neutrophils, condition & impaired enzyme

Chronic Granulomatous Disease (CGD) inactivating mutation of affecting NADPH oxidase [x-linked]. Tests include: Blue Nitrobluetetrazolium test & Green Dihydrorhodamine flow cytometry

What is obesity hypoventillation syndrome?

Chronic fatigue, dyspnea, difficulty concentrating and evidence of hypoventillation (patial pressure of CO2 greater than 45mmHg while awake). Hypoxemia with a normal alveolar to arterial oxygen gradient.

If HBeAG persists for several months and host anti-HBeAg remain low or undetectable levels suspect what?

Chronic hepatitis B infection with high infectivity

Symptomatic management of PAD (peripheral artery disease) includes what two things:

Cilostazol (a PDE-inhibitor that inhibits platelet aggregation & acts as a direct arterial vasodilator) & a graded EXERCISE PROGRAM

47 year old woman, progressive joint pain and swelling. Stiff in the AM, proximal IP joints, metacarpal phalangeal joints & wrists bilaterally...serum antibodies would react with what?

Citrullinated Peptides anti-CCP have a high specificity for Rhreumatoid Arthritis

What HLA class does Ankylosing Spondylitis fall under?

Class I - HLA-B27 which are expressed by ALL NUCLEATED CELLS (hence class 1) and present endogenous antigens to CD8+ cytotoxic T cells.

Which peptides are displayed ONLY by anitgen presenting cells?

Class II MHC peptides, so if the acidification of lysosomes with antigen presenting cells is prevented impaired interaction with T lymphocytes will occur upon antigen exposure

What class of anti-arrythmic drugs predominantly block potassium channels and inhibit outward potassium currents during phase 3 of the cardiac action potential?

Class III antiarrythmic drugs (amiodarone, sotalol, dofetilide) predominantly block potassium channels and inhibit the outward potassium currents during phase 3 of the cardiac action potential...thereby prolong repolarization & total action potential duration

Male infant, normal genitalia 1-2 weeks after birth with vomitting, hypotension, hyponatremia, hyperkalemia

Classic salt-wasting 21-hydroxylase deficiency (deficient cortisol & aldosterone synthesis)...adrenal androgen overproduction

Tight junctions use what protein?

Claudins/Occludins

Decreased skin turgor, sunken in eyes, tight jaw muscles. Bug & what neurotransmitter release is inhibited?

Clostridium Tetani (tetanus). Metaloprotease exotoxin (tetanospasmin). Retrograde axonal transport. GLYCINE & GABA [regulate lower motor neurons] release is PREVENTED = muscle spasms and hyperreflexia

What type of necrosis does the heart undergo?

Coagulative necrosis (develops after irreversible ischemic injury in most tissues) most common variant.

12 year old girl, pain in her legs, intercostal vessels are pulsatile and femoral pulses are diminished....what condition are her symptoms likely associated with?

Coartation of the aorta from Turner Syndrome 45,X...loss of PATERNAL X CHROMOSOME

Subacute combined degeneration is what kind of vitamin deficiency and where in the spinal cord is the lesion?

Cobalamin = Vitamin B12 deficiency = subacute combined degeneration (demyelination). The Cobalamin(B12) deficiency may result from pernicious anemia, gastrectomy, ileal resections or the TAPEWORM (DIPHYLLOBOTHRIUM LATUM) and strict vegan diets over 4-5 years. DORSAL COLUMNS (loss of vibration sensation, position in space), SPINOCEREBELLAR TRACT (ataxia), LATERAL CORTICOSPINAL TRACT(spastic paresis)

Cocaine exerts its effect at the same place as what other pharmaceutical? Where is this plac?

Cocaine inhibits reuptake of monoamines(norepi, epi,dopa,serotonin) resulting in sympathetic stim...shares site of action with TCAs.

Image showing sphreules (larger than surrounding RBCs) with some rupturing and some empty...what bug?

Coccidioides immitis

Patient with gout (BUT PREVIOUS PEPTIC ULCER DISEASE) gets meds that cause nausea and diarrhea, what has been given?

Colchicine (because peptic ulcer disease is contraindicated for NSAID)

Angiogram shows right lower lobe pulmonary artery occlusion, why is there no tissue necrosis?

Collateral Circulation ~ lung infarction is a rare complication of pulmonary embolism due to the collateral blood flow supplied by the pulmonary and bronchial arteries. (if pulmonary blocked, bronchial can supply)

Patient presenting with polydipsia, polyuria and low urine specific gravity after 8 hours of water restriction...also being treated for bipolar, where in the nephron is messed up?

Collecting duct...lithium-induced diabetes insipidus is the result of lithium's antagonizing effect on the action of vasopressin on principal cells within the collecting duct system.

What nerve is vulnarable to injury following a fibular neck fracture?

Common peroneal nerve = weakness of dorsiflexion(deep peroneal nerve) & eversion(superficial peroneal nerve) as well as LOSS OF SENSATION OVER THE DORSUM OF THE FOOT

What is splitting?

Commonly seen in borderline personality disorder, splitting is like all or none. One day person X is awesome, next day person X is worst person alive.

Changes in the Emax/ED curve based on competitive vs noncompetitive?

Competitive = change Ed50=shift right. Competitive you can keep increasing to outcompete. Non competitive = change Emax = Shift down

What is Pica ?

Compulsive consumption of nonfood and/or non-staple food (crushed ice) it is common in pregnancy and often seen in association with IRON DEFICIENCY ANEMIA. Pregnant patients should be assessed throughout pregnancy for the presence of pica, especially if they have unexplained weight loss

Patient's history of fire setting, stealing and aggressive behavior is most likely explained by what disorder?

Conduct disorder...persistent pattern of violating major societal norms or rights of others (aggression toward animals/people, theft) ANTISOCIAL disorder CANNOT be diagnosed under 18.

Difference between conducting and sensorineural hearing loss?

Conducting hearing loss = bone conduction will be greater than air conduction (Abnormal Rinne test) and Webere test will lateralize to AFFECTIVE EAR. In SENSORINEUAL HEARING LOSS, air condction will be greater than bone condution (normal rinne test) and weber will lateralize to the unaffected ear.

Difference between pneumococcal conjugate vaccine and polysaccharide vaccine?

Conjugate vaccine (given to babies) induces a more robust immune response throught B & T cell activation.

Normal Sodium, Low potassium, High bicarbonate + High blood pressure at a low age. Whats up?

Conn syndrome (most common cause) ~ primary hyperaldosteronism...from a unilateral adrenal adenoma. Bilateral adrenal hyperplasia is also a possibility.

Gap junctions use what proteins?

Connexins...essential in contractions. Connexin density increases in uterus before delivery

DiGeorge (CATCH-22)

Conotroncal cardiac defects (tetralogy of fallot, truncus arteriosus, interrupted aortic arch); Abnormal Facies Thymic hypoplasia/aplasia (t-cells messed up) Craniofacial (cleft lip) Hypocalcemia/hypoparathyroidism

What is the most common persistent side effect of the first line medicine for chronic cancer pain?

Constipation. [opiods are the first line treatment for chronic cancer pains]. Opiods stimulate the Mu receptors in the gastrointestinal tract, decreasing secretions and gastric motility. Adequate fluid intake and daily laxatives are suggested for prophylaxis.

Right temporal lobe lesion ~ meyer's loop injury, would result in what kind of visual impairment?

Contralateral Superior Quadrantanopia

How do adenosine and dipyridamole work via coronary steal?

Coronary arteriolar dilation: Collateral microvessels form adjacent pathways for blood flow to areas distal to an occluded vessel. Coronary steal makes blood flow in ischemic areas get reduced due to ARTERIOLAR VASODILATION IN NONISCHEMIC AREAS

Type II pneumocytes produce dipalmitoylphosphatidylcholine (DPPC a type of lethicin) which decreases alveolar surface tension...premature infants are at risk of surfact deficiency, what class of drugs helps?

Corticosteroids such as DEXAMETHOSONE can accelerate maturation of type 2 pneumocytes

Man with advanced HIV complains of headache & fever, india ink stain is used on CSF...spherical yeasts w/thick capsules. Bug & system most effected?

Cryptococcus neoformans fungus associated with CNS present in soil & pigeon droppings. LUNGS major organ effected.

What can be used to reverse serotonin syndrome?

Cyproheptadine

Delta 508 mutation and dry mucous membranes, what is going on and what is the TRUE issue?

Cystic Fibrosis [ATP-gated], deltaF508 mutation in the cystic fibrosis transmembrane conductance regulator protein...kids sweat profusely but lose chloride AND SODIUM. The loss of SODIUM is where the symptoms come from. HYPONATREMIA due to excessive salt wasting from sweat

The Dibasic amino acids (COLA) have a defective transporter in what disease?

Cystinuria (cytosine, ornithine, lysine, arginine)...defective transporter results in impaired renal & intestinal absorption of the COLA amino acids ~ can lead to KINDEY STONES = NEPHROLITHIASIS

What is the mitochondrial enzyme that activates caspases bringing about cell death? Intrinsic or extrinsic pathway?

Cytochrome C is a mitochondrial enzyme that activates caspases and indirectly brings about cell death through intrinsic pathway apoptosis

Patients facing a terminal illness (Elizabeth Kubler-Ross Model of Greif):

DABDA - Denial, Anger, Bargaining, Depression, Acceptance. "we can hope for the best but for now lets maximize the remaining time with your children"

28 year old pregnant woman comes to OBGYN her periods have been irregular and she smokes...decreased AFP level on screening, what is the likely etiology for decreased AFP level?

DATING ERROR ~ maternal serum quadruple screen is performed to assess risk of congenital defects in fetuses,,,accurate dating is important for determining whether levels of alpha-fetoprotein and other analytes are abnormal for gestational age (patients with irregular menses are at risk for inaccurate pregnancy dating).

Infant/child with liver and muscular involvement: hypoglycemia, hepatomegaly, ketoacidosis...muscle weakness and hypotonia, abundant quantities of multibranched polysaccharide...what enzyme is likely deficient?

DEBRANCHING ENZYME ~ Cori disease ~ leads to accumulation of glycogen with abnormally short out chains (limit dextrins) due to inability to degrade alpha-1,6-glycosidic branch points Patients present with hypoglycemia, ketoacidosis, hepatomegaly & muscle weakness and hypotonia.

New patient wants to go on a date with the doctor, what is appropriate response?

DECLINE THE INVITATION, EXPLAINING THAT DATING A PATIENT IS ALWAYS UNETHICAL

Kid with ADHD is prescribed methylphenidate...what should the parents be educated about? (side efx)

DECREASED APPETITE & WEIGHT LOSS~ decreased appetite and insomnia are the most common adverse effects of psychostimulant medications used to tx ADHD. They are usually mild and can be managed

Patient having a panic attack, what is the likely cause of the patient's symptoms?

DECREASED ARTERIAL PARTIAL CO2 TENSION. Panic attacks are associated with hyperventilation and decreased pCO2

Patient has new gallastones after an extensive small bowel resection due to bowel ischemia...why?

DECREASED CHOLECYSTOKININ RELEASE DUE TO LACK OF ENTERAL STIMULATION ~ the absence of normal enteral stimulation in patients receiving total parenteral nutrion leads to decreased cholecystokinin release, biliary stasis and increased risk of GALLSTONES.

Alcohol induced hepatic steatosis can be related to what?

DECREASED FREE FATTY ACID OXIDATION secondary to excess NADH production by the two major alcohol metabolism enzymes [alcohol dehydrogenase & aldehyde dehydrogenase]

Man with exertion retrosternal angina during physical activity is given a drug sublingually and reports rapid relief of pain, how did the drug improve his symptoms?

DECREASED LEFT VENTRICULAR END-DIASTOLIC VOLUME...nitrates exert their effect by direct vascular smooth muscle relaxation = vasodilation of peripheral VEINS & ARTERIES (not arterioles) DECREASED LV WALL STRESS(DUE TO REDUCED PRELOAD) ===DECREASED MYOCARDIAL OXYGEN DEMAND

Man has shortness of breath that worstens when he is laying flat in bed, needs to prop himself up at night to sleep well.....has crackles and & s3, what factors account for dyspnea?

DECREASED LUNG COMPLIANCE...in left ventricular failure, fluid accumulation in the lung interstitium results in decreased compliance.

34 year old woman has vulvar itching and vaginal discharge does not use condoms, microscope reveals budding cells....what is the most likely underlying cause of condition?

DECREASED NUMBER OF GRAM-POSITIVE BACTERIA IN THE VAGINA ~ gram+ bacateria are a normal part of vaginal flora and changes can lead to an overgrowth of yeast (commonly Candida) especially with antibiotic use and high estrogen levels, diabetes, immunosuppression.

What biochemical process is messed up from tissue hypoxia during septic shock>

DECREASED OXIDATIVE PHOSPHORYLATION...causing shunting of pyruvate to lactate

MOA of digoxin, what is the initial cellular triggering event>

DECREASED SODIUM EFFLUX ~ digoxin directly inhibits Na-K-ATPase pump in myocardial cells, leading to a decrease in sodium efflux and an increase in intracellular sodium levels. This reduces the activity of sodium-calcium exchanger, causing increased intracellular calicum and resulting in improved myocyte contractility and LV systolic function

PATIENT has bell's palsy and what is the one symptom of facial nerve palsy you cannot forget because you know they will asK?

DECREASED TEARING FROM THE RIGHT EYE = PARASYMPATHETIC INNERVATION TO LACRIMAL, SUBMANDIBULAR, AND SUBLINGUAL GLANDS

Pregnant woman has hypotension while supine, why?

DECREASED VENOUS RETURN ...pregnant women 20+ weeks gestation can experience compression of the IVC by the uterus while supine. This REDUCES VENOUS RETURN and cardiac output, which can result in hypotension and syncope.

Man has congestive heart failure and a dilated left ventrical heart is shown in the image...what will the contractile function show?

DECREASED VENTRICULAR CONTRACTION FORCE IN DILATED CARDIOMYOPATHY

Woman has nausea, abdominal pain, dizziness after a surgery and she has lupus and takes prednisone...compared to the normal physiologic response to surgery, this patient most likely has which of the following patterns of HPA-axis activity?

DECREASED: CORTICOTROPIN-RELEASING HORMONE; ACTH; CORTISOL ~ suppression of the HPA axis by glucocorticoid therapy is the most common cause of adrenal insufficiency.

Weakness confusion with fruity breath odor, patient is most likely deficient in a hormone that normally performs what actions?

DECREASES GLUCAGON SECRETION ~ insulin ~ Guy is in DKA (type1) ~ normal blood glucose levels are maintained by insulin & glucagon. Insulin suppresses glucagon release

21 year old having a seizure and has another is taken to hospital where IV lorarepam is given and another drug is given to prevent another seizure from coming, describe its mechanism of action?

DECREASES SODIUM CURRENT IN CORTICAL NEURONS ~ status epilepticus is a single seizure lasting greater than 5 minutes or multiple with incomplete recovery. TX: lorazepamn and PHENYTOIN ~ reduces ability of sodium channels to recover from inactivation

What would increase the ratio of forward flow volume to regurgant flow volume in patient with mitral regurgitation?

DECREASING LEFT VENTRICULAR AFTERLOAD

MID-shaft fracture of the humerous and wrist drop...what artery?

DEEP BRACHIAL ARTERY ~ the deep brachial artery and radial nerve run together along the posterior aspect of the humerus. MIDSHAFT FRACTURE = RISK OF INJURY TO RADIAL NERVE & DEEP BRACHIAL

Man comes to hospital, right leg injury after cycling accident, surgery competed but now there is a severe pain in right lower leg...increased tension within the anterior compartment what structure is compromised? Nerve

DEEP PERONEAL NERVE the patient has ACUTE COMPARTMENT SYNDROME ~ anterior compartment of the leg which includes the foot extensor muscles, anterior tibial artery & DEEP PERONEAL (FIBULAR) NERVE

15 year old difficulty with movement, significant kyphosis and high plantar arch...brother suffered from it and died, what neuropathologic findings?

DEGENERATION OF SPINOCEREBELLAR TRACTS ~ FRIEDREICH ATAXIA cerebellar ataxia from spinocerebellar tract degeneration & loss position/vibration sensation(dorsal column) HYPERTROPHIC CARDIOMYOPATHY ...FXN GENE

REVERSIBLE acute-onset confusional state characterized by a fluctuating level of consciousness with deficits in attention, memory & executive function?

DELIRUM

Two weeks after getting a cardiac transplantation patient is suffering from dyspnea on exertion what findings would be most consistent with acute graft rejection?

DENSE INTERSTITIAL LYMPHOCYTIC INFILTRATE ~ acute cardiac transplant rejection occurs weeks following transplantation and is primarily a CELL-MEDIATED PROCESS. On histolofy there is DENSE MONONUCLEAR LYMPHOCYTIC INFILTRATE with cardiac myocyte damage.tx: IMMUNOSUPPRESSIVE DRUGS

17 year old girl spaces out a lot, in her own world...had breakup recently but experienced spacing out before and after breakup...cooperative with midly anxious mood/flat affect. Diagnosis?

DEPERSONALIZATION/DEREALIZATION DISORDER. Depersonalization = feelings of detachment from or feelings of observing yourself). Derealization = experiencing surroundings as unreal.

60 year old man, dyspnea on exertion, gained 30 pounds over a few months. PE: darkening of skin, proximal muscle weakness. CXR: lung mass. Labs: elevated serum cortisol and adrenocorticotropic hormone levels what studies would be helpful in diagnosing?

DEXAMETHASONE SUPPRESSION TEST. Cushing syndrome caused by a pituitary adenoma or ectopic ACTH secretion will present with elevated ACTH levles. High-dose dexa suppression test will suppress ACTH and cortisol levels when cushing's is caused by a pituatry adenoma (cushing disease) but NOT when it is caused by ectopic ACTH (small cell carcinoma)

20 year old woman comes to ER due to intractable vomiting and abdominal pain, has several week history of anorexia and weight loss (has hypothyroidism and has not taken her levothyroxine in 2 days due to vomitting) She is started on high-flow IV saline what else should be given?

DEXAMETHASONE she is in acute adrenal insufficiency (adrenal crisis) in addition to fluids she needs immediate glucocorticoid supplementation

An individual effected with a point mutation responsible for NEUROPHYSIN synthesis will likely suffer from what?

DIABETES INSIPIDUS (neurophysins are carrier proteins for oxytocin and ADH=vasopressin. Point mutations in NEUROPHYSIN II underlie most cases of hereditary hypothalamic DIABETES INSIPIDUS = a disease resulting from insufficient ADH release into the systemic circulation.

67 year old man with fatigue, weakness and itching...renal arterioles have deposition of homogenous, glassy material in subendothelial space that stains pink with PAS strain, what condition?

DIABETES MELLITUS - chronic fatigue, weakness, itchiing...uremia toxicity 2/2 chronic kidney disease. Hyaline arteriosclerosis seen in patients with HTN or diabetes.

Man is given an interscalene nerve block for rotator cuff surgery, what other muscle not in the upper limb will be paralyzed?

DIAPHRAGM~ so the nerve block should be avoided in patients with chronic lung disease or with contralateral phrenic nerve dysfunction.

1 day old dx with hyperphenylalanemia, placed on phenylalanine-restricted diet with tyrosine supplementation...months later he has normal serum phenylalanine level. Neuro abnormalities include axial hypotonia & microcephaly...elevated prolactin what cofactor deficiency?

DIHYDROBIOPTERIN REDUCTASE ~ patient likely has deficiency of dihydrobiopterin reductase based on combo of hyperphenylalanemia & elevated prolactin. This situation counts for 2% of PHENYLKETONURIA

Patient with rest&nighttime angina with transient ST elevation has what? What triggers this?

DIHYDROERGOTAMINE (trigger) VARIANT/PRINZMETAL ANGINA(disease) ~ prinzmeta angina is characterized by episodic, transient attacks of coronary vasospasm (at rest and at night), producing temporary transmural myocardial ischemia with ST-segment elevation. Triggers: smoking, cocain, triptans/DIHYDROERGOTAMINE. TX: tobacco/drug cessation and vasodilator therapy

Patient uses methrotrexate to terminate gestational sac, what substance will likely accumulate in embryonic tissue as a result?

DIHYDROFOLATE POLYGLUTAMATE. MTX competitive inhibits DHF reductase which normally catalyzes the synthesis of a tetrahydrofolate. DHF reductase inhibition will result in ACCUMULATION OF DHF

Male with AML underwent successful infuction chemo with doxorubicin, comes to office complaining of progressive exertrional dyspnea & orthtopnea...what is responsible for patient's symptoms?

DILATED CARDIOMYOPATHY (-rubicin = anthracycline chemotherapeutic agents form free radicals in the myocardium) Cumulative dose-related DILATED CARDIOMYOPATHY -> L&R ventricular CHF

Old woman taking 12 different medications has frequent dizziness and is disoreinted to time/place. What should be discontinued and why?

DIPHENHYDRAMINE ~ beers criteria = common medications to avoid in older adults. Beers criteria include: anticholinergics(1 gen antihistamines); alpha blockers; tricyclic antidepressants; benzodiazepines (and other sedating medications), antipsychotics, most antiarrythmics & skeletal muscle relaxants.

Where does the lowest osmolarity (~100 mOsm/L) occur in the kidney?

DISTAL CONVOLUTED TUBULES ~ dehydration stimulates Adh secretion, Adh acts on collecting ducts increasing their permeability to water. DCT & ThickAL have most dilute water ~ the metanephric mesoderm (blastema) gives rise to the glomeruli, Bowman's space, proximal tubules, the loop of Henle & Distal convoluted tubules.

If there is an insult that interferes with the structures formed by the meteanephric mesoderm, what adult derivatives will fail to develop?

DISTAL CONVOLUTED TUBULES ~ the metanephric mesoderm (blastema) gives rise to the glomeruli, Bowman's space, proximal tubules, the loop of Henle & Distal convoluted tubules.

Woman comes in with lump, breast cancer diagnosed. Her mom had ovarian cancer and her aunt had breast cancer...the patient most likely inherited a mutation normally responsible for what?

DNA REPAIR...hereditary breast cancer is most commonly associated with mutations in BRCA1 & BRCA2...these tumor suppressor genes are involved in DOUBLE STRANDED DNA REPAIR and the mutation increases risk of developing breast and ovarian cancer. AUTOSOMAL DOMINANT

How does therapeutic ionizing radiation (x-rays, gamma rays) work?

DNA double-strand breakage: breakage of both strands (single can be repaired) or Free radical formation reactive oxygen species are formed by ionization of water (causing cell & DNA damage).

For DNA polymerase 3 what is moving 5'->3' and what is moving 3'->5'?

DNA pol 3 (5'->3' DNA synthesis) (3'->5' exonuclease PROOFREADING (REMOVAL OF MISMATCH BASEPAIRS DURING DNA REPLICATION]

Only DNA pol I has 5'->3' exonuclease (DNA POL II & DNA POL III have 3'->5' exonuclease(proofing) what does the DNA pol 1 exonuclease do?

DNA pol I 5'->3' exonuclease is used to remove RNA primer synthesized by RNA primase.

What is unique about DNA Pol I & prokaryotes?

DNA pol I is the only prokaryotic polymerase that also has 5'->3' exonuclease is used to remove RNA primer synthesized by RNA primase, in ADDITION to 5'->3' polymerase and 3'->5' exonuclease.

Patient is drunk and has acute pancreatitis, he says he wants to go home...what should you do?

DO NOT ALLOW THE PATIENT TO LEAVE THE ED AND REASSESS WHEN SOBER

Graph showing increased renal blood flow and cardiac output..both are increasing, what drug will do this?

DOPAMINE

HLA class 2 diseases include:

DR/DP/DQ (DaRheumatoid) DR-4 rheumatoid arthritis, type 1 diabetes mellitus, celiac disease. Class 2 HLA are expressed on antigen-presenting cells: macrophages/dendritic cells

Woman complains of double vision of 1 day, shining light into her eye causes constriction of her right pupil but not her left, shining in her left causes constriction of only her right pupil....what else is likely to be found in this patient?

DROOPING OF THE LEFT EYELID (optic nerve2 is repsonsible for AFFERENT, oculomotor nerve3 is responsible for EFFERENT)...levator palpebra is innnervated by 3 hence lacking

32 year old man with pericardial fluid accumulation and late diastolic collapse of the right atrium, what physical exam finding is most likely to be seen?

DROP IN PULSE AMPLITUDE DURING INSPIRATION ~ patient has pericardial effusion with associated cardiac tamponade. (beck's triad: hypotension, elevated JVP, muffled heart sounds) DECREASED SYSTOLIC PULSE PRESSURE DURING INSPIRATION

10 year old girl bloating, diarrhea and flatulence, short stature & weight loss...what is suspected, what should be biopsided?

DUODENAL BIOPSY. Celiac disease, small intestine becomes inflammed and atrophies in response to gluten = malabsorption of vitamin D. Serum Ca(decreased). Serum phosphorus(decreased). Serum PTH(increased). IgA against tissue transglutamase.

Two people given the two different forms of polio vaccine, what will be the measurable difference in immunoglobulin?

DUODENAL IgA will be different. Live attenuated oral SABIN produces a stronger mucosal secretory IgA immune response than the inactivated Salk vaccine...Mucosal/Duodenal IgA will be measurable difference

Non-invasive urease breath test 13C-Labeled Urea is an evaluation for what condition?

DUODENAL ULCERS due to H. Pylori...13C labeled Urea is used to monitor for the presnce of 13C labeled carbon dioxide (indicated the presence of H pylori product urease in the stomach)

Guy undergoes stress testing (Cardiac) what is the most significant factor limiting coronary blood supply during the test?

DURATION OF DIASTOLE

4 year old boy brought to ED for difficulty breathing and a productive cough, PMHx of recurrent episodes of sinusitis and otitis media. Cardiovascular exam shows a cardiax PMI that is palpated in the right 5th intercostal space. What is dysfunctional?

DYNEIN ARMS ~ kartegner's syndrome ~ primary ciliar dyskinesia: situs inversus, chronic sinusitis & bronchiectasis. It occurs due to mutations that impair the structure or function of cilia. Cystic fibrosis also causes chronic respiratory infections but its NOT associated with situs inversus

Oral direct thrombin inhibitor that inactivates both circulating & clot-associated thrombin

Dabigatran (has the same uses as the factor Xa inhibitors.

Prolonged exposure to loud noises causes hearing loss due to damage of what specifically in the ear?

Damage to stereociliated hair cells of the ORGAN OF CORTI

Woman allergic to vanco gets on MRSA medication and has muscle pain and increased CPK, what is drug and how does it work?

Daptomycin MAINTENANCE OF MEMBRANE POTENTIAL [it causes depolarization of bacterial cellular membrane and inhibition of DNA, RNA and protein synthesis.]

Patient with dyspnea, orthopnea, ankle swelling --> nausea, vomitting, headache + Pulmonary crackles and pitting edema, given Nitroprusside. What is wrong and how will it impact pressure-volume curve

Decreased afterload and preload...Nitroprusside can be given in hypertensive heart failure. It is a short acting balanced venous and arterial vasodilator that decreases both preload and afterload. Since CHANGE IS BALANCED = STROKE VOLUME IS MAINTAINED

Young obese patient with respiratory symptoms and central obesity (with no history of pulmonary disease) = obesity related-restrictive lung disease, how will his pulmonary testing look?

Decreased: FEV1, FVC, ERV, TLC Normal: RV [when you're fat everything is down but the RV]

What is a Charcot-Bouchard aneurysm rupture

Deep intraparenchymal hemorrhage is most commonly caused by hypertensive vasculopathy of the small penetrating branches of the cerebral arteries.

What enzyme is responsible for most cases of congenital adrenal hyperplasia (CAH)?

Defiency of 21-hydroxylase woman with ambigious genitalia (males initially clinically silent until salt-wasting is identified)

Acute onset "confusional state" characterized primarily by waxing and waning mental status changes and impaired attention is describing what medical term, what is the treatment of choice?

Delrium...low-dose antipsychotics (haloperidol) are the meds of choice to tx behavioral and psychotic manifestations of delirium

Difference between schizophrenia and delusional disorder?

Delusional disorder +1 delusions for +1 months...NO OTHER PSYCHOTIC SYMPTOMS. Behavior is not bizarre and functioning is NOT significantly impaired (apart from delusions)

What is required to convert G-3-P to 1-3BPG in glycolysis? (and if missing could be a cause of severe muscle weakness in a kid)

Depleted NAD+

What can be used as treatment for mild hemophilia A? how does it work?

Desmopressin works by increasing circulating factor 8. Classic manifestations of hemophilia A include: hemarthrosis, delayed and prolonged bleeding

Immigrant hispanic female patient comes in with belly pain, doc says its food poisoning but patient later collapses and a ruptured ectopic pregnancy is diagnosed. What step could have prevented the delay in diagnosis?

Determining whether the patient needs language assistance.

Diabetic autonomic neuropathy is common in what type of diabetes and how does it usually present?

Diabetic autonomic neuropathy is common in type 1 diabetes and can cause overflow incontinence due to inability to sense a full bladded & incomplete emptying. PVR (post-void residual) testing with an ultrasound or cath can confirm inadequate bladder emptying.

Dizygotic twins: explain # of amnions and chorions?

Diamnionic (2) Dichorionic (2). Dizygotic twins result from fertilization of two oocytes by two different sperm. ALWAYS have two amnions & two chorions

During inspiration the diaphragm is responsible for the fall what what pressure? When that fall is reduced where is the source of insult?

Diaphragmatic contraction is responsible for most of the intrapleural pressure fall during inspiration...if the contractions are becoming weaker with no decrement in phrenic nerve stimlation there is pathology at the NEUROMUSCULAR JUNCTION.

Nondihydroyridine calcium channel blockers can be used in the setting of atrial fib, what are common side effects?

Diltiazem & verapamil (nondihydroyridine CCBs) can cause: constipation, bradycardia, AV conduction block (negative chronotropic effect) and worstening of heart failure in patients with reduce left ventricular function (negative ionotropic effect)

How does diptheria toxin hurt (unimmunized ~ immigrant patients)

Diptheria Extotoxin inhibits host cell protein synthesis by catalyzing the ADP-ribosylation oh host cell elongation factor 2. Corynebacterium diptheria are non-motile, encapsulated, gram-positive rods...on microscopic examination they are found in clumps (like chinese characters) METACHROMIC GRANULES STAIN WITH ANALINE DYES (LIKE METHALINE BLUE)

Patient with hypertension enrolls in new drug study where long-acting medication causes direct relaxation of smooth muscle of artioles but does not effect veins. How?

Direct arteriolar vasodilators lower blood pressure but trigger reflex sympathetic activation and stimulate the renin-angiotensin-aldosterone axis = tachycardia and edema [SODIUM & FLUID RETENTION]

Doctor messes up an order but the patient ends up being ok, what should the doctor say/do to patient?

Disclose the error with an explanation and apologize.

What can happen if there is disruption of the tuberoinfundibular pathway?

Disruption of tuberoinfundibular pathway by D2 receptor blocking antipsychotics may cause hyperprolactinemia with galactorrhea and amenorrhea.

How do you calculate the case-fatality rate?

Divide the number of fatal cases by the total number of people with the DISEASE (that specific disease like MRSA deaths / people with MRSA)

What is closed-loop communication and how does it prevent error?

Doctor says order to nurse and he/she legit says the exact same thing back. Doctor will then confirm!

How does the HepB virus replicate?

Double-stranded DNA -> +RNA template -> dsDNA progeny [although HBV is a DNA virus, it replicates via reverse transcription]

Acyclovir (a guanosine analog) works on HSV1/2 & VZV but not EBV and CMV, why>

Drug phosphorylation rate...monophosphorylation of acyclovir by a viral thymidine kinase is the first (and rate-limiting) step in conversion of acyclovir to more powerful active triphosphate form.

Schizophrenic patient taking risperidone brought to office due to breast tenderness and no menstrual period for 3 months, what is the cause?

Drug-induced amenorrhea ~ drug-induced hyperprolactinemia due to risperidone (inhibits D1 & D2 receptors and inhibits serotonergic and alpha-adrenergic pathways) ELEVATED PROLACTIN LEADS TO AMENORRHEA

Two anti-neoplastic drugs are shown to inhibit thymidylate formation, DrugX can be overcome by N5forymlTetrahydrofolate and DrugY is not affected...what are the two drugs?

DrugX(Methotrexate) DrugY(Fluorouracil) ....both inhibit thymidylate formation, but the chemotherapeutic effects of methotrexate is overcome by (folinic acid, leucovorin ~ n5formyl-tetrahydrofolate)

Explain properties of drugs with a high blood/gas partition coefficient?

Drugs with high blood/gas partition coefficients are more soluble in the blood, demonstrate slower equailibration with the brain, and have longer onset times.

How can risk of "wrong site" surgery be reduced?

Dual identifiers (have surgeon and head nurse INDIVIDUALLY confirm incision site)

Why are right-sided pressures lower than left sided pressures in the heart?

Due to lower resistance in the pulmonary vasculature. Right ventricular diastolic pressure is similar to right atrial/central venous pressure

How is the third heart sound best heard?

During early diastole with the bell of the stethoscope over the cardiac apex while the patient is in the left lateral decubitus position at end expiration.(can be a normal finding in healthy young adults)

Elderly patients with dementia or hemiparesis may have WHAT risk factor for aspiration pneumonia?

Dysphagia! (dense air opacities in the superior region of the right lower lobe).

What is a sustained, involuntary muscle contraction, which forces a part of the body into abnormal, sometimes painful movements/postures?

Dystonia

What is the most common cause of UTI?

E Coli ( Escherichia Coli) most common cause of UTI in both health and old adults...it is a NORMAL PART OF GI FLORA. Special adhesive proteins called fimbriae allow some strains to colonize and ascend the urinary tract [uti, pyelonephritis, or bacteremia] UTI most common cause of E Coli bacteremia

What bug can cause HUS and how does HUS present?

E Coli O157:H7 can cause HUS, which normally presents in children under 10 years old and is commonly contracted after eating undercooked ground beef

Kid has apical displacement of the tricuspid valve leaflets, decreased right ventricular volume & atrialization of the right ventricle...what condition did his mom have?

EBSTEIN'S ANOMALY ~ mom has BIPOLAR DISORDER = TOO MUCH LITHIUM (teratogenic -> ebstein's anomoly)

patient accidentally write 20 instead of 2.0 makes it past pharmacists and nurses, what is the best way to fix this problem?

EDUCATE PHYSICIANS TO AVOID USE OF TRAILING ZEROS ..avoiding use of unsafe abbreviations and trailing zeros in medication orders can help reduce the incidence of medication errors

Woman taking a drug says she still has bouts of depression but takes double on "bad days" and skips on "good days" appropriate response?

EDUCATE THE PATIENT ABOUT THE RISKS OF IRREGULAR DOSING (physicians should always address misconceptions)

Woman with hypothyroidism and diabetes has a low Vit B12 & normal folate level...how will he gastrin, gastric pH & parietal cell mass look?

ELEVATED GASTRIN, HIGH GASTRIC PH & DECREASED PARIETAL CELL MASS

What is the confirmatory test for for menopause?

ELEVATED SERUM FSH CONFIRMS MENOPAUSE DIAGNOSIS ~ menopause is defined by the permanent cessation of menses for 12 months.

Patient has numbness and tingling in her feet, small ulcer on right foot not healing, bilateral absense of ankle reflexes...neuro symptoms are associated with what?

ENDONEURL ARTERIOLE HYALINIZATION ~ neuropathy from diabetes mellitus is caused by microangiopathy which leads to nerve ischemia.

What is the cellular response to UV damage?

ENDONUCLEASE NICKING OF THE DAMAGED DNA STRAND (or covalnt bonds between pyrimidine dimers) ~ mutations in nucleotide excision repair cause XP a condition characterized by severe photosensitivty and development of skin cancers at a young age.

Patient has atherosclerosis what cell type is involed in the first step in the pathogenesis?

ENDOTHELIAL CELLS

Patient with history of allograft transplatation has linear,shallow ulceration in the lower esophagus. What is the most likely finding of a biopsy sample?

ENLARGED CELLS WITH INTRANUCLEAR INCLUSIONS ~ CMV esophagitis can occur in immunosupprssed (transplant) odynophagia, dysphagia accompanied by fever/burning chest pain.

What is the only HIV polyprotein product that becomes glycosylated?

ENV gene polyprotein product is glycosylated to gp160 and proteolytically cleaved within the endoplasmic reticulum and Golgi apparatus to form the envelope glycoproteins gp120 & gp41

smear shows: Bite cells and heinz bodies

ENZYME DEFICIENCY ANEMIA (G6PD)

What are the most effective lipid-lowering drugs for primary and secondary prevention of cardiovascular events regardless of baseline levels

ENZYME INHIBITOR = HmG-CoA reductase inhibitor (statin)

Elevated CA-125 & BRCA mutation in mom, what is likely diagnosis? What is PROTECTIVE of this?

EOC epithelial ovarian cancer [due to frequency of trauma and repair to the ovarian surface. ORAL CONTRACEPTIVES, MULTIPARITY & BREASTFEEDING ARE PROTECTIVE BY DECREASING THE FREQUENCY OF OVULATION (risk factors: infertility, nulliparity, brca mutation

Question asking about cell type with a protein that has reverse transcriptase activity adding TTAGGG repeats to the 3' end of chromosomes?

EPIDERMAL BASAL CELLS protein = telomerase, adds TTAGGG repeats to the 3' end of chromosomes (telomere region)...uses RNA-dependent DNA polymerase. Stem cells have VERY LONG TELOMERES due to high telomerase activity

What are clue cells (how do they look under microscope) what bug?

EPITHELIAL CELLS WITH GRAM-VARIABLE RODS ~ bacterial vaginosis; Gardenerella Vaginalis [metronidazole & clindamycin tx)

Keratin is a marker of cells of what origin?

EPITHERLIAL SURFACES...keratin is a marker of epithelial cell origin

Man comes in with high PSA, adenocarcinoma is confirmed and surgery takes place..during the surgey the nerves in the fascia surrounding the prostate gland are injured, what is the most likely consequence?

ERECTILE DYSFUNCTION

What is suspected in an elderly woman with jaw claudication(ache) and an episode of amaurosis fugax(transient monocular visual loss)? What test/lab should be ordered?

ERYTHROCYTE SEDIMENTATION RATE ~ Giant Cell Arteritis (ESR and C-reactive protein have very high sensitivity for giant cell (temporal) arteritis)

Alcoholic man dies arriving to ED, there are foci of hemorrhage and necrosis in the mammillary bodies...brain findings would show a decrease in what?

ERYTHROCYTE TRANSKETOLASE ACTIVITY ~ chronic th1m1ne (vitamin B1) deficiency impairs glucose utilization in the CNS...this is due to decreased activity of the enzymes that use thiamine as a cofactor (pyruvate dehydro, akg dehydro, transketolase)

7 year old girl with diffuse rash, 2 days earlier had red/flushed cheeks...today has stomach, back,extremities...siblings had similar symptoms where does the infectious agent replicate?

ERYTHROCYTES = fifth disease(erythema infectiousum) benign childhood illness caused by parvovirus b19. Parvovirus B19 replicates in erythrocyte precursors in the bone marrow

With most anemias increase in what blood parameter is most likely to be elevated?

ERYTHROPOIETIN PRODUCTION IN THE KIDNEY (in response to anemia-induced hypoxia)

Endometrial biopsy is abnormal and patient requires surgery. Mass:yelllow & firm, small cuboidal cells in sheets with gland-like structures containing acidophilic material with a pink/eosinophilic center. What is most likely secreted:

ESTROGEN ~ granulosa cell turmors are sex-cord stromal tumors of the ovary that secrete estrogen & can cause endometrial hyperplasia. CALL-EXNER BODIES (cells arranged in a microfolliculr or rosette pattern) are seen on microscopy. On gross path: turmor is yellow from lipid in THECA CELLS

Describe the toxin present in traveler's diarrhea>

ETEC that produces heat labile (LT-CHOLERA LIKE) and heat stable ST enterotoxins. LT (CHOLERA LIKE) activates AC leading to increased intracellular cAMP and ST activates guanylate cyclase leading to increased cGMP (both cause water and electrolyte loss)

6 year old kid inattentive, frequently stares into space lasts for several seconds and occurs many time throughout the day. What drug should be given?

ETHOSUXIMIDE (or sodium valproate) kid is having absence seizures (petit mal seisures)...momentary lapses in awareness.

Raised, painful, pruritic nodule that grows beyond wound borders

EXCESS COLLAGEN FORMATION = keloid

Kind brought to ER with poor feeding and irritability, has diffuse skin erythema and epidermis easily comes off what kind rxn/respone?

EXOTOXIN-MEDIATED SKIN DAMAGE Staph Scalded Skin Syndrome ...produces EXFOLIATIN epidermal sloughing with gentle pressure (nikolky's sign)

Woman taking meds for her diabetes and hypertension comes in talking about her recent weight loss due to a new supplement with caffiene and herbs, what should physician say?

EXPLAIN THAT WEIGHT LOSS SUPPLEMENTS ARE POORLY REGULATED AND HAVE POTENTIAL RISKS ~ physicians should routinely ask their patients about the use of herbal preparations and nutritional supplements and advise them on the quality, safety & efficacy of these products.

Man has a tonic-clonic seizure and MRI of the brain shows a 2.5cm cyst in sylvian fissure that has MINIMAL ENHANCEMENT and no associated edema...what is the most likely means of acquisiton?

EXPOSURE TO INFECTED STOOL ~ NEUROCYSTICERCOSIS ~ TAENIA SOLIUM ~ PORK TAPEWORM EGGS....the eggs are excreted in feces of human carriers.

Initial exposure to monoclonal antibodies agasint CD21 is most likely to prevent cell infection in which virus?

Ebstein-Barr virus. EBV is a herpes virus responsible for acute mononucleosis, nasopharyngeal carcinoma, and lymphomas (like burkitt lymphoma). Envelop glycoprotein of EBV binds to CD21 (aka CR2) the cellular receptor for C3d complement component...cd21 is normally present on B cell surface & nasopharyngeal epithelial cells...therefor anti CD-21 would interfere with EBV attachment to B cells.

What class of anti-fungals specifically work on the fungal cell wall only?

Echinocandins (caspofungin, micafungin) antifungal meds that inhibit synthesis of the polysaccharide glucan(essential for fungal cell wall)

The tissue found in a meckel's diverticulum is described as:

Ectopic tissue (ectopy) - Meckel Diverticulum small intestine ectopic tissue found in 2% of population [incomplete obliteration of OMPHALOMESENTERIC DUCT...they are most commonly gastric epithelium(80%) gastric acid production leads to bleeding/symptoms

What is effect modification?

Effect modification occurs when the effect of an exposure on an outcome is modified by ANOTHER variable.

Relationship between gastrin level and Zollinger-Ellison syndrome?

Elevated gastrin level (refractory peptic ulcer) likely related to a gastrin secreting tumor (zollinger-ellison syndrome) 20-30% associated with Multiple Endocrine Neoplasia type 1 (MEN1) 3 p's of MEN1 = parathyroid adenomas, pituitary tumors, pancreatic endocrine tumors.

Pulmonary function testing of someone with Emphysema

Emphysema = Restrictive Lung disease = a-1-antitrypsin = Decreased(FEV1/FVC), Increased(total lung capacity), Decreased(diffusing capacity)

How does Entacopone work?

Entacapone is a COMT inhibitor that increases the bioavailibility of levodopa by inhibiting its peripheral methylation...combining entacapone with levodopa helps increase the plasma half-life

Eosinophils play a role in host defense in parasitic infection (also regulate type 1 hypersensitivity, how do they do their action?

Eosinophils (when stimulated by antibodies bound to a parasitic organism) destory parasite via a antibody-dependent cell-mediated cytotoxicity from their cytoplasmic granules.

Woman has lumps under her armpit, they are biopsided and there is a clump of cells that have positive immunochemical staining for cytokeratin...the cells come from what lineage?

Epithelial...Cytokeratins form the keratin-containing intermediate filaments that make up the cytoskeleton of almost all epithelial cells. Makes sense they would stain because of CYTOKERATIN is a commonly used IMMUNOHISTOLOGICAL MARKER FOR EPITHELIAL-DERIVED TUMORS, SUCH AS BREAST CANCER (keratin is also the major protein component of hair and nails.

What are Betel Nut chewing & consumption of foods containing N-nitroso compounds an important risk factor for?

Esophageal Squamous cell carcinoma in asian countries

What is essential tremor and what is the first line for it?

Essential tremor is an autosomal dominant movement disorder. Patients report their symptoms improve with alcohol consumption. The FIRST LINE = PROPRANOLOL

Action of Etoposide?

Etoposide (chemotherapeutic agent that inhibits the sealing activity of topoII [irinotecan & topotecan = topoI)

How does theophyline toxicity present?

Excessive CNS stimulation (tremor, insomnia, seizures), Gi disturbances & cardiovascular abnormalities. Theophylline = adenosine receptor antagonist, primarily metabolized by hepatic cytochrome enzymes [drugs that inhibit cyt can cause toxicity]

Study participant is showing an increase in cardiac output and heart rate, but PaO2 & PaCO2 remain the same...what kind of study is going on?

Exercise study...PaO2 & PaCO2 are able to stay about the same because of V/Q matching...relatively constant arterial blood gas volumes [venous oxygen is decreased, venous CO2 is increased]

Common asthma triggers include:

Exercise, cold air, respiratory infection, exposure to inhaled allergens (dust mites, roaches, pet dander, mold, pollen)

What are the 3 ways a child can get Downs syndrome?

Extra chromosome 21 (trisomy resulting from nondisjunction); Unbalance roberstonian translocations or Mosaicism

What cholesterol-lowering agent reduces intestinal absorption of cholesterol?

Ezetimibe reduces intestinal absorption of cholesterol....amount of dietary cholesterol reaching the liver decreases. To compensate, the liver increases LDL receptor expression drawing cholesterol out of the circulation

20 year old progressive skin rash over past year, long-standing history of intermittent burning sensation in palms/soles exacerbated by stress & fatigue.Papules found in gluteal, inguinal & umbilical areas. Lab shows undetectable alpha-galactosidase A. What disease and major complication?

FABRY DISEASE (X-linked Recessive; alpha-galactosidase A deficiency) RENAL DISEASE/RENAL FAILURE, cardiac or cerebrovascular

Woman has a firm, nontender swelling of right cheek...MRI reveals neoplastic mass in parotid gland, if left untreated the patient is most likely to develop what?

FACIAL DROOP ~ CN7 exits stylomastoid foramen and courses through the parotid gland where it divides into its 5 terminal branches that innervate the muscles of facial expression. Malignant tumors can cause ipsilateral face droop

Patient can no long go outside because everyday noises are too loud, what nerve is messed up?

FACIAL NERVE (n. to stapedius)

Nurse keeps coming in with severe scratches and lesions some with fecal bacteria, what psych diagnosis?

FACTITIOUS DISORDER = intentional falsification or inducement of symptoms with goal to assume "sick role" get attention.

Woman with two previous births of babies with defects is pregnant and amniotic fluis shows elevated acetylcholinesterase...what is going on?

FAILURE OF FUSION OF THE EDGES OF THE NEURAL PLATE...if either neuropore does not fuse, an opening persists between the neural tube and the amniotic cavity that allows leakage of AFP and AChE = NTD

21 year old college athelete has recent weight loss and fatigue coach wants him to put on weight but he loses weight no matter how much he eats and has been really thirsty...what study would be best to estabilsih the diagnosis?

FASTING BLOOD GLUCOSE LEVEL ~ type 1 diabetes mellitus (insulin deficiency) Diagnosis can be made via fasting blood glucose or hemoglobin A1c

Crazy woman improves and is discharged on olanzapine at her 3 month check-up what studies/labs should be done?

FASTING GLUCOSE & LIPID PANEL (olanzapine is a second-generation antipsychotic they are associated with metabolic adverse effects increased risk of diabetes)

Man gets sick from fava beans, what disease? The substance generated during the conversion is necessary for what biochemical process?

FATTY ACID SYNTHESIS. G6PD deficiency is rate limiting step in formation of NADPH in pentose pathway...end goal is for NADPH to reduce gluthathione and biosynthesis of cholesterol, fatty acids, steroids.

What drug is given in hypertensive emergency (IV) to lower BP and increase natriuresis?

FENOLDOPAM ~ selective peripheral dopamine-1 receptor agonist. Lowers BP in HTN emergency especially in patients with renal insufficiency...causes renal perfusion and arteriolar dilation

Amniocentesis is performed on a 35 year old pregnant woman, what is the phospholipid content determined for?

FETAL LUNG MATURITY ...phospholipids (including surfactant = dipalmitoyl phosphatidylcholine). The amnionic fluid lecithin(phosphotidylcholine) to sphingomyelin ratio (L/S ratio) assess fetal lung maturity...mature = L/S ratio of 2 or more.

34 year old man severe chest pain and abdominal pain while shopping, cardiac arrest...histochemical autopsy reveals a defect effecting a large extracellular glycoprotein normally found in blood vessels, periosteum & zonular fibers?

FIBRILLIN-1 = MARFAN SYNDROME (fibrillin-1 acts as a scaffold for elastin) Aortic root dilation with dissection and rupture is a common cause.

AA patient with back and lower extremity pain, has had similar episodes in the past and his hemoglobin is 6.7, PE shows scleral icterus & bony tenderness. How would spleen look on histopath imaging?

FIBROSIS & ATROPHY patient likely has sickle cell disease (autosomal recessive) After autosplenectomy patients are predispoed to infections with encapsulated bacterial organisms.

Weakness in foot dorisflexion and eversion and loss of sensation over lateral shin and dorsal foot, where is nerve injury?

FIBULAR NECK ~ common peroneal nerve injury

Baby born incomplete division of the forebrain (prosencephalon) into two complete hemispheres...what is it called and what is it an example of?

FIELD DEFECT - holoprosencephaly...FIELD DEFECT

By 3 years old what should a kid be able to do?

FINE MOTOR: parallel play, speak simple sentances copy a circle, use utensils & ride a tricycle

Man undergoing cardiac stress test shows prolonging of QRS duration to a greater extent at a high heart rate....what drug is he likely taking?

FLECAINIDE ~ Potent sodium channel blockers that exhibit strong use-dependence by prolonging the QRS duration to a greater extent at higher heart rate.

IV drug user has aortic valve endocarditis, there is an abscess and small fistula between aortic root and RV, what will the direction of blood flow be?

FLOW FROM AORTIC ROOT TO RIGHT VENTRICLE CONTINUOUSLY....question about difference in cardiac pressures. AORTOCAVITARY FISTULAS can be seen in bacterial endocarditis....continuous flow from high pressured aorta to low pressured RV

Patient with UTI and urosepsis which is likely complicated by ARDS will end up hypoxic and in need of mechanical ventillation why?

FLUID ACCUMULATION IN THE ALVEOLAR SPACES ~ SEPSIS is one of the most common risk factors for ARDS.

Bullemic patient described in question stem, what drug would you prescribe?

FLUOXETINE (combined with nutritional rehab & psychotherapy) = 1ST LINE FOR BULLIMIA NERVOSA

What anemia is commonly associated with alcoholism?

FOLIC ACID ANEMIA

Patient have HIV collitis (first part of question is knowing what virus is associated with HIV collitis) what medicine was given (IV that does not require intracellular activation)?

FOSCARNET

Parents bring kid in for evaluation he's 16; neurobehavioral problems, prominent forehead, large ears, long/narrow face, large testes, prominent chin...what is diagnosis?

FRAGILE X (patients often have a developmental delay & neuropsych problems...findings can overlap with autism/adhd)

Question about biochem/genetics of cystic fibrosis the difference between basepair # on gel electrophoresis is 28, therefor what kind of mutation is CFTR gene in the CF kid?

FRAMESHIFT MUTATION ~not divisible by 3, deletion/addition of bases not divisible by 3 in the coding region will result in a frameshift mutation...altering the reading frame of the genetic code, resulting in nonfunctional proteins

rats exposed to carbon tetrachloride suffer rapid & extensive liver damage, LM exam of liver shows fatty change & hepatocyte necrosis. These changes are the result of:

FREE RADICAL INJURY ~ the p450 microsomal oxidase system plays an important role in detoxification. In carbon tetrachloride poisoning, however, it produces free radicals that start a vicious cycle of hepatic injury

Woman is sick needs transfusion her friend who is her health care proxy (durable power of attourney for health care) says do it, her husband says don'twhat do you do?

FRIEND = health care proxy = legally designated to make medical decisions in the event that patient loses capacity.

How is glucose normally brought into the cell?

Facilitated diffusion (carrier-mediated transport) glucose moves from high conc to low with the help of GLUT transmembrane receptor proteins [these GLUT proteins are STEREOSELECTIVE FOR D]

Cross section of right coronary artery showing soft/yellow/lipid core with a genetic bases, what is the likely disease and poor receptor function of what organ?

Familial Hypercholesterolemia(autosomal dominant) - Liver. Patient's RCA occlusion via large atheromatous plaque (soft, yellow, lipid core)...result of hetero or homozygous LDL receptor mutations, which cause hepatocyte under-expression of functional LDL receptors

Defects in ApoE3 and ApoE4 + xanthomas & premature atherosclerosis?

Familial dysbetalipoproteinemia (type 3 hyperlipoproteinemia) autosomal recessive characterized by elevated cholesterol & triglyceride levels

Relationship between retinoblastoma, 2 hits and osteosarcoma?

Familial retinoblastoma results from mutations of each of the 2 Rb genes (two hits)...patients have increased risk of secondary tumors...especially OSTEOSARCOMAS later in life.

The development of respiratory distress, diffuse neurological impairment(confusion) and an upper body petechial rash(thrombocytopenia) within days of severe long bone fractures is characteristic of what?

Fat embolism syndrome. The fat emboli occluding the pulmonary microvasculature stain black with osmium tetroxide

Difference in hypothalamic set point in fever and heat stroke (difference in management)?

Fever(hypothalamic set point is elevated; Aceminotphen/Ibuprofen OR SUPPORTIVE CARE). Heat stroke(hypothalamic set point is normal; RAPID EXTERNAL COOLING).

Statins are known to have a side effect of myopathy, this side effect is increased by adding what kind of drug?

Fibrates (gemfibrozil) + Statins = severe myopathy

Cellular/myxoid stroma that encricles and sometimes compresses epithelium-lined glandular and cystic spaces?

Fibroadenomas= most common benign tumor of the breast

Man with reccurent C Diff Collitis is placed on an ORAL MACROCYCLIC ANTIBIOTIC that inhibits the sigma subunit of RNA polymerase...

Fidaxomicin = macrolytic antibiotic that inhibits RNA polymerase. (metro and vanco could also be used)

Androgenic alopecia (male-pattern hair loss) causes loss at anterior scalp & vertex what is effective as tx?

Finasteride [DECREASE CONVERSION OF TESTOSTERONE TO DHT, SAVES HAIR] also effects the prostate (closer to butt on CT) BPH may cause static urinary obstruction. 5-ALPHA-REDUCTASE INHIBITORS (finasteride) inhibit the action of androgens on the prostate gland...preventing further enlargement ~ blockage.

Patient has bladder outlet obstruction (urinary hesistancy, forced strain) How is it treated? Effecting what?

Finasteride effects the prostate (closer to butt on CT) BPH may cause static urinary obstruction. 5-ALPHA-REDUCTASE INHIBITORS (finasteride) inhibit the action of androgens on the prostate gland...preventing further enlargement ~ blockage.

What effect do fish oil supplements have on VLDL

Fish oil supplements containing high concentrations of omega-3-fatty acids lower triglycerides by decreasing production of VLDL and apolipoprotein B.

Relationship between blood flow and radius, what formula? What happens when the flow is reduced by a factor of 16?

Flow(Q) = [P1-P2]/R [when the flow is reduced by a factor of 16 the lumen is decreased by 50%]

Patients with chronic hemolytic anemia are predisposed to what?

Folic acid deficiency and macrocytic changes due to increased erythrocyte turnover. Sickle cell patient develops macrocytic anemia (this is due to FOLIC ACID DEFICIENCY)

Nicotinic blockade (diplopia, dysphagia) following community picnic, where canned food was primary source. Whats up? What part of NM synapse is messed up?

Food poisoning via clostridium botulinum toxin (a preformed neurotoxin). DESCENDING PARALYSIS...3D's (diplopia, dysphagia, dysphonia). Clostridium Botox poisoning blocks PRESYNPATIC exocytosis of Ach vesicles.

Patient with CMV develops gancyclovir-resistant CMV, what drug that causes hypocalcemua and hypomagnesemia is administered? What analog is it?

Foscarnet (a pyrophosphate analog) can be used for ganciclovir-resistant CMV infections. Foscarnet can chelate calcium and lead to renal wasting of magnesium...both of those results can lead to seisures.

What contributes most to insulin resistance in type 2 diabetes?

Free fatty acid formation, type 2 diabetes is the result of insulin deficiency (chronically elevated FFA levels contribute to insulin resistance by impairing insulin-resistant glucose uptake and increasing hepatic gluconeogenesis)

What is the FASTEST reversal of warfarin OD?

Fresh frozen plasma (vitamin K and ffp are used to reversing warfarin-induced anticoagulation, FFP rapidly reverse's warfarin's effects whereas Vitamin K requires time for clotting factor re-synthesis.

Man comes to ED with weakness, malaise, dark urine. Recent bacterial skin infection. PE: icterus, anemia with elevated reticulocyte count & Abnormal erythrocytes on smear. Question asking between what two things/pathway is deficient?

G-6-P -> 6-PHOSPHOGLUCONATE patient has G6PD deficiency (X-linked recessive) G6PD is the rate-limiting enzyme of the PENTOSE PHOSPHATE PATHWAY....the deficiency results in episodes of hemolytic anemia during times of increased oxidative stress

Thickened gallbladder wall on xray (calcified) firm mass in right upper quadrant...what is she at risk for?

GALLBLADDER ADENOCARCINOMA ~ porcelain gallbladder....potential manifestation of chronic cholecystitis = increased risk of gallbladder adenocarcinoma

Woman with 3 days of upper abdominal pain, nausea and vomitting especially after fatty meals....got a red/distended gall bladded +patchy necrosis..what events led to this?

GALLBLADDER OUTFLOW OBSTRUCTION = acute calculous cholecystitis = gallstone obstruction of the CYSTIC DUCT ...which can lead to bacterial invasion.

Man with nocturnal cough, LM of esophagus endoscopy shows elongation of the lamina propria papillae with several eosinophils and neutrophils scattered within the squamous epithelium. Takes pantoprazole and 2 weeks later feels better, what was the cause of his symptoms?

GASTROESOPHAGEAL JUNCTION INCOMPETENCE ~ GERD

Woman has hyperlipidemia and elevated LDL, she has a hx of gallstones and refused surgery. What drug shoud be avoided?

GEMFIBROZIL and other fibrates inhibit the rate-limiting step of acid synthesis CHOLESTEROL 7-ALPHA-HYDROXYLASE = decreased cholesterol solubility and promoting gallstone formation.

What is fragile X caused by?

GENE METHYLATION ~ increased CGG trinucleotide repeats on the fragile x mental retardation 1 gene...leading to HYPERMETHYLATION & INACTIVATION OF FMR1

23 year old white student collapses and dies suddenly while taking an exam, no past medical history...what is the cause of the patient's condition?

GENETIC ~ hypertrophic cardiomyopathy (pictured) extreme myofiber disarray with interstitial fibrosis on cardiac histology strongly suggests hypertrophic cardiomyopathy (HCM). Almost 100% of cases of HCM result from mutations in genes encoding cardiac sarcomere proteins (BETA-MYOSIN HEAVY CHAIN)

Child with repeated infections (otitis media, pneumonia) Also has diarrhea + giardia, injected with Candida antigen and got a large nodule within 48 hours, patient's lymph nodes are most likely lacking what?

GERMINAL CENTERS (reccurent infections + low immunoglobulin levels = X-linked agammaglobulinemia...mutation in the Bruton tyrosine kinase gene so pre-b cells don't develop into mature B lymphocytes so GERMINAL CENTERS & primary lymphoid follicles do not form due to absence of B cells).

What is the usual cause of neonatal intraverntricular hemorrhage?

GERMINAL MATRIX ( can present with lowered consciousness and hypotonia) Ultrasound will show blood in lateral ventricles but it is due to GERMINAL MATRIX BLLED

In a patient with gigantism what is the relationship between GH & IGF-1?

GH is secreted by the anterior pituitary (increasing glucose & FFAs) The growth-producing effects are primarily mediated by insulin-like growth factor-1 (IGF-1 which is released by the LIVER) JAK-STAT pathway. IGF-1 binds to specific TYROSINE KINASE receptor on cells in boy and stimulates bone,muscle,tissue growth

55 year old male shows neuronal shrinkage & intense cytoplasmic eosinophilia...if patient survives eventually that area would show what?

GLIAL HYPERPLASIA the glial cells repair(astrocytes), phagocytose(microglia). Gliosis is proliferation of astrocytes in an area of neuron degeneration and it forms a glial scar which compensates for volume loss that occurs after neuronal death.

Mutations in what gene are a cause of maturity-onset diabetes of the young?

GLUCOKINASE = maturity-onset diabetes of the young. [glucokinase has a lower glucose affinity than other hexokinases]

Consistent binge drinking and not eating, patient presents with confusion...suppression of what is the cause of the patient's hypoglycemia?

GLUCONEOGENESIS

Renal metabolism of what amino acid is important for maximizing acid excretion in the patient? (currently peeing uric acid and has a low pH)

GLUTAMINE - acidosis stimulats renal ammoniagenesis, a process by which renal tubular epithelial cells metabolize glutamine to glutamate, generating ammonium that is excreted in chronic acidotic states.

54yo man with cirrhosis agitated and confused, PE:abdominal distension, decreased liver span and testicular atrophy, also has jerky irregular flexion-extention tremor...what will be elevated in the patient's ASTROCYTES

GLUTAMINE. Hepatic encephalopathy is caused by increased levels of ammonia and other neurotoxins in the circulation that lead to increased inhibitory neurotransmission & impaired excitatory neurotransmitter release.

Patient with type 2 diabetes, what treatment option would increase patient's serum C-peptide level?

GLYBURIDE (sulfonylureas & meglitinides) improve blood glucose levels in patients with type 2 diabetes mellitus by increasing secretion of insulin (and C-peptide) from pancreatic B-cells [C-peptide is a great marker for endogenous B-cell insulin secretion)

What oral diabetes medication increases insulin secretion by pancreatic beta cells independent of blood glucose concentration and has a high incidence of hypoglycemia in the elderly?

GLYBURIDE ...sulfonylureas bind to their receptor on pancreatic beta cells to inhibit the ATP-dependent potassium channel.

Which amino acid will be found in abundance investigating collagen (question calls it triple helical structures followed by fibrils)?

GLYCINE (Gly-X-Y)

Gross, painless hematuria in an older adult with rounded/polygonal cells with abundant clear cytoplasm?

GLYCOGEN AND LIPID ACCUMULATION (clear cell carcinoma...most common form of renal cell carcinoma)

Order of events of base excision repair:

GLYCOSYLASE(cleaves altered base), ENDONUCLEASE(cleaves 5' end), LYASE(cleaves 3' sugar), POLYMERASE(fills nucleotide gap), LIGASE(seals the nick)

Man with end-stage HCV infection gets liver from dead person, initially is fine and 1 week later gets nausea, vomitting, rach, pain, diarrhea..what is the cause?

GRAFT T CELL SENSITIZATION AGAINST HOST MHC ANTIGENS = GVHD = T lymphocytes found in donor organ become sensitized against MHC antigens of recipient and attack the host's tissues...[skin, liver, GI most common]

Major depressive disorder

GREATER THAN 2 WEEK PERIOD, 5 or more symptoms: depressed mood, loss of interest, sleep distrubance, loss of energy, apetite disturbance, psychomotor agitation/retardation, impaired concentration, guilt & suicidal thoughts.

There is a specific cofactor required to go from Oxaloacetate to phosphoenolpyruvate, the cofactor is derived from the TCA cycle what is it?

GTP is synthesized by SUCCINYL-CoA SYNTHETASE DURING THE CONVERSION OF SUCCINYL-CoA to succinate in the citric acid cycle. During gluconeogenesis, phosphoenolpyruvate carboxykinase uses GTP to synthesize phosphoenolpyruvate from oxaloacetate.

Woman has adenocarcinoma of the sigmoid colon, molecular testing of the cancer cells reveals a mutation in KRAS gene resulting in constituitive activation of Ras protein. Under normal circumstances the Ras protein is only active when bound to what substance?

GTP ~ regulation of Ras-MAPK signal transfuction pathway requires a balance ebtween GTP-bound(active) and GDP-bound(inactive) Ras proteins. RAS gene mutations are implicated in the development of malignant tumors.

Research: animals are immunized to protein fragmetns derived from peripheral nerve myelin...animals make T-cell response. Autoantibody against non-protein determinants is also present. Segmental demyelinization of peripheral nerve ensues...what is this analogous to?

GUILLAIN-BARRE SYNDROME ~ ganglioside-like substance in their lipopolysaccharide layer. Antidbodies formed against this substance can cross-react with ganglioside components of myelin. [CAMPYLOBACTER JEJUNI]

Classic galactosemia results from what deficiency?

Galactose-1-phosphate uridyl transferase..vomitting, lethargy, jaundice, escherichia coli sepsis

Chron's patients are prone to developing what kind of stones? What is the reason

Gallstones; increased bile acid wasting. [terminal ileum is typical location in chron's disease] decreased bile acid reabsorption = bile + cholesterole saturation = gallstones

Osteocytes have long intracanillicular processes that extend through the ossified bone matrix...the osteocytes remain connected to each other via:

Gap junctions (they must be able to exchange nutrients and waste products

Grayish-white vaginal discharge, with a fishy odor....what is unique about staining and what cells are seen?

Gardenerella Vaginalis = anaerobic GRAM-VARIABLE rod +potassium hydroxide "whiff test" +Clue cells (tx metronidazole or clindamycin)

What are gardos channel blockers used in?

Gardos channel blockers are also used in sickle cell patients to reduce polymerization of Hb S by inhibiting the efflux of potassium&water from the cells preventing dehydration.

Patient has posterior duodenal ulcer, what vessel is at highest risk of being messed up?

Gastroduodenal Artery (it lies along the posterior wall of the duodenal bulb and is likely to be eroded by posterior duodenal ulcers...can be life-threatening).

Lysosomal storage disease with "wrinkled tissue paper" disteneded macrophages

Gaucher Disease (distended macrophages = gaucher cells) - Autosomal Recessive lysosomal storage disease. [B-glucocerebrosidase deficiency]

What is gender and when do children fully understand the concept of gender?

Gender is the innate feeling someone has of being male or female. Children develop concept of gender by age 3-4 and permenancy of gender around 5-6. "son is playing with barbies...it is developmentally normal for them to explore the world by engaging in activities associated with the opposite gender).

College kid worried about grades, whether friends like him or not, finances, says worry is interfering with daily tasks...says symptoms have gotten worse over past seven months.

Generalized anxiety disorder = excessive, chronic worry over multiple issues lasting greater than 6 months. (TX: antidepressants & CBT)

What is the variation in peak bone mass determined by?

Genetic (Black women higher bone mass & lower risk of fracture) & Environment (nutrition & physical activity)

Patient's painful recurrent vesicular lesions on penis shaft and Tzank Smear are consistent with what? What treatment could have possibly prevented reccurance?

Genital herpes simplex virus (reactivation due to latent HSV-2 infection hiding in sacral ganglia S2-4 DRG). Can be suppressed or minimized via daily oral valcyclovir 1x daily dose and good oral bioavailibility

What is genomic imprinting and what is it caused by?

Genomic imprinting refers to phenomenon where an offspring's genes are expressed in a parent-specific manner. Genomic imprinting is caused by DNA methylation [epigenetic process where genes are silenced via methylating CYTOSINE groups]

4 year old, 3 days of fever irritability & oral lesions, parents are concerned due to refusal to eat/drink painful oral ulcer base scraping shows multinucleated giant cells. What is it?

Gingivostomatitis (PRIMARY INFECTION) Multinucleated giant cells is suggestive of HSV infection...painful vesicles appear

2+ weeks after ischemic stroke what I seen on histology?

Glial Scarring

3rd pharyngeal arch

Glossopharyneal nerve + 3rd aortic arch + COMMON CAROTID ARTERY

What accounts for the difference in response of the body to oral glucose vs IV glucose?

Glucagon-like peptide 1 = Increatin (GLP-1 & GIP) are incretins and they stimulate insulin release following oral consumption of glucose...incretin-stimulated insulin release is independent of the increase in insulin secretion brought on by elevations in the blood gluose level

Man with progressive visual impairment, poorly controlled diabetes...bilateral clouding of lens, what metabolic conversion is contributing to the current condition?

Glucose to Sorbitol [polyol pathway] In hyperglycemic states, ALDOSE REDUCTASE converts glucose to sorbitol at a rate faster than sorbitol can be metabolized so it accumulated in certain cells such a lens cells...causing an influx of water and resulting in osmotic cell injury.

How can degradation of triglycerides in adipose tissue be used to synthesize glucose?

Glycerol produced by the degradation of TGs can be used by GLYCEROL KINASE in the liver and kidney to synthesize glucose during gluconeogenesis. STEM: DKA patient, which enzyme provides the substrates necessary for gluconeogenesis

Weightlifter going for heaviest lift of his life holds weight above head before his arms involuntarily give way...what structure is responsible for sudden muscle relaxation?

Golgi Tendon Organ (feedback system monitors and maintains muscle force)

Recurrent acute monoarthritis & history of myeloproliferative disorder?

Gout - Negatively Birefringent/Needle-Shaped. Monosodium urate crystals in joints & soft tissues

50 year old woman 6 month history of progressive fatigue and generalized pruritis (high serum alkaline phosphatase), sister has sjogrens Physical exam shows hepatomegaly; histo findings will resemble what disease?

Graft vs. host disease (similar on histology) Patient has primary biliary cholangitis: lymphotic inflammation & destruction of intrahepatic bile ducts.

Woman presents with malaise and she smokes occasionally, but since she has been sick has developed an aversion to smoking...lab results: Anti-HAV IgM+, Anti-HBs+...what's up?

HAD STEAMED OYSTERS AT A NEIGHBORHOOD RESTAURANT...Hep A is fecal-oral (sanitations and crowding) Steamed shellfish here in US.

Researcher doing a study but once physicians find out they are being watched, what happens?

HAWTHORN EFFECT MOTHA ****A = observer effect = tendency for a subject to change their behavior when they know they are being watched.

What Hep B marker is most likely to result in the passage vertically of a virus?

HBeAg = marker for viral replication & increased infectivity...HBeAg in the mother greatly increases the risk of vertical transmission of the virus.

2 month old infant perinatal hepatitis B exposure, mom has HBsAg and HBeAg, infant has not been evaluated but on physical exam is unremarkable.What will infant's labs show?

HBeAg ~ Maternal viral load & HBeAg status are the strongest risk factors for infant infection. [infants born to mom's with positive HBeAg have 90% chance)

What do small nuclear ribonucleic acids do?

HELP REMOVAL OF INTRONS FROM RNA TRANSCRIPTS ~ question was in reference to a patient with SLE who has autoantibodies directed to snRNPs.snRNA is synthesized by RNA Pol II and complexes with specific proteins to form small nuclear ribonucleoproteins snRNPs. snRNPs are important part of spliceosomes which REMOVE INTRONS FROM PRE-mRNA TO FORM MATURE mRNA.

Man got stabbed in the neck, vitals normal but he has right-sided hemiparesis and there is loss of proprioception and vibration sensation below C8 on right side and loss of pain/temp on left side MRI of spine will reveal what?

HEMISECTION OF THE RIGHT SIDE OF THE CORD = brown-sequard syndrome [ipsilateral spastic paralysis, loss of tactile/vibration/position & contralateral loss of pain/temp]

Schistocytes on blood smear suggestion what? What kind of toxins can produce that in a kid?

HEMOLYTIC UREMIC SYNDROME ~ E COLI O157:H1 ~undercooked ground beef. Shiga-like toxin. MICROANGIOPATHIC HEMOLYTIC ANEMIA

Bleeding after a tooth extraction and a history of hemarthrosis are suggestive of what? Adding what can help this?

HEMOPHILIA 8=A; 9=B decreased levels of 8 or 9 lead to failure to convert prothrombin to thrombin and then you can make thrombi.

Picture of messed up liver of 65 yo man who died from progressive liver disease, changed and unchanged hepatocytes demonstrate foreign DNA fragments in the genome. These fragments most likely belong to which organism?

HEP B VIRUS (DNA virus) integrates its DNA into the host genome, thereby increasing the risk for hepatocellular carcinoma. In contrast, Hepatitis C virus (RNA virus), lacks reverse transcriptase and does not integrate into the host genome,

What is delta agent infection?

HEP D (WOULD DISAPPEAR IF EVERYONE WAS VACCINATED AGAINST HEP B)

The main mechanism of excess copper removal in a healthy human body is what?

HEPATIC EXCRETION INTO BILE (absorbed copped is used to form ceruloplasmin, which accounts for 90-95% of circulating copper. Senescent ceruloplasmin and the unabsorbed copper are secreted into bile and excreted in stool (primary route for copper elimination)

Which hepatitis is associated with spotty necrosis on histology?

HEPATITIS A = FEVER, ANOREXIA, DARK-COLORED URINE...self-limited, prodromal symptoms and signs of cholestasis

Patient with fever & joint pain has atypical lymphocytes with an RNA-dependent DNA-polymerase. Virus is partially double-stranded circular DNA what virus?

HEPATITIS B = PARTIALLY DOUBLE-STANDED CIRCULAR DNA & reverse transcriptase. +Capsid

Sexually active man with history of IV dug abuse is feeling fatigue and losing weight, LM reveals large hepatocytes filled with finely granular, homogenous, pale pink cytoplasm. What is the diagnosis?

HEPATITIS B ~ FINELY GRANULAR, DIFFUSELY HOMOGENOUS,PALE EOSINOPHILIC CYTOPLASM = GROUND-GLASS = PARTIALLY DOUBLE-STANDED CIRCULAR DNA & reverse transcriptase. +Capsid

man with bad pyelonephritis needs to be started on a drug with mainly non-renal clearance and needs to have hepatic metabolism, what characteristic is needed?

HIGH LIPOPHILICITY ~ high Vd, good penetration into CNS are preferentially processed by the liver into more polar compounds for easier elimination in the bile & urine.

Man with no previous diabetes diagnosis is showing signs of insulin resistance, what finding would be most suggestive of insulin resistance?

HIGH WAIST CIRCUMFERENCE ~ insulin resistance ~ type 2 diabetes

What area of the brain demonstrates atrophy in alzheimer's

HIPPOCAMPUS ~ hippocampal atrophy on MRI is highly suggestive of diagnosis

Child possibly stung or poked, PE: edematous and erythematour plaque with mild central pallor...what substance is most likely repsonsible for skin findings observed in the patient?

HISTAMINE - allergic rxn Type1 HSR (wheal and flare rxn)...promotes antibody class switching to IgE

Biopsy shows: Endomysial mononuclear infiltrate; patchy necrosis. PE: symmetrical proximal muscle weakness. Increasing difficulty climbing stairs, getting up from a chair and carrying heavy objects. What autoantibody?

HISTIDYL-tRNA SYNTHETASE ~ POLYMYOSITIS. Polymyositis is characterized by symmetric proximal muscle weakness. It is associated with antinuclear & anti-Jo-1 autoantibodies. ENDOMYSIAL MONONUCLEAR INFLAMMATORY INFILTRATE & PATCHY MUSCLE FIBER NECROSIS.

Explain the pathology of Huntington's Disease (CAG)

HISTONE DEACETYLATION. HD is an autosomal dominant neurodegenerative disease that results in a GAIN OF FUNCTION. HYPO-ACETYLATED HISTONES BIND TIGHTLY TO DNA

Image of small ovoid bodies within a macrophage...paitent has HIV and cough/low fever/hepatosplenomegaly

HISTOPLASMA CAPSULATUM

What is the most important thing for dianosis of suspected tetanus and why?

HISTORY & PHYSICAL EXAM ~ diagnosis is CLINICAL...has to be penetrating wound in someone who has not been vaccinated (sardonic smile, muscle spasms, trismus)

Inflammatory acne what is pathogensis & what type of secretion mechanism?

HOLOCRINE - cell lysis releases ENTIRE contents of cytoplasm & cell membrane [THIS IS ACNE]. [sebaceous glands & meibomian glands]

7 year oid has increased blood oxygen saturation between 2 right sided chambers (artrium & ventricle) what PE finding will be heard?

HOLOSYSTOLIC MURMUR OVER THE LEFT STERNAL BORDER ~ VSD is the most common congenital heart disease. Small VSD produces a HOLOSYSTOLIC MURMUR THAT IS LOUDEST over the LEFT MID-STERNAL BORDER

Koilocyte is a sign of what?

HPV (1-4 skin warts-verruca vulgaris; 6,11 genital warts-condylomata acuminata; 16,18-cervical, vaginal, vulvular & anal neoplasia)

HIV positive patient is non-compliant and later returns with a hard mass with superficial ulceration in anal canal, what is responsible?

HPV (types 16 & 18) are strongly associated with anal and cervical squamous cell carcinoma [HIV infection increases risk of anal carcinoma]

What types of HPV cause warts? What location in the respiratory tract can the warts pop up?

HPV 6,11 condyloma acuminatum dsDNA virus that loves STRATIFIED SQUAMOUS EPITHELIUM will infect TRUE VOCAL CORDS

What kind of virus causes gingival stomatitis in children (describe virus)

HSV-1 = DNA VIRUS, DOUBLE-STRANDED, ENVELOPED

What is hydrocephalus ex-vacuo?

HYDROCEPHALUS IS AN ENLARGEMENT OF THE VENTRICLES (WITH OR WITHOUT ELEVATED PRESSURE) CAUSED BY NEURONAL VOLUME LOSS. AID dementia can be associated with HYDROCEPHALUS ex-vacuo due to significant cortical atrophy, which allows the ventricles to expand while maintaining normal pressure

What kidney medication can be prescribed after someone passes a kidney stone (calcium oxalate) to prevent future stones?

HYDROCHLOROTHIAZIDE ~ thiazide diuretics effectively increase calcium reabsorption from the nephron...they are indicated in patients with nephrolithiasis secondary to hypercalciuria and hypercalcemia.

60 year old man, new onset patchy loss of vision...flame-shaped retinal hemorrhage...what is the cause?

HYPERTENSION (severe hypertension in retinal precapillary arterioles causes endothelial disruption, leakage of plasma into arteriolar wall and fibrinous necrosis.

CT OF DISSECTION (TEAR IN TUNICA INTIMA) what is the most important risk factor?

HYPERTENSION = single most important risk factor for the development of INTIMAL TEARS leading to aortic dissection. Hypertension, smoking, diabetes mellitus & hypercholesterolemia are all major risk factors for atherosclerosis, which predisposes more to aortic aneurysm formation than aortic dissection.

Explain high altitude sickness

HYPOBARIC HYPOXIA IS THE PRIMARY CAUSE. High altitude: headache, fatigue, nausea, dizziness, & sleep disturbance, hyperventilation. HYPOBARIC HYPOXIA is primary cause, it leads to increased resp rate w/development of resp alkalosis which is compensated by renal excretion of bicarb within 24-48 hours.

Man has bad sleep and snoring PE: bulky tongue and crowded, narrow oropharynx...electrical stim of what nerve may improve pathophysiologic cause of the patient's symptoms?

HYPOGLOSSAL NERVE ~ obstructive sleep apnea (reccurent episodes of upper airway collapse during sleep, neuromuscular weakness...stimulation of HYPOGLOSSAL NERVE using implantable nerve stimulator causes tongue to move forward slightly)

16 year old beauty pageant girl has strict diet and exercise plan, her last menstrual period was 8 months ago, what is the likely mechanism of her amenorrhea?

HYPOTHALAMIC SUPPRESSION (BOSS) = functional hypothalamic amenorrhea results from loss of pulsatile gonadotropin-releasing hormone release from the hypothalamus and is caused by weight loss, strenuous exercise, systemic illness or abnormal eating habits. Loss of cyclic gonadotropin release leads to a decrease in LH & FSh secretion from the pituitary -> low estrogen

homeless man unresponsive, arterial blood is 60 mmHg and partial pressure of oxygen in alveolie is 71 mmHgm what is the most likely cause?

HYPOVENTILATION ~ the partialpressure of oxygen in the alveoli (PaO2) is normally 104mmHg...blood pO2 generally reaches the PaO2 by the time it passes through the first third of the alveolar capillaries due to a high rate of O2 diffusion through the respiratory membrane. But once oxygenated blood enters the systemic circulation, its pO2 drops to about 100mm Hg due to the addition of deoxygenated blood from bronchial circulation

Patient with cardiogenic shock & evidence of watershed necrosis on autopsy suffered from what?

HYPOXIC-ISCHEMIC ENCEPHALOPATHY ~ watershed infarcts occur between the zones of perfusion of the anterior,middle & posterior cerebral arteries. These infarcts typically appear as bilateral wedge-shaped strips of necrosis over the cerebral convexity, parallel & adjacent to the longitudinal cerebral fissure.

What bug is the most common cause of epiglottis(fever, stridor, dyspnea)? What is its major virulence factor?

Haemophilus Influenzae is the most common cause of EPIGLOTTITIS(fever, stridor, dyspnea), PRP ( Polyribosylribitol phosphate) is a capsule component and major virulence factor for H influenze type B

What kind of hemoglobin is predominant form at birth? How does this hide disease?

HbF contains Gamma-globulin instead of Beta-globulin...patients with homozygous B-thalassemia (major) are often asymptomatic at birth due to presence of gamma globulins and HbF [once body switches to HbA symptoms show]

Bloom syndome = BLM gene what protein is encoded (and therefor messed up during bloom syndrome)

Helicase (helicase is positioned right where DNA is unwinding...it unwinds DNA during DNA replication) Bloom = growth retardation, facial anomalies, photosensitive skin rash, immunodeficiency due to chromosomal instability & breakage).

What is a common result of a lacunar stroke?

Hemibalism = wild, involuntary flinging movements of the proximal limbs

Why is having valine at the 6th position of the beta-globin chain worse than having glutamic acid?

Hemoglobin S (HbS) contains valine in place of glutamic acid at the sixth amino acid position of the B-globulin chain. THIS PROMOTES HYDROPHOBIC INTERACTION among Hb molecules and results in HbS polymerization and erythrocyte sickling.

relationship between bactrim and G6PD deficiency?

Hemolysis induced by TMP/SMX(bactrim) is a very common way for g6pd deficiency to be diagnosed.

New born girl positive coombs test + jaundice, hepatomegaly, generalized edema ...many nuclear erythrocytes whats up?

Hemolytic disease of the new born Rh+ dad Rh- mom and Rh+ baby on second birth will be attacked by Anti-Rh+ antibodies causing lysis of fetal RBCs. ERYTHROCYTE OPSONIZATION BY MATERNAL ANTIBODIES

Palpable lower extremity purpura, abdominal pain, arthralgias & hematuria. Disease? Mechanism?

Henoch-Schonein Purpura (IgA immune-complex mediated type3 HSR; vasculitis) CIRCULATING IMMUNE COMPLEXES

Tumor cells express CD31 (endothelial marker), person has exposure to arsenic, thorotrast & polyvinyl chlrodie...what dx?

Hepatic angiosarcoma is associated with exposure to carcinogens such as arsenic, thorotrast & polyvinyl chloride. Tumor cells express CD31 (an endothelial marker)

What is hepatic encephalopathy and what is it precipitated by?

Hepatic encephalopathy is caused by increased levels of ammonia and other neurotoxins in the circulation that lead to increased inhibitory neurotransmission & impaired excitatory neurotransmitter release.

What disease is associated with deficiency Galactose-1-Phosphate uridyltransferase? How does it present?

Hereditary galactosemia, vomitting & feeding intolerance, jaundice, hemapatomegaly but symptoms start as soon as breast feeding begins!

Physical & mental retardation, megaloblastic anemia, elevated urinary orotic acid levels...what disease and deficiency? What can be given?

Hereditary orotic aciduria, defect in uridine 5'-monophosphate synthase (UMP). Uridine supplementation can improve symptoms as uridine is converted to UMP via nucleoside kinases.

What is responsible for the variability in clinical presentation between a male patient and his female sister when they both have a mitochondrial inherited disease?

Heteroplasmy = distribution of normal/mutated mitochondria during mitosis ...some are healthy some are mutated (HETEROPLASMY)

Is it potassium influx only or high potassium and some sodium conductance that defines the resting membrane potential?

High potassium & SOME sodium, since resting membrane potential is a DIFFERENCE.

What histone protein is located outside the core and helps promote chromotin compaction?

Histone H1

What is homocytienuria and what is it commonly caused by?

Homocystineuria condition leads to hypercoagulability & thromboembolic occlusion. Homocystinuria is most commonly caused by a defect in cystathione synthase (cannot for cystiene from homocysteine). Cysteine is essential in these patients and homocysteine accumulation results in prothrombotic resulting in premature thrombotic events.

Relationship between starvation & hormone sensitive lipase (old woman falls in bathroom urine has high ketones, serum has glucose)

Hormone sensitive lipase (found in adipose tissue) breaks down stored triglycerides into free fatty acids and glycerol...during times of starvation horome-sensitive lipase provides substrates for hepatic gluconeogenesis & ketone body formation

9-month old brought to the ED after 2-minute seizure, patient had fever then fever went away and 3 days later returned. What bug?

Human Herpesvirus ^ (rose6ola)

What blood pressure medicine is contraindicated in patients taking high amounts of lithium?

Hydrochlorothiazide. Chronic Lithium toxicity (confusion, ataxia, neuromuscular excitability) can be precipitated by volume depletion and drug interactions with thiazide diuretics, ace inhibitors & NSAIDs)

New Alzheimer's medicine - changes beta pleated sheets to alpha helices...the conformational change is the result of reorganization of what?

Hydrogen Bonds (HYDROGEN BONDS ARE THE PRINCIPAL STABILIZING FORCE FOR THE SECONDARY STRUCTURE OF PROTEINS) secondary structure). The primary structure: AA linked to each other via peptide bonds. Secondary structure: B-sheets use hydrogen bonds between all residues of anti-parallel strands. Alpha helices hydrogen bond between every FOURTH amino acid.

What disease is hydroxyurea used for and how does it work?

Hydroxyurea is used in sickle cell anemia patients by INCREASING HEMOGLOBIN F SYNTHESIS. Hydroxyurea is reserved for patients with frequent pain crises

Patient has reccurent sinopulmonary & gastrointestinal infections, failure to thrive, defective signaling between CD4 T cells & B cells...what syndrome?

Hyper IgM syndrome. Results from defective immunoglobin class switching due to a defect in CD40 ligand and CD40 interaction.

Three types (times) of organ rejection? Method?

Hyperacute(on table), Acute(less than 6 months, Chronic(months to years). HOST T CELL SENSITIZATION AGAINST GRAFT MHC ANTIGENS(mhc hla antigens)...causes mononuclear infiltrate on histology and graft dysfunction.

What kind of volume contraction is diabetes insipidus?

Hyperosmotic volume contraction...diabetes insipidus, dehydration, and profuse sweating = loss of free water. (less fluid, increased osmolarity)

Lesions in the Ventromedial nucleus of the hypothalamus can cause what?

Hyperphagia (excessive hunger) & obesity

Weight loss (despite normal apetite), tachycardia, warm/moist skin, tremor, frightened stare & restlessness are signs of what? If there is an anxiety component now what is it?

Hyperthyroidism (weight loss, tachy, warm/moist skin, tremor). Anxiety due to a general medical condition is diagnosed when symptoms are the physiological consequences of an underlying medical condition (like hyperthyroidism)

Patient needs radioactive iodine therapy...what substance would decrease the effect of radiotherapy through competitive inhibition of iodine transportation?

Hyperthyroidism due to graves disease is treated with radioactive iodine therapy...X-PERCHLORATE...perchlorate & pertechnetate ions (as well as iodide isotopes) are absorbed into the thyroid gland via sodium-iodide symporter...high levels of either will reduce iodine uptake via COMPETITIVE INHIBITION.

Pretibial myxedema and exopthalmos are pretty specific features of what?

Hyperthyroidism due to graves disease is treated with radioactive iodine therapy...X-PERCHLORATE...perchlorate & pertechnetate ions (as well as iodide isotopes) are absorbed into the thyroid gland via sodium-iodide symporter...high levels of either will reduce. ACCUMULATION OF GAGs WITHIN TISSUES

Narcolepsy (recurrent lapses into sleep multiple times a day occuring at least 3 times in 3 months, can be triggered by laughing or crying) What will be LOW in CSF?

Hypocretin-1 (orexin-A) or hypocretin-2(orexin-b)...they are produced in the LATERAL hypothalamus and function to promote wakefullness and inhibit REM sleep-related phenomena

In males, incomplete fusion of the urethral (urogenital) folds results in what?

Hypospadia = an abnormal opening of the urethra proximal to the glans penis along the ventral shaft of the penis.

A collection of "tan" colored fat found inside baby during surgery, what is the likely result of removing this tissue?

Hypothermia...brown adipose tissue comrpises 5% of body mass in neonates and helps keep them warm. Brown adipose contains several intracytoplasmic fat droplets and many more mitochondria compared to white adipose. They produce heat via UNCOUPLING OXIDATIVE PHOSPHORYLATION W/THERMOGENIN

Man has improved during hospital stay and gives you a card with 4 basketball tickets in there because they were given to him and can't go, appropriate response?

I APPRECIATE YOUR THOUGHTFULNESS AND WILL TREASURE THE CARD YOU HAVE MADE FOR ME. HOWEVER, IT WOULD NOT BE RIGHT FOR ME TO ACCEPT THE TICKETS (unethical and problematic to accepted gifts over $5-$10 bucks....still say thank you tho)

Patient is not compliant with medication, appropriate response?

I CAN UNDERSTAND HOW IT MIGHT BE DIFFICULT TO TAKE MEDICATION DAILY WHEN YOU'R FEELING GOOD

Best method for confronting angry patients?

I SEE YOU ARE UPSET; I THINK WE SHOULD DISCUSS YOUR CONCERNS AND QUESTIONS [non-defensive & acknowledging patient's anger]

Old man doesn't want to know disease status (son says he doesn't want to know, then patient confirms) but its cancer, what do you tell patient's son?

I WILL NOT INFORM HIM OF THE RESULTS IF THAT IS HIS PREFERENCE ....physicians must understand and respect the beliefs of cultures that value beneficienceand nonmaleficence over autonomy

Woman having baby by a different man than her husband and she asks you not to tell him (he Is also your patient) what is the correct response?

I WILL NOT TELL YOUR HUSBAND ~ patient confidentiality is strongly protected because patients must feel free to disclose details of all aspects of their lives so that physicians can provide optimal care

Best friend of physician...friend has been getting antidepressants since college but claims she cannot get in to see her doc, can you give her some antidepressants?

I WOULD LIKE TO HELP, BUT I AM UNCOMFORTABLE PRESCRIBING FOR SOMEONE I AM NOT TREATING (btw treating family & friends is ONLY acceptable and ethical in an emergency)

Child, bone marrow biopsy reveals hypocellularity and an abundance of fat cells...

IDIOPATHIC APLASTIC ANEMIA ~ pancytopenia with bone marrow hypocellularity

What cytokine attenuates the immune response through inhibition of Th1 cytokines?

IL-10 plays an important ANTI-INFLAMMATORY and immunomodulatory role in the pathogenesis of inflammatory bowel disease (like Chron's). IL-10 attenuates the immune response through inhibition of Th1 cytokines, redcution of MHC-Class II expression and suppression of activated macrophages and dendritic cells. IL-10 is the reason for reduced abdominal symptoms in patient between doctor visits.

What is IL-2 produced by and what does it stimulate the growth of? What impact can this have on cancer therapy?

IL-2 is produced by helper T cells and stimulates the growth of CD4+ & CD8+ T cells & B cells. IL-2 also activates NK cells & monocytes. Increased T cell & NK cell activity is thought to be responsible for IL-2's anti-cancer effect on metastatic melanoma & renal cell carcinoma.

Sensitized Th2 cells exposed to inhaled allergens can produce a substance that promotes B-lymphocyte class-switching to IgE....what IL?

IL-4. Sensitizied Th2 cells secrete IL-4 & IL-13 which together promote B-lymphocyte class switching for IgE synthesis.

Man has paroxysmla episodes of breathlessness and wheezing, sputum microscopy shows many granule-containing cells and crystalloid masses...what interleukin?

IL-5, granule-containing cells are most likely charcot-leyden crystals....Chronic EOSINOPHILIC bronchitis in asthmatics, response to IL-5 released by allergen-activated Th2 cells

Wound starts to release pus, what molecule is involved in mediating the accumulation of pus?

IL-8, a chemokine produced by macrophages that induces chemotaxis and phagocytosis in neutrophils

Patient presents with symptoms/signs of small bowel obstruction & an abdominal x-ray reveals gas within gallbladder and biliary tree...where is gallstone stuck?

ILEUM. Gallstone ileus results from passage of a large gallstone through a cholecystenteric fistula into the small bowel where it ultimately causes obstruction at the ILEUM

Kid has appendectomy, but days later has burning pain at surgical scar radiating to the suprapubic region....there is loss of sensation over the right suprapubic area. What nerve is most likely injured?

ILIOHYPOGASTRIC provides sensation to the suprapubic and gluteal regions and motor function to the anterolateral abdominal wall musces.

Fetal thymus shows a certain population of cells identified as CD4+ & CD8+ these cells are best characterized as what?

IMMATURE CORTICAL T LYMPHOCYTES ...immatue t-lymphocytes express both CD4 & CD8 cell surface antigens in addition to a complete TCR or a pro-TCR. These lymphocytes exist in the thymic cortex where they undergopositive slection and in the thymic medulla where they undergo negative selection

Femoral vein access is needed, based on pulse felt from femoral artery...what is the anatomical location of vein compared to artery?

IMMEDIATELY MEDIAL TO THE FEMORAL ARTERY (navel...lateral to medial)

Difference between precursor B-ALL and T-ALL?

IMMUNOPHENOTYPING NECESSARY. B-ALL:Tdt+. CD10+, CD19+ [t=cd2,3,4,5,7,8,Tdt,CD1a]

How does FLUTAMIDE work?

IMPAIRED ANDROGEN-RECEPTOR INTERACTION...non-steroid anti-androgen that acts as COMPETITIVE inhibitor of testosterone receptors...it is used in combination with long-acting gonadotropin-releasing hormone agonists to tx prostate cancer.

3 week old boy brought to ED for lethargy, was delivered at home "naturally" with no vaccinations. On exam he has a bulging anterior fontanel and eyes are driven downward, CT confirms intracranial hemorrhage. What caused the infant's condition?

IMPAIRED CLOTTING FACTOR CARBOXYLATION ~ NEONATAL VITAMIN K DEFICIENCY (intracranial, GI, cutaneous, umbilical, surgical site bleeding). Vitamin K deficiency results in impaired clotting factor carboxylation. Newborns are at risk for Vit K deficiency due to POOR TRANSPLACENTAL TRANSFER of Vitamin K and low content in breast milk.

Woman had poor prenatal care and was taking medication for poorly controlled HTN, what is the likely cause of neonate's abnormalities?

IMPAIRED METABOLISM OF ANGIOTENSIN...fetal urine is a major source of amnionic fluid...low Ang II = oligohydramnios, pulmonary hypoplasia, limb contractures and calvarium defects.

Hydrocephalus, intracranial calcifications & chorioretinitis, infection and stage of pregnancy?

IN-UTERO INFECTION ~ Congenital Toxoplasmosis is a transplacental infection, expecting mothers should avoid cat feces,

Patient with recurrent bacterial meningitis, +petehcial rash on trunk and extremities +fever,headache....what immune system impairment is responsible?

INABILITY TO FORM THE MAC patients unable to form the membrane attack complex (C5-C9) often experience recurrent infections by the Neisseria species.

Woman with E Coli strain that does NOT produce glucuronidase and does not ferment sorbitol...what strain and how does the toxin work?

INACTIVATES RIBOSOMAL SUBUNITS ~ EHEC O157:H7 [can lead to HUS]

Baby with downs has a large, reducible midline abdominal protrusion more pronounced when he cries, what is the most likely cause?

INCOMPLETE CLOSURE OF THE UMBILICAL RING = Uncomplicated umbilical hernia (reducible, asymptomatic, resolve spontaneously)

40 year old womn sudden onset right arm weakness and aphasia, takes oral contraceptives what is going on?

INCOMPLETE FUSION OF THE ATRIAL SEPTUM PRIMUM AND SECUNDUM. Taking oral contraceptive puts her at risk of hypercoagulabilty

Increase in GFR with decrease in RPF leads to what FF?

INCREASE FF [FF = GFR/RPF]

2 day old boy brought to intensive care unit from nursery with tachypnea & hypoxia +cyanosis. Died. Autopsy showed decreased numbers of granules containing parallel stacks of membrane lamellae...what pathological process?

INCREASED ALVEOLAR TENDENCY TO COLLAPSE...surfactant (produced by type II pneumocytes works to decrease the surface tension in alveoli = facilitating lung expansion during respiration)

11 year old girl above-average intelligence but bad grades, disruptive in school and at home...frequently loses stuff, parents want medication that will help. What is the MOA of the most appropriate drug?

INCREASED AVAILABILITY OF NOREPINEPHRINE & DOPAMINE ~ child has ADHD stimulant drugs are first-line they work by INCREASING AVAILABILITY OF NOREPINEPHRINE AND DOPAMINE IN THE PREFRONTAL CORTEX.

Man is a smoker and has chronic cough oxygen stat decreases with moderate exertion, what other nonheart/lung response will his body likely have?

INCREASED ERYTHROPOIETIN PRODUCTION ~ COPD CAN CAUSE HYPOXIA BAD ENOUGH TO STIMULATE ERYTHROPOIETIN PRODUCTION by cortical cells of the kidney

17 year old boy comes in for eval regarding bilateral breast enlargment, slightly painful. Has history of learning disabilities testes are small and firm, super tall and super skinny...what would labs likely show?

INCREASED FSH ~ Klinefelter syndrome 47,XXY male nondisjunction of the sex chromosomes.

Image shows uric acid stones on regular micrscopy and wants to know what is the underlying mechanism leading to the stone formation?

INCREASED HYDROGEN ION EXCRETION IN THE KIDNEY

What kind of increased pressure in a patient with suspected mitral stenosis suggests that there is also aortic valve involvement?

INCREASED LEFT VENTRICULAR DIASTOLIC PRESSURE. (isolated mitral stenosis usually doesn't involve diastolic pressure...but if there is aortic involvement it will)

Third heart sound (S3) low-frequency sound occuring just after S2, what is it associated with?

INCREASED LEFT VENTRICULAR END-SYSTOLIC VOLUME = LEFT VENTRICULAR SYSTOLIC FAILURE

In spherocytosis explain the profile of the RBC

INCREASED MEAN CORPUSCULAR HEMOGLOBIN CONCENTRATION, elevated lactate dehydrogenase, reticulocytosis & decreased haptoglobin

Medical pilot has glaucoma on fundoscope exam and is prescribed latanoprost eyedrops...how do they work?

INCREASED OUTFLOW OF AQUEOUS HUMOR. Topical prostaglandins (LATANOPROST) are a preferred treatment for open-angle glaucoma

Episodic headache, tachycardia, diaphoresis are highly suggestive of what? How is it confirmed?

INCREASED RELEASE OF CATECHOLAMINES (PHEOCHROMOCYTOMA)...from chromaffin cells of adrenal medulla. Confirmed via elevated levels of catecholamines & metanephrines (catecholamine breakdown products) in urine or plasma confirms diagnoses.

What compensatory mechanism prevents peripheral edema in a patient with COPD an cor pulmonale

INCREASED TISSUE LYMPHATIC DRAINAGE ...right heart failure increases CVP leading to a rise in capillary hydrostatic pressure, net plasma filtation and interstitial fluid pressure. As INTERSTITIAL FLUID PRESSURE INCREASES SO DOES LYMPHATIC DRAINAGE TO COMPENSATE

Patient presents with lithium toxicity 2/2 NSAID use and requires urgent hemodialysis to lower his blood lithium levels, what would incease the rate of drug removal?

INCREASING SURFACE AREA OF THE MEMBRANE ~ diffusion speed acorss a semipermeable membrane increase with higher molecule concentration gradients, larger membrane surface areas & increased solubility of the diffusing substance

How does getting the squatting position improve symptoms in someone with tetralogy of fallot?

INCREASING SYSTEMIC VASCULAR RESISTANCE...decreases right-to-left shunting, thereby INCREASING pulmonary blood flow and improving oxygenation status.

5-month old brought to office for right scrotal enlargement (fluid collection around right testes) this embryological defect can lead to what?

INDIRECT INGUINAL HERNIA (hydrocoele from patent processus vaginalis)

Failure of a patient's Neutrophils to turn Blue on NITROBLUE TETRAZOLIUM TEST?

INFECTION CAUSED BY CATALASE-POSITIVE ORGANISMS. CGD = failure of NEUTROPHILS to turn blue on NITROBLUE TETRAZOLIUM TEST is characteristic of CHRONIC GRANULOMATOUS DISEASE (x-linked mutation infecting NADPH oxidase)

Surgeon is repairing a hernia, which landmarks would be used to distinguish an indirect from a direct hernia?

INFERIOR EPIGASTRIC VESSELS (indirect is lateral to inferior epigastric vessels) [direct hernias are located medially to inferior epigastric vessels and they do NOT protrude into the scrotum]

Contrast image of femoral artery with contrast and another artery is labeled and is more medial and shooting in the upward direction, what artery?

INFERIOR EPIGASTRIC branch of the external iliac artery takes of immediately proximal to the inguinal ligament. Provides blood supply to the lower anterior abdominal wall as it runs superiorly and medially up the abdomen.

Ct of a woman who got a lot of UTIs as a kid, it's showing a horseshoe kidney and question is asking what event prevented the proper ascent of the organ seen on CT?

INFERIOR MESENTERIC ARTERY ~ horseshoe kidney has kidneys fused at the poles, and the isthmus of the horseshoe kidney usually lies anterior to the aorta and posterior to the IMA. IMA limits ascent of horshoe kidney

Patient with rectal bleeding has bulging purplish-blue mucosal lesions above the dentate line...ligation of the vessels is likely to involve what?

INFERIOR MESENTERIC VEIN TRIBUTARIES

Man is stabbed in upper right quadrant pringle manueve is done and he is still bleeding...what is pringle manuever and where is blood coming from?

INFERIOR VENA CAVA (occlusion of portal triad iin the hepatoduodenal ligament = pringle manuever)

Patient hooked up to GnRH infusion, what would be the indication for that?

INFERTILITY...pulse administration of GnRH stimulates FSH & LH release and is useful for infertility

Change in location of pain during appendicitis, what change causes the major pain?

INFLAMMATION OF THE PARIETAL PERITONEUM, this causes a more somatic pain that shifts from the umbilical region to McBurney point (1/3 from ASIS)

28 year old woman with eye irritation and double vision...also has weight loss, mood swings and heart palpitations. PE: bilateral PROptosis given meds and sent home, how did drugs work?

INFLAMMATORY INFILTRATION....she has hyperthyroidism 2/2 Graves disease. Retro-orbital fibroblasts are stimulated by cytokines released from infiltrating Th1 cells to produce excessive amounts of GAGs. High dose GLUCOCORTICOIDS (prednisone...goal to decrease inflammation) are used to control severe Grave's opthalmopathy

Man with alzheimers dementia brought into hospital by daughter(power of attorney) man has UTI and health team forgets to give him meds for 2 days, nothing bad happens but he has to be in hospital for two days longer...what should be done?

INFORM THE DAUGHTER AND DISCOLE THE ERROR WITH AN APOLOGY

Man is having a heart attack and is rushed to the hospital, woman comes into ER claiming to be his wife and wants to know if he is okay, what should physician say?

INFORM THE WOMAN THAT THE PATIENT IS STABLE, BUT THAT SHE WILL HAVE TO WAIT UNTIL HE CAN GIVE PERMISSION TO SHARE ANY DETAILS...basic information can be shared under HIPAA

Where would you want to provide anesthesia for someone undergoing a quad tendon repair?

INGUINAL CREASE ~ a femoral nerve block at the inguinal crease with anesthetize the skin and muscles of the anterior thigh (quad, femur & knee)

Hwo do you reverse risperidone toxicity (neuroleptic malginant syndrome)?

INHIBITION OF CALCIUM ION RELEASE FROM SARCOPLASMIC RETICULUM OF SKELETAL MUSCLE (DANTROLENE = DIRECT-ACTING SKELETAL MUSCLE RELAXANT)

Woman with painless bleeding after sex, cervical biopsy shows malignant squamous cells, E6 & E7 has integrated into the host genome...what is likely mechanism for their involvement?

INHIBITION OF CELL CYCLE REGULATORY PROTEINS patient has carcinome of cervix 2/2 HPV....HPV relies on the inhibitory effects of viral proteins E6 & E7 on the cell cycle regulatory proteins p53 & Rb. This allows cells infected with HPV to undergo unchecked cell proliferation and evade apopotsis = genomic instability & malignant transformation

What action does grapefruit juice have on drugs?

INHIBITION OF CYTOCHROME P450 ENZYMES IN THE GUT WALL ~ calcineurin inhibitor nephrotoxicity with resultant impairement of renal function if the most significant effect of CYCLOSPORINE. CYP3A is responsible for cyclosporine metabolism in the small intestine and liver. Grapefruit juice inhibits enzyme and increases nephrotoxicity of cyclosporine by raising circulating drug levels.

Mechanism of action of Terbinafine?

INHIBITION OF SQUALENE EPOXIDASE (this inhibits synthesis of fungal membrane ergosterol)

Patient given a vaccine containing inactivated viral components that he normally receives each year...he is exposed to live virus through natural infection, the patient's prior exposure to the vaccine is likely to result in what?

INHIBITION OF VIRAL ENTRY INTO THE CELLS (inactivated/killed vaccines generate a humoral immune response instead of a cell-mediated immune response. Live-attenuated viral vaccines can generate a strong cell-mediated immune response in addition to providing humor immunity.

Woman has episodes of intractable vomitting and feeling like the room is spinning, dysfunction of what structure explains symptoms?

INNER EAR woman is experiencing a dysfunction in the vestibular system & experience vertigo. (vertigo ~ vestibular)

Woman has viral RNA causing headache and myalgias and new onset flaccid paralysis...she is confused and has a rash on her trunk & arms. The agent uses what modes of transmission?

INSECT BITE ~ west nile virus = ss flavivirus transmitted by female CULEX mosquiotoes. NEUROINVASIVE: encephalitis, meningitis, flaccid paralysis.

Looking to implant a pacemaker, application of the radiofrequency at which of the following locations is most likely correct?

INTERATRIAL SEPTUM NEAR THE OPENING OF THE CORONARY SINUS ~ The AV node is located on the endocardial surface of the right atrium, near the insertion of the septal leaflet of the tricuspid valve and the orifice of the coronary sinus.

Ribavirin mechanism of action?

INTERFERING WITH THE DUPLICATION OF VIRAL GENETIC MATERIAL [includes a lethal hypermutation causing a defective 5' cap formation on viral mRNA transcripts]

Patient has pure motor weakness affecting the contralateral arm, leg and low face + clasp knife spasiticity...where is the lesion?

INTERNAL CAPSULE STROKE (upper motor neuron lesion causing spastic rigidity, hyperreflexia & paresis...can affect any part of the pyrimidal motor system, including the corticospinal tracts of spinal cord, medulla, pons, midbrain, internal capsule and precentral gyrus (primary motor cortex)

Retinal artery occlusion (sudden onset vision loss, cherry-red spot in the macula) what is path of embolus?

INTERNAL carotid -> OPTHALMIC artery -> RETINAL artery

(following the old dementia man UTI) more cases of expired medications fail to be renewed, what is the most appropriate next step?

INTERVIEW PHARMACY, NURSING, MEDICAL STAFF ON UNITS WHERE THIS OCCURRED = root cause analysis (what, how, why these events occurred)

Man has seizure (new) and cavernous hemangioma is found, what is he at highest risk for?

INTRACEREBRAL HEMORRHAGE ~ cavernous hemangiomas are vascular malformations that occur most commonly within the brain parenchyma; they carry an increased risk of intracerebral hemorrhage and seizure

Patient has unilateral hearing loss in the setting of facial numbness and weakness is highly suggestive of what?

INTRACRANIAL SCHWANOMA = BETWEEN THE CEREBELLUM AND THE LATERAL PONS (at the cerebellar pontine angle)

Woman found at the store unconscious she has type 1 diabetes, her blood glucose is 34 mg/dL what is best way to normalize her blood glucose until ambulance arrives?

INTRAMUSCULAR GLUCAGON (if this was a medical setting you could treat the sever HYPOglycemia with IV glucose[50% dextrose ampule]

3 week old baby has palpable swelling in the neck, continues to feed well but appears comfortable only when held sideways...childlikes looking to right and cries when head is turned to the left....there is also a firm swelling on the left side of his neck, what is going on?

INTRAUTERINE MALPOSITION = Congenital Torticollis (a soft tissue mass may be palpable in the inferior 1/3 of the affected SCM. Stretching & Therapy is dx

Survey programs has found increasing incidence of healthcare-related bacteremia sespsis from straphylococcal infections, this is due to an increase in what?

INTRAVASCULAR DEVICES ~ intravascular catheters are indeispensable but they cause a LOT of central venous catheter-associated bloodstream infections

Research focusing on renal tubular transport proteins, substance identified that specifically and completely inhibits glucose transport in proximal renal tubules...in a participant glucose clearance would best approximate the clearance of what?

INULIN...glucose is normally filtered at the glomerulus and completely reabsorbed, inhibition of this Na-coupled carrier mediated transport of glucose would cause Glucose = GFR = clearance of inulin

older man with persistent headache and pain in the jaw when chewing food, histo shows multinuclear giant cells and inernal elastic membrane fragmentation, therapy is likely going to reduce the risk for what?

ISCHEMIC OPTIC NEUROPATHY ~ giant cell arteritis = characterized by granulomatous inflammation of the media, with fragmentation of the interal elastic lamine of medium and small branches of the carotid artery. Irreversible blindness is a severe complication of GCA (require immediate glucocorticoid therapy)

Substance1 without Substance2 has a faster reaction rate than Substance1 with Substance2 but the maximum velocity is still eventually reached, describe the relationship...

IT IS A COMPETITIVE INHIBITOR OF THE ENZYME ...competitive inhibitors compete with the substrate for active binding sites on enzymes.

Picture of a heart labeled Rv and LAD with a "slit-like tear" in the infected myocardium...when does it happen and what is it?

IT TYPICALLY OCCURS WITHIN 5-14 DAYS AFTER AN ACUTE MI it is a FREE WALL RUPTURE (catastrophic mechanical complication of transmural MI)

What is the ONLY drug that slows heartrate with no effect on contractility? How does it work?

IVABRADINE is the only drug that SLOWS HEARTRATE(negative chronotropic effect) with no effect on cardiac contractility. Uses FUNNY SODIUM CHANNELS PROLONGING THE SLOW DEPOLARATION PHASE.

Man with febrile illness...lives in eastern massacusetts has splenomegaly, no rash/wounds...+intraerythrocytic organisms on peripheral blood smear. what is source of infection?

IXODES TICK ~ babesiosus (blood smear shows INTRAERYTHROCYTIC RING INCLUSIONS)

What are the two most important opsonins (coating proteins)>

IgG & C3b...opsonization occurs when host proteins such as immunoglobulins or complement bind to foreign cells such as bacteria and coat the surface, enhancing phagocytosis. Both IgG and C3b faciliatate phagocytosis.

Woman is pregnant and after reviewing her labs the physican administers an injection of anti-Rh(D) immunoglobulin, administered antibodies most likely belong to what class of Ig?

IgG....Anti-Rh immune globulin consists of IgG and anti-D antibodies that opsonize Rh+ fetal erythrocytes,promoting clearance by maternal reticuloendothelial macrophages and preventing maternal Rh sensitization. It is routinely administed to Rh-negative women at 28 weeks gestation and immediately postpartum

Moms blood(B negative) Baby blood(A negative) mom's blood has anti-A antibodies circling, how come hemolysis did not occur in the baby (aka what class were mom's antibodies)

IgM (cannot cross the placenta) with maternal blood types A&B erythroblastosis fetalis and hemolytic disease of the newborn do not occur as the naturally occuring antibodies (anti-a/b) are IgM and cannot cross the placenta. [in type O mothers the antibodies are predominantly IgG and can cross causing hemolysis]

What is the serum of a rheumatoid arthritis patient going to contain (anti what?)

IgM antibodies against Fc portion of human IgG, Positive rheumatoid factor. ANTI-CCP peptide

What is the most important hip flexor muscle (use to sit up without using hands)

Iliopsoas = psoas major + minor + iliacus [external abdominal obliques and rectus abdominus are also used]

What anatomical landmark is best used for lumbar puncture?

Illiac Crest

What is the signifcance of the paramesonephric ducts and what can happen during disruption?

In females the paramesonephric ducts fuse to form fallopian tubes, uterus, cervix & upper vagina...disruptions can lead to a variety of muellerian tract anomolies. INCOMPLETE LATERAL FUSION OF THE PARAMESONEPHRIC DUCTS RESULTS IN A BICORNUATE UTERUS (WILL HAVE INDENTATION IN CENTER OF FUNDUS)

What is a Gumma?

In late (tertiary) syphilis ...years after infection,Gummas show up as painless, indurated granulomatous lesions that progress to white-gray rubbery lesions that may ulcerate. Tertiary syphillis also has ascending aortic aneurysms and aortic valve insufficiency(murmur, high-pitched tambour S2)....neurosyphillis can occur at any stage.

Effects of DDAVP/desmopressin

In mild hemophilia A & type 1 von Williebrand disease DDAVP increases circulating factor 8 & endothelial secretion of vWF to stop bleeding. INCREASE IN ENDOTHELIAL PROTEIN RELEASE.

Patient with bone pain, fatigue, anemia, kidney disease & hypercalcemia...high affinity for proteasome catalytic site, what is the result?

In multiple myeloma neoplastic plasma cells make a lot of monoclonal immunoglobin or immunoglobulin fragments...the cells are susceptible to proteasome inhibition due to large amout of protein they make...toxic amounts of proapoptotic proteins increases cellular apoptosis.

What is incidence and what is special about calculating it?

Incidence is the number of new cases of a disease in a certain population at risk over a given time period [new cases/population - old cases]. Prevalence is the total number of cases in the population over a given time period.

incidence vs. prevalence?

Incidence is the number of new cases of a disease in a certain population at risk over a given time period. Prevalence is the total number of cases in the population over a given time period.

Pulmonary function testing of someone with COPD?

Increased TLC, Decreased FEV1/FVC, Decreased FVC, Increased residual volume

Relationship between statins and myopathy?

Increased hepatocyte LDL receptor recycling. ALSO CAUSE MUSCLE PAIN. Statins inhibit HMG-CoA reductase, the rate limiting step in cholesterol synthesis...this then requires hepatocytes to increase their surface expression of LDL receptors and take up more LDL = huge reduction in cholesterol and LDL concentration.

How does airway resistance change as you move from the trachea to the terminal bronchioles?

Increases from trachea to medium sized bronchi, then decreases for bronchioles and decreases even more for terminal bronchioles. FIRST TEN GENERATIONS OF BRONCHI CONTRIBUTES TO MOST OF THE TOTAL AIRWAY RESISTANCE OF THE LOWER RESPIRATORY TRACT. [RESISTANCE IS MAXIMAL IN THE 2ND TO 5TH GENERATION AIRWAYS, INCLUDING SEGMENTAL BRONCHI]

The individualized subunits of hemoglobin are structurally analogous to what? How would it look on oxy-hemo dissociation curve?

Individual subunits of hemoglobin are structurally analogous to MYOGLOBIN...if separated the monomeric subunits will demonstrate hyperbolic oxygen-dissociation curve similar to that of myoglobin

Inherited defects involving what result in disseminated mycobacterial disease?

Inherited defects in INTERFERON-GAMMA SIGNALING pathway result in disseminated mycobacterial disease in infacy/childhood

Patient taking statins concomitant administration of ERYTHROMYCIN would result in what?

Inhibition of cytochrome 3A4...which can result in statin-induced myopathy & rhabdomyolysis.

Patient is 80 and has smoked for 50 years, has COPD and has been hospitalized multiple times....what is important to cover during the rest of the admissions process?

Inquiring about the presence of ADVANCED DIRECTIVES and the patient's wishes for end of life care.

what is essential for the elimination of the bacterium Listeria Monocytogenes?

Intact CELL-MEDIATED IMMUNITY is essential for the elimination of the listeria monocytogenes bacteria.

Hemidesmosomes use what kind of protein?

Integrins

Baby born with abnormal sexual differentiation...karyotype: 46 XY, biopsy of gonadal tissue shows lack of sertoli cells but normally functioning leydig cells...what phenotype?

Internal(male&female) External(male). Leydig is responsible for testosterone[->DHT] which is responsible for external genitalia(hence male external) Sertoli is responsible for anti-muellerian hormone but it wasn't there you get get muellerian(woman) stuff

Progressive dyspnea, fine crackles, clubbing & diffuse reticular opacities

Interstitial lung disease [radial traction is increased in pulmonary fibrosis] [radial traction is DECREASED in emphysema]

What is the biochemical target of pioglitazone

Intracellular nuclear receptor ~ Thiazolidinediones exert their glucose-lowering effect by improving insulin sensitivity...they binds to PPAR-gamma an intracellular nuclear receptor that acts as a transcriptional regular of many genes involved in glucose & lipid metabolism

What happens to intra & extra cellular potassium during diabetic ketoacidosis?

Intracellular potassium moves out (increased extracellular, decreased intracellular). But most DKA patients normal to increased serum potassium levels despite a total body potassium deficit. Replacement of potassium is a crucial step in DKA management.

What can result from Vitamin A overuse?

Intracranial hypertension, skin changes and hepatosplenomegaly

What is the most important prognostic marker in a bladder cancer?

Involvement of the muscular layer...the majority of bladder carcinomas are urotherlial (transitional) cell carcinomas [painless gross hematuria is a characteristic sign] TUMOR PENETRATION OF THE BLADDER WALL (MUSCULAR LAYER INVOLVEMENT) IS THE MAJOR DETERMINANT OF PROGNOSIS.

Study where vagus nerve is removed in pigs then drug is administered and the drug significantly increases bronchial diameter but no effect in subjects without a vagus nerve, what is the drug?

Ipratropium, an anticholinergic agent (derivative of atropine)...blocks the action of Ach at muscarinic receptors, preventing bronchoconstriction and reducing the parasympathetic stimulation

patient has perforated peptic ulcer and as a result partial gastrectomy & gastrojejunostomy are performed what long-term supplement will the patient likely need?

Iron....iron absorption occurs predominantly in the duodenum and proximal jejunum...bypass can result in an iron deficiency...malabsoprtion of iron is the big dog problem

What is recall bias?

It reflects the inability to recall specific events that may or may not have lead to an outcome. [often seen in retrospective studies like case-control] especially because people who have suffered an adverse event are MORE likely to recall isk factors. This bias is a threat to the validity of a study.

Polycythema Vera mutation and increased sensitivity to what?

JAK2/V617F mutation & INCREASED BONE MARROW SENSITIVITY TO GROWTH FACTORS

Woman post-cholecystectomy is curious about what part of digestive tract will absorb fats?

JEJUNUM (bile salts emulsify lipid breakdown products, forming water-soluble micelles that facilitate lipid absorption in the JEJUNUM (FATTY ACIDS, MONOGLYCERIDES, CHOLESTEROL)

Replication of microbial cells requires synthesis of two daughter strands of DNA using the parent strands as templates....what processes will differe the most between the two daughter stands formed at each repliation fork?

JOINING OF DNA FRAGMENTS BY LIGASE...DNA replication occurs in 5'->3' direction on BOTH strandrs. Lagging strand occurs discontinously and is composed of short stretches of RNA primer plus newly synthesized DNA segments (okazaki fragments) so lagging strand synthesis requires the repetitive action of DNA primase & ligase

Elevated serum aldosterone levels can manifest with hypertension, hypokalemia and muscle weakness...what kind of tumor?

JUXTAGLOMERULA CELL TUMOR it increases levels of renin and aldosterone and is indicative of secondary hyperaldosteronism: renovascular disease, malignant hypertension & renin-secreting tumors

Patient tries rare fish at japanese restaurant -> weakness, dizziness, parasthesia...what kind of poisoning? What kind of toxin?

Japanese Pufferfish poisoning from TETRODOXIN [binds to VOLTAGE-GATED SODIUM channels in nerve & cardiac tissue...preventing sodium influx and depolarization]

Question gives you standard deviation, average and wants to know where 95% of values fall...

Just take 2 times the standard deviation and add/substract it from the average

15 year old boy with periodic sudden-onset jerking movements involving both arms...what does he have and what drugs should be given?

Juvenile myoclinic epilepsy - BROAD-SPECTRUM ANTI-CONVULSANT (VALPROIC ACID)

Patient with pyruvate dehydrogenase complex deficiency (inherited inborn error of metabolism causing lactic acidosis & neuro defects) Patients unable to convert pyruvate to acetyl CoA what can provice energy in the form of acetyl CoA without increasing lactate production?

KETOGENIC AMINO ACIDS (lysine & leucine)

Activation (via a single nucleotide substitution) mutation what gene is likely involved?

KRAS = proto-oncogene. [apc, brca1, rb, tp53 = anti-oncogenes...their inactivation causes cancer]

Man has adenocarcinoma (6cm mass in ascending colon) hemicolectomy is performed but post-op shows the tumor has invaded several mesenteric lymph nodes and is expressing EGFR, therapy with an inibitor is considered...an activating mutation in what will make the therapy INEffective?

KRAS ~ KRAS MUTATION CAUSING ANTI-EGFR RESISTANCE...activating mutations of the KRAS gene lead to constitiutive activation of the EGFR pathway, promoting increased cell proliferation & growth. Tumors harboring these mutations are resistant to chemo with anti-EGFR drugs (cetuximab & panitumab)

(adenoma to carcinoma sequence) woman has 1 early adenomatous polyp and 1 late, a mutation in what would make early go to late?

KRAS...the size of adenomatous polyps determines their malignant potential. Adenomas <1cm are unlikely to undergo malignant transformation, wheras those >4cm are very likely to progress to adenocarcinoma. KRAS protooncogene mutation facilitates the growth of adenomas by causing uncontrolled cell prolferation.

Alcoholic with pneumonia symptoms (fever, chills, chest pain, productive cough) with blood-tinged sputum?

Klebsiella Pneumonia (encapsulated, lactose-fermenting, gram-negative bacillus that appears mucoid in culture) blood tinged = currant jelly sputum.

Man has gynecomastia, small/firm testes, really long lower extremities. Disease? Level of Testosterone/LH/FSH/Sperm count

Klienfelter (47, XXY) increased LH, increased FSH, decreased testosterone, no sperm count. [small firm testes...azoospermia]

Man confused and severe headache, nausea, nuchal rigidity, CSF: neutrophils & protein. gram stain of CSF would likely show what?

LANCET-SHAPED, GRAM-POSITIVE COCCI IN PAIRS ~ bacterial meningitis from Strep Pneumoniae (the most common cause of bacterial meningitis in adults of all ages)

Normal cells found in epithelium have the shape of a tennis racquet and demonstrate some myeloid surface markers (also interact with T lymphocytes)

LANGERHANS CELLS = dendritic cells found in the skin that act as professional antigen presenting cells. These cells are derived from the myeloid cell line and they possess characteristic racquet-shaped intracytoplasmic granules known as BIRBECK GRANULES

12 year old boy with mild intellectual disability has cytogenetic studies done that shows a small gap near the tip of the long arm of the X chromosomes. What is likely on physical exam?

LARGE EARS, LONG FACE, MACROORCHIDISM = FRAGILE-X SYNDROME CGG repeats. FMR1 gene...dysmorphic facial features (large jaw, protruding ears, long face)

Picture of the chemical formula of nitroglycerin is shown and the question is what is the major target?

LARGE VEINS ~ nitroglycerin acts primarily as a venodilator, it decreases preload which decreases myocardial oxygen demand and thereby treats angina pectoris

Study where men who took antioxidants for 5 years had a lower stroke risk, what explains why the relative risk of stroke is lower with antioxidant use?

LATENT PERIOD ~ the concept of a latent period can be applied to both disease pathogenesis and exposure to risk modifiers.

Patient has diplopia and drooping eyelid, his eye is looking down and out...what muscle is likely to still work?

LATERAL RECTUS (LR6)...lesions in cranial nerve 3 cause ptosis, a downward and laterally deviated eye, impaired pupillary constriction and accomodation and diagonal diplopia. (most dreaded cause of CN3 palsy is an enlarging intracranial aneurysm)

Pulmonary artery occlusion pressure will have the same readout as what other pressures?

LEFT ATRIAL AND LEFT VENTRICULAR END-DIASTOLIC PRESSURES

Under normal circumstances, Pulmonary Capillary Wedge Pressure (PCWP) is closely associated with what pressures?

LEFT ATRIAL AND LEFT VENTRICULAR END-DIASTOLIC PRESSURES...MITRAL STENOSIS LEADS TO AN INCREASED LA PRESSURE THAT IS REFLECTED AS AN ELEVATED PCWP.

35 yo indian woman with history of rheumatic heart disease, has dysphagia...barium swallow reveals extrinsic compression of the mid-esophagus...enlargement of what is likely causing dysphagia?

LEFT ATRIUM - cardiovascular dysphagia can result from external compression of the esophagus by a dilated and posteriorly displaced left atrium in patients with rheumatic heart disease and mitral stenosis/regurgitation

Reduced vision in left eye: hemorrhage in left temporal hemiretina...transmission of visual information through which neural structures will be disrupted?

LEFT LATERAL GENICULATE BODY (also: ipsilateral optic nerve, lateral optic chiasm, optic tract, lateral geniculate body, optic radiations, primary visual cortex)

Man with type 2 diabetes has occasional dizziness doppler ultrasound of left vertebral artery shows retrograde (caudal) flow instead of normal anterograde flow. What artery is occluded?

LEFT SUBCLAVIAN ~ Subclavian steal syndrome occurs due to severe stenosis of the PROXIMAL SUBCLAVIAN ARTERY, which leads to reversal in blood flow from the contralateral vertebral artery to the ipsilateral vertebral artery. Patients may have symptoms related to arm ischemia in the affected extremity (exercise-induced fatigue, pain, paresthesias) or vertebrobasilar insufficiency (dizziness/vertigo)

What does the presence of hemosiderin-laden macrophages in pulmonary alveoli tell you?

LEFT VENTRICULAR DYSFUNCTION ~ prussian blue = iron. Presence of iron indicates chronic elevation of pulmonary capillary hydrostatic pressures, most commonly as a result of LEFT-SIDED HEART FAILURE

Baby with irritability, dystonia, poor feeding & MAPLE SYRUP scent to patient's urine...what should be avoided in diet?

LEUCINE, ISOLEUCINE, VALINE. MSUD (Maple Syrup Urine Disease)...autosomal recessive inborn error of metabolism due to branched-chain alpha-ketoacid dehydrogenase complex (BCKDC) deficiency. = neurotoxic from too much leucine

Person has pneumonia, has parapneumonic effusion...what components of that effusion would make neutrophils attack?

LEUKOTRIENE B4. Leukotriene B4 is the most potent chemotactic eicosinoid...it stimulates neutrophil migration to sites of inflammation [other important chemotactants include: 5-HETE(leukotriene precursor) complement component C5a & IL-8]

Patient with previous MI and suggested congestive heart failure due to LV systolic dysfunction...once his current condition is stabilized what medication should be prescribed?

LISINOPRIL ~ ACE inhibitors & Ang II receptor blockers are recommended in all patients with systolic heart failure. MORTALITY

What metabolic process underlies drug-induced lupus?

LIVER ACETYLATION ~ Drug induced lupus is characterized by abrupt onset of Lupus symptoms (fever, arthralgias, pleuritis) with positive anti-histone antibodies. It has been linked to drugs metabolized by N-acetylation in the liver (procainamide, hydralazine, isoniazid). Genetically predisposed individuals who are slow acetylators are at greater risk for developing DrugLE

Man forgot to eat after taking his insulin and passed out, glucagon was administered...metabolic changes in which of the following organs is most likely responsible for patient's recovery?

LIVER glucagon increases serum glucose by increasing hepatic glycogenolysis and gluconeogenesis....glucagon also stimulates insulin secretion from the pancreas.

Why is methadone so helpful in reducing cravings? What property?

LONG HALF LIFE. Methadone is a FULL mu-opiod receptor agonist its LONG HALF LIFE suppresses cravings and withdrawal symptoms. Methadone = drug of choice for opiod abuse

Autopsy of man shows: Enlarged Left Atrium, Increased myocardil mass, Increased left ventricular thickness...patients heart reflects what?

LONG-STANDING HYPERTENSION

Woman has prolactinoma what is she at greatest risk for?

LOSS OF BONE MASS ~ hyperprolactinemia causes suppression of gonadotropin-releasing hormone, which leads to reduced estrogen in women. Low estrogen levels are a risk factor for accelerated bone loss.

During operation the left glossopharyngeal nerve is accidentally transected, what is most likely to be seen

LOSS OF GENERAL SENSATION AT THE TONSILAR LINING (afferent loss of gag reflex, loss of sensation in the upper pharynx, posterior tongue, tonsils, middle ear cavity, loss of TASTE posterior 1/3 of tongue)

What is the classic presentation of legionella pneumophila and what is the most common lab abnormality?

LOW SERUM SODIUM(HYPONATREMIA) legionella pneumophila is a faculative intracellular gram-neg bacillus that can cause systemic infection (fever, cough, confusion, diarrrhea)

Woman has benign acquired melanocytic nevus, during histologic analysis her cells are found to contain a condensed body composed of heavily methyated DNA at the periphery of the nucleus...what is this region of DNA likely associated with?

LOW TRANSCRIPTION ACTIVITY [X-inactivation = lyonization, results in clusters of cells throughout the body that express either maternal or paternal X chromosome. Conversion of the inactivated X chromosome into compact hetereochromatin(barr body) composed of heavily methyated DNA in tight association with deacetylated histones.

Boy bought to ED with "right hand clumsiness" fell off a tree while climbing with a friend and now cannor perform fine finger movements with his right hand...what structure was injured?

LOWER TRUNK OF THE BRACHIAL PLEXUS

what is the true initiating event of acute appendicitis?

LUMEN OBSTRUCTION ...obstruction of the lumen of the appendix is the first event in pathogenesis of ACUTE APPENDICITIS....poop,lymphoid follicles, foreign bodies, tumors...can cause the obstruction

Woman with a history of breast cancer treated via radical mastectomy & radiation now has persistent right arm swelling, what is the patient at risk for?

LYMPHANGIOSARCOMA ~persistent lymphedema (with chronic dilation of lymphatic channels) predisposes to the development of lymphangiosarcoma, a rare malignant neoplasm of the endothelial lining of lymphatic channels. Cancer may rise approximately 10 years later

Woman has red rash on her chest for 2 months, rahs is itchy and feels rough to touch. Its gotten worse and there is right axillary lymphadenopathy (no breast massses) what is the cause?

LYMPHATIC OBSTRUCTION this is classic Peau d'organge (erythematous, itchy breast rash with skin textusre changes analogous to an organge peel. [this is a key derm presentation of inflammatory breast cancer and is caused by cancerous cells obstructing lymphatic drainage due to spread to the dermal lymphatic spaces].

What is attributed to the rubber-like properties of elastin?

LYSYL OXIDASE ....rubber-like properties of elastin are due to extensive cross-linking between elastin monomers....this is faciliated by

10% of patients lamigotrine will develop what? What is MOA of lamigotrine?

Lamotrigine is an anticonvulsant that works by blocking voltage-gated sodium channels (tx seisures and bipolar)...up to 10% of patients will develop a benign rash...more severe rash SJS/TEN could result

Gram positive, alpha hemolyticm optochin-sensitive, bile-soluble diplococci?

Lancet Shaped Strep Pneumoniae

Differential clubbing and cyanosis without blood pressure or pulse discrepancy are pathognomonic for what?

Large patent ductus arteriosus complicated by Eisenmenger syndrome (reversal of shunt flow from left-to-right to right-to-left)

Major key in difference between ADHD and Learning disorder?

Learning disorders are characterized by difficuluties with key academic skills (reading, writing, math) this results in performance well below expectations for age level.

Non-pharmacologic treatments for insomnia include: sleep hygiene, stimulus control, relaxation, sleep restriction & CBT. Describe stimulus control:

Leaving the bedroom if unable to fall asleep within 20 minutes.

Patient brought in with esophageal varices (chronic drinker) thanks to portal hypertension, where did this blood come from?

Left Gastric Vein(portal) Esophageal(systemic)

Peptic ulcer on lesser curvature of stomach, what artery most likely messed up?

Left gastric artery

Man is stabbed: laterally directed anterior chest wall stab wound at the fifth intercostal space along the left midclavicular line. What structure injured?

Left lung

How can left-sided heart disease cause pulmonary hypertension?

Left sided heart disease can cause pulmonary hypertension by increasing pulmonary venous pressure and congestion (this leads to a passive increase in pulmonary arterial pressure which is worstened by reactive vasoconstriction)

In biventricular pacemakers where do the left ventricuar leads course through?

Left ventricular leads in biventricular pacemakers course through the coronary sinus, which resides in the atrioventricular groove on the POSTERIOR aspect of the hear

Hospital has increased nosocomial pneumonias, bug is cultured on buffered charcoal yeast extract agar supplemented with L-cysteine & iron. What bug and how is it contaminating hospital?

Legionella Pneumonia = COLONIZATION OF THE HOSPITAL WATER SYSTEM

Recent onset of fever, confusion, headache, diarrhea and mild cough...smoker...sputum gram stain reveals numerous neutrophils but no bacteria, what is the cause of patient's disease?

Legionella Pneumonia = COLONIZATION OF THE HOSPITAL WATER SYSTEM; doesn't always STAIN!

What is seen in patients with galactosemia?

Lenticular accumulation of galactitol in lenses of patients with galactosemia can cause osmotic damage and development of cataracts (cataracts may be the only manifestation of galactokinase deficiency).

Patient wants adjustable gastric band, what structure must the band pass through in order to successfully surround the stomach?

Lesser omentum [double layer of peritoneum that extends from the liver to the lesser curvature of the stomach and beginning of the duodenum]...it is divided into the hepatogastric & hepatoduodenal ligaments

Female pelvic floor muscle strengthening (due to droppin some pee when coughing/laughing) what muscle is targeted?

Levator Ani...pelvic floor/kegel exercises target the levator ani to improve support around urethra and bladder.

Lumbar spinal stenosis relieved by leaning forward on the walker/stroller...what ligament has become thickened?

Ligamentum flavum(ligament). Intervertebal disc herniation, osteophyte formation

Multiple Sclerosis can cause demylination of an optic nerve, in question stem it was left...how would pupillary light reflex be effected?

Light into right eye = both right & left constrict. Light into left eye = neither constrict

What bug is characterized by gastroenteritis, septicemia & meningoencephalitis?

Listeria Monocytogenes...it most commonly effects patients with depressed cell-mediated immunity. Outbreaks have been seen with DAIRY sources.

Woman getting treatment for mood swings and sleep problems begins losing hair, constipation, dry skin, gaining weight...what med is responsible?

Lithium....hypothyroidism and nephrogenic diabetes insipidus are the most common adverse effects of long-term lithium therapy. Serum TSH levels and renal function (BUN) should be monitored

Infant born to a 27 year old immigrant is evaluated for hearing loss, physical exam shows white pupils, continuous machine-like murmur...what could have prevented this?

Live-attenuated vaccine. Mother had Rubella...kid has congenital rubella (remember MMR is live attenuated, but is contraindicated in pregnancy so give to kids and to women a child-bearing age who are not pregnant(.

What organ has dual/collateral blood supply and therefore would not be as risk for infarction following a left atrium occlusion?

Liver [CNS would be most likely to infarct]

Pregnancy increases risk for DVT, what drug is a great choice

Low molecular weight heparins (such as enoxaprin are best treatment choice)

Saggital image of head/brain asking you to identify the area responsible for vomitting currenly being experienced by a chemo patient?

Low on 4th ventricle. Acute nausea following administration of systemic chemotherapy results from stimulation of the chemoreceptor trigger zone (CTZ), which lies in the area postrema of the dorsal medulla near the 4th ventricle

What has been the leading cause of cancer mortality in women & men in the US since the 1980s?

Lung Cancer [tobacco use -> small cell lung cancer]. Graph will have a dramatic increase.

Man is stabbed on the right side above the clavicle between mid clavicular and lateral sternal lines...what structure was most likely injured?

Lung Pleura

Kid has 3 days burning with urination gram stain shows gram negative intracellular diplococci...what is the tx?

MACROLIDE & 3RD GEN CEPHALOSPORIN = Ceftriaxone & Azithromycin

Woman presents with toxic shock sydrome, what cell types a responsible for her rash and fever?

MACROPHAGES & T LYMPHOCYTES (staph aureus) They interact with MHC complex on APCs and cause a non-specific widespread activation of T cells (releasing IL-2 from T cells and IL-1 and TNF from macrophages

Autopsy on 14 year old kid who died in a car accident, shows several minimally raised yellow spots on the inner surface of the adbominal aorta...what is the mostlikely predominant cell type in these lesions on LM?

MACROPHAGES ~ fatty streaks are the earliest lesions of atherosclerosis and can be seen as early as the second decade of life. They appear as a collection of lipid-laden macrophages/FOAM CELLS in the INTIMA that can progress to atherosclerotic plaques

Areas of keratinization are shown on histology of a man who is a smoker with new difficulty swallowing, what is likely prognosis?

MALIGNANCY WITH PROBABLY POOR PROGNOSIS ~ keratin pearls ~ squamous cell carcinoma

Science experiment: beta-oxidation of fatty acids is inhibited, an increase in what substances is most likely responsible for the observed effect?

MALONYL-CoA. The rate-limiting step of de novo fatty acid synthesis is Cystosolic acetyl-CoA carboxylase converts acetyl-CoA to malonyl-CoA...malonyl-CoA also inhibits the action of mitochondrial carnitine acyltransferase INHIBITING BETA-OXIDATION of newly formed fatty acids

Woman brought to physician for follow-up with her husband(he's concerned because she doesn't take her medicine) she admits to non-compliance, appropriate response?

MANY PATIENTS FIND IT DIFFICULT TO TAKE MEDICATION EVERY DAY, TELL ME MORE ABOUT WHAT MAKES IT HARD FOR YOU? OPEN ENDED NON-JUDGMENTAL

Experiment: cells purified so that only deoxythmidine residues are left, what type of nucleic aci is most likely to bind the strongest to latex beads?

MATURE mRNA, the poly-A tail is not transcribed from DNA it is added as a post-transcriptional modification downstream of the consensus sequence AAUAAA located near the 3' end of the mRNA molecule. This tail protects mRNA from degradation within the cytoplasm after it exits the nucleus.

Woman brought to with linear skull fracture at junction of frontal, parietal, temporal & sphenoid bones...a branch of what artery is likely severed?

MAXILLARY (middle meningeal artery, enters skull at the foramen spinosum) Fracture at the pterion --> epidural hematoma

What is the MCL and what is important about this Uworld question?

MCL = medial collateral ligament ...resists force pushes on the knee medially...increased laxity of the knee with the valgus stress test indicates injury to the MCL. DAMN UWORLD GIVES YOU THE ANTERIOR VIEW OF THE LEG PAY ATTENTION

Man with sudden-onset vomiting and severe upper abdominal pain that radiates to his back, AST:ALT is greater than 2 acute pancreatitis is diagnosed what additional lab finding Is MOST specific?

MEAN CORPUSCULAR VOLUME OF 108fL ~ after gallstones, alcohol abuse is the second most common cause of acute pancreatitis. Macrocytosis and an AST:ALT ratio >2 are indirect indicators of chronic alcohol consumption. Alcohol-related macrocytosis can occur INDEPENTLY of FOLATE DEFICIENCY.

Elevated creatine kinase is suggestive of a myopathy somewhere, what test could give us a better idea in finding the cause?

MEASURING TSH. Common causes of myopathy with elevated CK include: HYPOthyroidism, muscular dystrophies, inflammatory muscle disease, HMG-CoA reductase inhibitors. MEASURE TSH.

Woman falls and fractures her hip, what artery being injured most likely leads to osteonecrosis in the patient?

MEDIAL CIRCUMFLEX ~ the MEDIAL circumflex artery and its branches feed the FEMORAL HEAD AND NECK.

5 year old caucasian boy with recent-onset gait instability and gait ataxia, MRI shows midline posterior fossa mass and biopsy shows primative cells and amny mitotic figures. What is the patient likely suffering from?

MEDILLOBLASTOMA ~ second most common brain neoplasm of childhood. It is located in the cerebellum, often at the vermis, and consists of sheets of small, blue cells. Like other "PNET" tumors, medulloblastomas are poorly differentiated and have a bad prognosis

Woman comes to ED due to 6 days of fatigue and intermittent fevers, recently returned from central africa...she is also pregnant, what is best med?

MEFLOQUINE ...she has PLASMODIUM infection (blood smear shows RBCs filled with multiple, smaller rings)

What is responsible for the toxic events observed in meningitis and meningococcemia?

MEGNINGOCOCCAL LIPOOLIGOSACCHARIDE (LOS) ....blood levels of LOS correlate closely with morbity and mortality

Kid born with harsh, holocystolic murmur to a 36 year old woman, kid has flat face, protruding tongue, small ears...what genetic thing happened during embryogeneiss?

MEIOTIC NONDISJUNCTION (21, 18, 13...D.E.P) Down's most commonly caused by MATERNAL MEIOTIC NONDISJUNCTION...flat face, protruding tongue, small ears

10 year old girl being evaluated for short stature: height in 5th percentile, weight is in 50th percentile, no breast buds, no axillary hair, low hairline, short wide neck, broad chest, widely-spaced nipples. What genetic process caused this?

MEIOTIC NONDISJUNCTION she has Turner Syndrome (45X) most commonly caused by PATERNAL MEIOTIC NONDISJUNCTION during gametogenesis. Lose X chromosome, lose SHOX gene which is responsible for long bone growth

Unprovoked syncope in a previously asymptomatic young person from a congenital QT prolongation syndrome...what is going on.

MEMBRANE POTASSIUM CHANNEL PROTEINS~ two most important congenital syndromes with QT prolongation - Romano-Ward syndrome and Jervell and Lange-Nielsen syndrome - throught to result from mutations in K+ channel protein

Man has mental status change and double vision, memory has been declining over the years and neuro exam shows horizontal nystagmus evoked on lateral gaze with bilateral weakness of the lateral rectus muscles. Thiamine is infused and he recovers but what will not recover?

MEMORY LOSS ~ man has Wernicke Encephalopathy (oculomotor dysfunction, ataxia, confusion (Wernicke triad)...most symptoms resolve after thiamine administration. Korsakoff is a complication resulting in memory loss and cofabulation

Enlarged thyroid, elevated calcitonin level, mucosal neuromas & marfanoid habitus (long arms/fingers)

MEN2b - multiple endocrine neoplasia type 2b [caused by germline mutations of RET proto-onco gene. Pheochromocytomas can results - flushing, headaches, paroxysmal hypertension, diaphoresis.

Low frequency tinnitis/ringing (with feeling of fullness), Vertigo, Sensorineural hearing loss which worstens over time...whats up?

MENIERE disease = increased volume & pressure of ENDOLYMPH in the vestibular apparatus.

What is the most common presentation of Cryptococcus Neoformans infection?

MENINGOENCEPHALITIS = Cryptococcus Neoformans = MUCICARMINE staining of lung tissue and bronchoalveolar washing.

How can hemorrhagic cystits in a chemotherapuetic patient be prevented?

MESNA (2-MERCAPTOETHANSULFONATE)

Man has CML and team uses reverse transcription polymerase chain reaction to diagnose...what will be detected by this test?

MESSENGER RNA TRANSCRIPT CONTAINING BCR & ABL EXONS ...uses reverse transcription to create a complementary DNA template that is then amplified using standard PCR procedure.

23 year old man with cramping abdominal pain and weakness for 2 weeks, blood smear shows basophilic stippling on a background of hypochromic microcytic anemia...what is the likely cause?

METAL POISONING ~ LEAD POISONING (basophillic stippling & microcytic anemia) young children ingesting paint chips or industrial workers are at risk

Hepatocellular carcinomacells are going to invade the basement membrane, what type of substance is expressed in order to do so?

METALLOPROTEINASES (zinc-containing enzymes that degraded components of ECM & basement membrane) ~ likelihoof of plaque rupture is related to plaque stability rather than plaque size of the degree of luminal narrowing. Plaque stability largely depends on the mechanical strength of the fibrous cap. INFLAMMATORY MACROPHAGES REDUCE PL

1 Year ago man had a nonobstructive atherosclerotic plaque with a hypodense core than occupies 40% of the lumen, now he is in Ed with acute severe chest pain and is found to have thrombotic occlusion of the proximal left anterior descending artery. What intraplaque activity of what enzyme most likely resulted in the MI?

METALLOPROTEINASES ~ likelihoof of plaque rupture is related to plaque stability rather than plaque size of the degree of luminal narrowing. Plaque stability largely depends on the mechanical strength of the fibrous cap. INFLAMMATORY MACROPHAGES REDUCE PLAQUE STABILITY BY SECRETING METALLOPROTEINASES

Woman finally becomes pregnant, the oocyte was most likely arrested in what stages of meiosis, immediately prior to fertilization?

METAPHASE OF MEIOSIS II ~ prior to fertilization, secondary oocytes are arrested in metaphase of meiosis II. Primary oocytes are completely developed in female embryos by the fifthem month of gestation, at which point they are arrested in prophase of meiosis I

Woman comes in with periodic reddining of face/neck+warmth, PE: purple vascular lesions around her nose...she pees out 5-HIAA and imaging shows tumor in small intestine, describe patient's condition?

METASTATIC CARCINOID...Intestinal carcinoids that metastasize to the liver and extraintestinal carcinoids release vasoactive substances that avoid first-pass metabolism resulting in carcinoid syndrome

MOA of methimazole monotherapy (and relationship to propylthioruracil)

METHIMAZOLE = COUPLING OF IODOTYROSINES ....propylthiouracil decreases conversion of T4 to active hormone T3

Woman has a UTI and is started on Trimethoprim...what other drug has the same intracellular target and what is it?

METHOTREXATE ~ folate antimetabolite INHIBITS DIHYDROFOLATE REDUCTASE

Nodular and commendal acne in a patient in their 20s, what jumps to top of list as a reason?

METHYLTESTOSTERONE ~ anabolic steroids. Methyltestosterone is an androgen, steroids promote follicular epidermal hyperproliferation & excessive sebum production.

Myasthenia Gravis is associated with abnormalities of what? What pharyngeal pouch is it derived from?

MG is associated with Thymus abnormalities (thymoma, thymic hyperplasia). Both the THYMUS & INFERIOR PARATHYROID GLANDS arise from the 3rd pouch.

34 year old woman symptoms of myasthenia gravis, what is MG and how is Goodpasture Syndrome related?

MG is caused by interference of IgG autoantibodies [impaired functioning of nicotinic cholinergic receptors at the neuromuscular junction from IgG autoantibody interference]. Goodpasture is autoantibodies directed at basement membrane collagen of renal glomeruli and lungs

What is the tensilon test?

MG patient currently take a long-acting acetylcholinesterase inhibitor (like pyridostigmine) has an exacerbation of MG (crisis) Patient is enfused with EDROPHONIUM(TENSILON) and if there is clinical improvement patient just needs more pyridostigmine since patient is currently undertreated.

What is important to remember about the MHC class I antigen presenting cells (what else is on them?)

MHC Class I + B2Microglobulin. CD8+ cells recognize foreign antigens presented with MHC class I proteins. Each MHC class I molecule consists of a heavy chain and a B2 microglobulin

Histology of brain of a man with 1week history of hemiplegia and expressive aphasia...histo staining for lipids, what cells marked most intensely by the lipid stain in picture?

MICROGLIAL...remember the glial scar. Microglial move to the area of ischemic infarct approximately 3-5 days after the onset of ischemia and phagocytize the fragments of neurons, myeline & necrotic debris. [cystic space replaces the necrosis, and astrocytes form a glial scar along the periphery].

Individuals who demonstrate increased activity of a specific intracellular enzymes are more susceptible to developing benz(o)pyrene-induced lung cancer, what enzyme is overactive?

MICROSOMAL MONOOXYGENASE [most cacrinogens enter body in inactive state and are converted to active metabolites via cyt P450 oxidase system

What is the characteristic histological finding in reyes syndrome?

MICROVESICULAR STEATOSIS OF HEPATOCYTES without inflammation & cerebral edema. (reyes = aspirin/nsaid when child has a viral illness like the flu)

Curve showing systemic venous blood, alveolar capillary blood, systemic arterial blood....why is there a tiny dip between alveolar blood and systemic arterial blood?

MICTURE WITH DEOXYGENATED BLOOD ...blood in left artrium/ventricle has lower pO2 because of mixture of oxygenated blood from the pulmonary veins with deoxygenated blood from broncial circulation & thebesian veins

patient loses consciousness/blood flow and MRI shows loss of grey-white matter differentiation with sulcal effacement, damage to what area of the brain is the most likely cause of the patient's pupillary findings?

MIDBRAIN - upper midbrain contains neural structures that mediate the direct and consensual pupillary light reflex.

Question stem mentions baby vomiting very soon after birth but also mentions FIBROUS BANDS EXTENDING FROM CECUM AND RIGHT COLON TO THE RETROPERITONEUM, CAUSING EXTRINSIC COMPRESSION OF THE DUODENUM...what embryological process failed?

MIDGUT ROTATION AROUND THE SUPERIOR MESENTERIC ARTERY - intestinal malroation results when the midgut undergoes incomplete embryological counterclockwise rotation.

Triangle of treatment of thyrotoxicosis is aimed at doing minimizing what 3 things? What class of drugs do two of those things?

MINIMIZING: (1&2 from beta-blockers) 1) PERIPHERAL CONVERSION OF T4 TO MORE ACTIVE T3, 2) SYMPATHETIC OUTFLOW AND/OR ITS ACTIONS ON TARGET ORGANS 3) thyroid hormone synthesis & release

EM picture of cell, question asks about small circular DNA molecules that resemble a bacterial chromosome...further analysis shows these molecules code for proteins, tRNA & rRNA what structure did these DNA molecules likely originate?

MITOCHONDRIA (mtDNA)

1 year old girl with abdominal pain, diarrhea, vomitting & weight loss...imaging shows villous atrophy and crypt hyperplasia? What diet change could fix this?

MODIFIED GRAIN DIET = celiac disease = hypersensitivity to gluten. Gliadin (breakdown product of gluten) causes immune-mediated reaction. Reversible with change in diet.

Pig's right renal artery is clipped down, what cell type is most likely to undergo hyperplasia?

MODIFIED SMOOTH MUSCLE CELLS OF THE AFFERENT ARTERIOLE...chronic renal hypoperfusion can cause hyperplasia of the JGA

Man has immediate-onset severe headache & blurry vision, ate a sandwich with fancy meats and cheeses...he has severe, atypical depression. PE: tremulous & diaphoretic...the medication used to tx patient's depression most likely affects what steps of monoamine transmission?

MONOAMINE DEGRADATION...patient is experiencing a hypertensive emergency and has signs of excessive sympathetic activity from ingesting TYRAMINE-CONTAINING FOODS (aged cheese, cured meats, draft beer). MAO (monoamine oxidase) is a mitochondrial enzyme that degrades excess monoamine neurotransmittersin presynaptic nerve terminals & detoxifies dietary tyramine in the GI

What is essential for the disease process of Shigella toxin?

MUCOSAL INVASION, Shigella invades the GI mucosa particularily via M CELLS THAT OVERLY THE PEYER'S PATCHES

Outbreak of water-borne gastroenteritis in Latin America: oxidase-positive, gram-negative, comma-shaped rods. How would stool microscopy look?

MUCUS & SOME SLOUGHED EPITHELIAL CELLS ~ Vibrio Cholerae (oxidase-positive, gram-negative, comma-shaped organism. Flecks of mucus and sloughed epithelial cells RICE WATER STOOL. (vibrio and e coli can cause a purely toxin-mediated watery diarrhea...modify electrolytes but no cell death)

Infant born with ambigous genitalia...there is a Y chromosome in gonadal cells, absence of the uterus is due to what substance

MULLERIAN INHIBITORY FACTOR (MIF) ~ embryonic testis secretes testosterone and mullerian inhibiting factor(MIF). MIF is responsible for regression of muellerian ducts that normally give rise to internal genitalia in the female fetus

Woman with left eye pain made worse by movement and occasional dimming of vision, episodes seen mostly after shot shower or intense workout, what is the likely diagnosis?

MULTIPLE SCLEROSIS ~ manifests with various self-limiting neurologic symptoms, usually in patients 20-30 years old. The most common initial symptoms are OPTIC NEURITIS, INTERNUCLEAR OPTHALMOPLEGIA & SENSORY DEFICITS. Symptoms usually worsen with heat exposure

What happens to muscle tone in a child with untreated congenital hydrocephalus?

MUSCLE HYPERTONICITY, HYPERREFLEXIA, POOR FEEDING, MACROCEPHALY ~ leads to spasticity due to stretching of the periventricular pyramidal tracts, developmental delays & seizures

What drug is used to tx migraines and epilepsy? Describe its teratogenic effects?

MYELOMENINGOCELE Valproate...valproate is most commonly used for seizure prophylaxis. Valproate is contraindicated in pregnancy leading to NEURAL TUBE DEFECTS - Myelomeningocele

Giving Dobutamine to a patient what is likely to be increased?

MYOCARDIAL OXYGEN CONSUMPTION. Dobutamine (do beta-1): Positive ionotropic effect( increase in cardiac contractility leads to increased cardiac output & decreased LV filling pressures) weak positive chronotropic effect increases HR&contractility but also INCREASES MYOCARDIAL OXYGEN CONSUMPTION

Experiment: ATP is injected into skeletal muscle, during muscle contraction ATP is observed to attach to the sarcome here...what does this cause?

MYOSIN HEAD DETACHMENT FROM THE ACTIN FILAMENT (during contraction cycle, Atp binding to myosin causes release of the myosin head from its binding site on the actin filament)

3-7 days after ischemic stroke what is seen on histology?

Macrophage/microglia ( phagocytosis begins)

Kid gets sick fever nasal discharge, never vaccinated, has several white spots with an erythematous base on his buccal muscosa what would appear next?

Maculopapular rash (measles)

A picture of eosinophils and shown and the question asks what the protein released from these cells does?

Major Basic Protein - Kills Helminths (also contributes to bronchial epithelial damage sustained by patients with atopic/allergic asthma

All antidepressants carry a risk of inducing what?

Mania! (antidepressants can induce mania in all patients especially those with unrecognized bipolar disorder) patients taking antidepressants should be monitored for mood elevation.

Inflammation: Steps of circulating inflammatory leukocytes into the inflamed tissue:

Margination, Rolling, Activation, Tight adhesion, crawling & transmigration

Trisomy 21 child has 3 bands on RFLP, what does this indicate?

Means he has 3 different versions of chromosome 21. If he has 1 band from dad 2 bands from mom, then he got BOTH of mom's homologous chromosomes and the problem occurred in MATERNAL MEIOSIS I.

Spontaneous/Painless bleeding in a previously healthy 4 year old girl, scanned using a 99Tc-Pertechnetate scan

Meckel's diverticulum "currant jelly stools" = FAILED OBLITERATION OF THE VITTELLINE (OMPHALOMESENTERIC) DUCT

What is the cause of a direct hernia?

Medial to inferior epigastric vessels...BREAKDOWN OF THE TRANSVERSALIS FASCIA [indirect is lateral and due to patent processus vaginalis]

What structures does the median nerve course between?

Median nerve (c5-t1): Flexor Digitorum superficialis & flexor digitorum profundus muscles (2nd). (1st) initially courses through ulnar and humeral head of pronator teres. Finally going through the carpal tunnel to enter the wrist

Patient with enlarging thyroid nodule + elevated calcitonin level is concerning for what?

Medullary thyroid cancer (parafollicular C cells)

what are menangiomas? And what happens when they present in the somatosensory cortex?

Meningiomas are common slow growing (benign) that typically rise in areas of dural relfection (falx cerebri, tentorium cerebelli). Lesions involving the primary somatosensory cortex typically result in: contralateral sensory loss (wheras damage to the parietal association cortex (particullary in non-dominant hemisphere) may cause contralateral hemineglect due to impaired visuospatial processing.

Haemophilus influenzae (typing based on presence of a polyribosyl ribitol phosphate(PRP) capsule) if a kid doesn't get full vaccine what is he going to be at risk of developing?

Meningitis!!!!! (also pneumonia, bacteremia, epiglottitis)

What cell layer is the spleen dervired from?

Mesoderm - along with muscle, CT, bone/cartilage, serosa linings (peritoneum), cardiovascular system, blood, lymphatics, spleen, internal genitalia, KIDNEYS & URETERS, adrenal cortex

What are the major dopaminergic pathways of the brain?

Mesolimbic/mesocortical[regulate cognition & behavior], Nigrostriatal[regulates coordination of voluntary movements], Tuberoinfundibular[inhibits prolactin secretion]

Saline-responsive vs. Saline-unresponsive Metabolic Alkalosis:

Metabolic alkalosis (ph > 7.45) can be from vomitting/NG suction & thiazide/loop diuretics (volume & Cl- depletion) [this metabolic alkalosis is saline-responsive]. Hyperaldosteronism leads to metabolic alkalosis that is SALINE-UNRESPONSIVE. Determining patients VOLUME STATUS & measuring urinary chloride concentration can ID source of metabolic alkalosis

What is the most common malignant neoplasm of the adult liver?

Metastatic liver disease. The most common malignant hepatic lesion is a metastasis from another primary site (breast, lung, colon)...not hepatocellular carinoma.

What does methotrexate (MTX) do? And what can reverse MTX toxicity?

Methotrexate is a FOLIC ACID analog that INHIBITS DIHYDROFOLATE REDUCTASE (DHFR) [enzyme normally converts dietary folic acid to tetrahydrofolate]. MTX functions as chemotheraputic to inhibit DNA synthesis. FOLINIC ACID (LEUCOVORIN) can reverse MTX toxicity.

What is an important mechanism of resistance to aminoglycosides?

Methylation of the aminoglycoside-binding portion of the ribosome (which inhibits the ability of aminoglycoside to interfere with protein translation.

What is the Metyrapone test?

Metyrapone stimulation test is a sensitive indicator of hypothalamic-pituitary-adrenal HPA axis integrity. In patients with intact HPA metryaprone will cause REACTIVE INCREASE IN ACTCH, 11-DEOXYCORTISOL & URINARY 17-HYDROXYCORTICOSTEROID LEVELS (in hPA is not intact metyraprone will cause a decrease in cortisol synthesis via inhibition of 11-b-hydroxylase).

Picture of squid axon membrane potential, where on graph is it most permeable to potassium?

Middle of downward slope K+ ion permeance is highest during the repolarization phase of the action potential.

What is a midline epiostomy and what structure below posterior vagina is likely to be cut?

Midline epiostomy further opens up the vaginal opening, cutting from posterior vagina to PERINIAL BODY [transects vaginal lining and submucosal tissue]

Oropharyngeal large, red lingual mass = lingual thyroid...what embryological process is this due to?

Migration...lowest part of evagination forms thyroid gland and the remaining portion form the thyroglossal duct

What parameter best correlates with the potency of an inhaled anesthetic?

Minimal alveolar concentration (MAC) = concentration in the alveoli that renders 50% of patients unresponsive to painful stimuli

What is the most common cause of nephrotic syndrome in children?

Minimal change disease...INCREASED SELECTIVE FILTRATION OF PROTEINS (MCD: systemic t cell dysfunction leads to the production of glomerular permeability factor, which causes podocyte foot process fusion and decreases the anionic properties of the glomerular basement membrane..loss of negative charge leads to selective albuminuria

Where does heme synthesis occur in erythrocytes?

Mitochondia (and partly cytoplasm) of erythrocytes is primarily where heme synthesis occurs. Mitochondria are necessary for the first 3 steps of heme synthesis and in chronic kidney disease you can end up with an anemia due to mature erythrocyte deficiency

What is the relationship between the heart and mitochondrial vacuolization?

Mitochondrial vacoulization is typically a sign of irreversible myocardial injury (involved mitochondria are permanently unable to generate ATP) [glycogen loss is reversible]

An abnormally prominent, upsloping left atrial "v wave" during cardiac catheritization is a major finding in what?

Mitral regurgitation. Patient presented with nonspecific heart failure symptoms including exertional dyspnea and fatigue (caused by inability to increased cardiac output) and orthopnea (due to elevated pulmonary circulation pressure and resulting pulmonary edema)

Blowing, holosytolic murmur best heard over the cardiac apex with radiation to the axilla (rheumatoid knee swelling)

Mitral regurgitation. Patient presented with nonspecific heart failure symptoms including exertional dyspnea and fatigue (caused by inability to increased cardiac output) and orthopnea. Rheumatic heart disease is a very common cause of mitral regurgiation in developing countries.

On cardiac auscultation, an opening snap followed by a diastolic rumbling murmur best heard over the apex of the heart...where is snap on pressure-volume loop?

Mitral stenosis and the snap most likely occurs nearest the bottom of the curve at the beginning of diastole.

What is the most common predisposing condition for native valve infective endocarditis?

Mitral valve prolapse [rheumatic heart disease is #1 in developing nations]

Unvaccinated pregnant woman has low-grade fever and maculopapular rash...over 48 hours rash spreads from face and chest to trunk, +postauricular lymphadenopathy....mom at risk for what, kid at risk for what? What disease?

Mom:Polyarthralgia. Infant:sensorineural deafness. MATERNAL RUBELLA...congenital rubella syndrome: sensorineural deafness, cataracts, PDA

The more common the disease, has what impact on the PPV?

More common the disease, the higher the PPV. The PPV (tp/(tp+fp)) is defined as the proportion of individuals with positive test results who actually have the disease.

How does increased estrogen activity impact levels of thyroid hormone?

More estrogen (pregnancy/therapy) increases THYROXINE-BINDING GLOBULIN = INCREASED TOTAL THYROID HORMONE (T4 pool)...but feedback control maintains normal levels of free (biologically active) thyroid hormone.

Most effective strategy for decreasing adverse outcomes and preventing avoidable readmission is what?

Most effective strategy is using a CHECKLIST (medication changes & follow-up appts) can effectively faciliatate patient's transition

How does autosomal RECESSIVE polycystic kidney disease appear at birth?

Most severe phenotype. Potter sequence seen (flattened facies, limb deformaties, pulmonary hypoplasia...all caused by oligohydramnios) Noticable via enlarged kidneys with diffuse small cysts. Low amniotic fluid. Child will be born in respiratory distress. Mutation is in PHKD1 [fibrocystin]

Facial pain, headache & black necrotic eschar in the nasal cavity + DKA =

Mucormycosis...MUCOSAL BIOPSY and histological exam are necessary to dx. Slides will show: broad-ribbon like nonseptate hyphae with right-angle branching. BLACK NECROTIC ESCHAR in nose is characteristic.

What parts of the intestinal wall are part of Meckel's diverticulum?

Mucosa, Submucosa, Muscular layers (all 3)

Easy fatiguability, constipation, back pain, elevated serum protein, azotemia in an elderly patient should raise suspicion for what?

Multiple Myeloma. Large eosinophilic casts composed of BENCE-JONES proteins are seen in tubular lumen in myloma cast nephropathy

Fungi form broad nonseptate hyphae that branch at right angles...

Murcomycosis (from Rhizopus, Mucor, Absidia species...paransal sinu involvement in IMMUNOCOMPROMISED OR DKA)

What do muscarinic antagonists do to Ach?

Muscarinic antagonists inhibit the POSTSYNPATIC action of ACh

Absent Biceps reflex, where is sensory loss?

Musculocutaneous (C5-7) [larteral cord of brachial plexus] {major forearm flexors & corocobrachialis}

Patient presents with skin anesthesia and hypopigmentation...schwann cells are under attack, what bug?

Mycobacterium leprae...Leprosy (Hansen disease) deforming infection of skin & nerves. Infection in US has been associated with contact with armadillos TT(tuberculoid) self limited(cell-mediated response) Th1. LL(lepromatous) weak CMI (Th2) more severe.

bacteria isolated from the lung of a person fails to decolorize with hydrochloric acid & alcohol after staining carbofuscin...what cell walla component is responsible for that?

Mycolic Acid [acid-fast stain identifies organisms that have mycolic acid present in their cell walls]

Patient has persistent cough & pulmonary infiltrate...blood added to an anticoagulated tube & placed into ice water...several minutes later clumping is detected but it uncoagulates when the tube is warmed...what's up?

Mycoplasma Pneumoniae ...question stem is looking at cold agglutinins. Mycoplasma pneumoniae, EBV & hematologic malignancy are the only illnesses with cold agglutinins (antibodies specific for RBCs that only cause agglutination at cold temperatures).

Military recruits: persisent cough, low-grade fever, malaise bug requires acellular medium enriched with cholesterol to grow (walking pneumonia described)

Mycoplasma Pneumoniae require cholesterol supplementation to grow...question stem is looking at cold agglutinins. Mycoplasma pneumoniae, EBV & hematologic malignancy are the only illnesses with cold agglutinins (antibodies specific for RBCs that only cause agglutination at cold temperatures).

Ehlers-Danlos syndrome is due to defective collagen synthesis, what step is messed up?

N-terminal propeptide removal [PROCOLLAGEN PEPTIDASE DEFICIENCY]

Question stem talking about a bug that requires staph aureus to grow on sheep blood agar, how does staph promote the growth? What does it supplement?

NAD+. Haemophilus influenzae is a "blood-loving" organisms that requires X (hematin) & V (NAD+) factors for growth. H influenzae can grow near the b-hemolytic staph aureus colonies that produce the needed factors X & V

Epistasis in a kid, stopped temporarily by parents pinching nose, stopped permanently by cauderization...what part of nose was cauderized?

NASAL SEPTUM Kiesselbach plexus is the watershed area od the nasal septum

How will Gliobastoma multiforme appear on histology?

NECROSIS AND VASCULAR PROLIFERATION

At the FRC the intrapleural pressure is what?

NEGATIVE, with a value of -5 cm H2O

Picture of a melanoma hx: mom had skin cancer. Question is asking which embryologic derivate its from?

NEURAL CREST. Melanoma is a malignancy of melanocytes which are embryologically derived from neural crest cells. [ABCDE: Asymmetry, BorderIrregular, Colorvaration, Diameter>6mm, Evolving

Man with hundreds of short/sessile/pedunculated bumps on his back presents...what is the disease and inheritance patter?

NEUROFIBROMATOSIS TYPE 1 (NF-1) = SINGLE-GENE AUTOSOMAL-DOMINANT DISORDER...it occurs due to the mutation of NF1 gene on chromosome 17. Café-au-lait spots, multiple neurofibromas and Lisch nodules are the most common symptoms.

Bipolar man is given medications in ED to calm down and the next day he is confused, has high temp and had muscle rigidity what is his current state called?

NEUROLEPTIC MALIGNANT SYNDROME ~ potentially life-threatning adverse reaction to neuroleptics (antipsychotics) = "lead-pipe" rigidity, hyperthermia,autonomic instability & mental status changes.

Man with down's has sharp cognitive decline what is likely to be increased?

NEURONAL AMYLOID PRECURSOR PROTEIN (neurofibrillary tandles and amyloid-beta plaques play an important role in the pathogenesis of Alzheimer disease. Down's patients have 3 copies of the amyloid precursor protein gene which increases amyloid-beta accumulation in the brain = increased risk for early-onset alzheimer

Man with recent-onset oliguria & high serum creatinine level has intranasal ulcer that has failed to heal...patient's condition is associated with antibodies against what?

NEUTROPHILS (nasal mucosal ulceration & glomerulonephritis are characteristic of granulomatosis with polyangiitis (wegner's) c-ANCA (antiNEUTROPHIL)...will target neutrophil proteinase 3

Pathogenesis of centriacinar emphysema is associated with chronic heavy smoking, what cells contribute to this pathology?

NEUTROPHILS...pathogenesis of centriacinar emphysema associated with chronic heavy smoking involves release of PROTEINASES (especially elastase) from infiltrating NEUTROPHILS & alveolar MACROPHAGES

Which type of neurofibromatosis do you get hearing loss?

NF2 (2 ears) chromosome 22, NF2 tumor suppressor gene codes the protein merline. BILATERAL ACOUSTIC NEUROMAS ~ tinnitus, vertigo, hearing loss.

12 year old gait instability and pruitic rash, similar skin rash in the past + loose stools, increased levels of neutral amino acids in the urine...what would help the patient's symptoms?

NIACIN (patient likely has HARTNUP DISEASE AR disorder affecting the neutral amino acid transporter in small intestine & proximal tubule of the kidney) Pellagra-like skin eruptions and cerebellar ataxia which occurs as a result of a NIACIN DEFICIENCY

What are the two most important factors of influencing coronary blood flow?

NITRIC OXIDE & adenosine. NO is the most important mediatory of coronary vascular dilation in large arteries and pre-arteriolar vessels. (NO synthesized from arginine and oxygen by endothelial cells) [adenosine is a product of ATP]

How does PCP work?

NMDA RECEPTOR ANTAGONIST ~ violent agitation and ataxia as well as nystagmus and deliruim (might not remember)

How to calculate number needed to treat?

NNT = 1/ARR [ARR = Control Rate - Treatment Rate]. LOWER NNT = more beneficial treatments

Immunocompromised woman with ring-enhancing focal lesion on brain MRI and surrounding edema in right frontal lobe with fever, headache and branching gram positive stain?

NOCARDIA ASTEROIDES ~ nocardia: pulmonary, CNS, skin disease in immunocompromised; pulmonary nocardiosis can present as cavitary pneumonia (BRANCHING GRAM-POSITIVE ORGANISMS ARE SEEN ON SPUTUM EXAM)

Man with persistent back pain for past several months, tenderness over bone regions what additional findings?

NODULAR PROSTATE & ELEVATED PSA LEVEL (bony pain in older men: osteoblastic bone metastases = prostate cancer, small cell lung cancer, hodgkin lymphoma)

25 year old primigravida has a stillbirth at 18 weeks gestation, her only symptoms during pregnancy were pain in both knees....baby had pleural effusion, pulmonary hypoplasia & ascites. What virus type, enveloped?

NON ENVELOPED, SINGLE-STRANDED DNA VIRUS ~ Parvovirus B19 can cause arthralgias with or without rash in adults and infect fetus may develop HYDROPS FETALIS (severe anemia, heart failure, pleural effusions, pericardial effusions, ascites)

5 year old girl who's grandma passed away 2 months ago talks to an empty chair in her room and mom is concerned, what is the likely diagnosis?

NORMAL BEHAVIOR ~ Hallucinations of recently deceased relatives in children are part of a normal grief reaction and may not be indicative of major psychiatric illness

45 yo man comes in with severe chest pain, diaphoresis, palpitations...patient dies 2 hours after onset of symptoms, autopsy shows 100% occlusion of LAD how will affect myocardium look on LM right after death?

NORMAL MYOCARDIUM ~after the onset of severe ischemia leading to MI, early signs of coagulative necrosis do not become apparent on LM until 4 hours after the onset of MI

Man with severe right flank pain that radiates to the groin, has gross hematuria...imaging shows a stone in the middle of the right ureter, what will his lab values likely show?

NORMOCALCEMIA, HYPERCALCIURIA...idiopathic hypercalciuria is the most common cause of calcium kidney stone disease. This condition is characterized by normal serum calcium levels with hypercalciuria.

What is the best method to determine if a gene is being transcribed in a culture of isolated cells?

NORTHERN BLOT = detects target mRNA in a sample to assess gene expression

What does the NPV represent?

NPV represents the probability of not having a disease, given a negative test result. NPV is based on DISEASE PREVALENCE

What two types of DMARDs can provide rapid symptom relief in RA?

NSAIDs and GLUCOCORTICOIDS (like PREDNISONE)

What is most likely responsible for development of colon cancer in a woman with a genetic analysis positive for Lynch Syndrome (hereditary nonpolyposis colon cancer)?

NUCLEOTIDE MISMATCHES THAT ESCAPE REPAIR ...lynch syndrome is an autosomal dominant disease caused by abnormal MISMATCH REPAIR. MSH2 & MLH1

During the myocyte contraction calcium efflux from cardiac cells prior to relaxation if primarily mediated by what?

Na+/Ca2+ exchange pump and sarcoplasmic reticulum Ca2+-ATPase pump. Intracellular calcium plays an important role in excitation-contraction coupling.

What is narcolepsy and what causes it?

Narcolepsy = chronic sleep disorder characterized by frequent overwhelming urges to sleep ...it results from depletion of hypocretin-secreting neurons in the lateral hypothalamus that are involved in maintaining wakefullness. +REM sleep related phenomena (hallucinations)

Where are meningiococci normally isolated from?

Nasopharynx & Oropharynx of asymptomatic carriers. The meningococcal fimbriae/pilli are important for bacterial attachment to epithelial surfaces. (if you can impair pili attachment to naso/oropharynx you can successfully make a vaccine)

What cells are responsible for the destruction of cells with decreased or absent MHC class 1 proteins on their surface?

Natural Killer Cells (NK) NK cells express CD 16 or CD 56. NK cells induce apoptosis and contain perforins and granzymes in cytoplasmic granules.

Diabetics often need two types of insulin, why?

Need a basal long actin insulin (NPH-twice daily; Glargine/Detemir-once daily) and a postprandial short-acting insulin (regular-peak2/5 hour; Lispro/Aspart/Glulsine)

14 year old intermittent shortness of breath, FEV1/FVC ratio is 83% what is most useful in excluding asthma?

Negative Methacholine challenge

What bug is associated with Waterhouse Friderichsen syndrome (adrenal gland destruction, DIC & shock)

Neiserria Meningitidis (also more commonly causes MENINGITIS)

What culture is used for Neiserria Species?

Neiserria can be isolated using a SELECTIVE culture called Thayer-Martin VCN (vancomycin/colistin/nystatin) inhibits growth of other contaminants

Urethritis in a young man, what two possible bugs? What is standard tx?

Neisseria Gonorrhea & Chlymidia Trachomatis ...severe PID can result in infertility(woman), MALE & FEMALE treatment must always cover both organisms with a 3rd gen cephalosporin (ceftriaxone-gonzo) + azithromycin or doxycycline(chlymidia)

What two bugs significantly increase the chances of ectopic pregnancy?

Neisseria Gonorrhea & Chlymidia Trachomatis ...severe PID can result in infertility(woman), MALE & FEMALE treatment must always cover both organisms with a 3rd gen cephalosporin (ceftriaxone-gonzo) + azithromycin or doxycycline(chlymidia)

What two things are the most common causes of Pelvic Inflammatory Disease? What can happen when not fully treated?

Neisseria Gonorrhea & Chlymidia Trachomatis ...severe PID can result in infertility, treatment must always cover both organisms with a 3rd gen cephalosporin (ceftriaxone) + azithromycin or doxycycline

Where are the cells in carcinoid tumor derived from?

Neuroendocrine cells

30yo female with new seizures has an intracranial mass detected on head CT scan, biopsy of mass reveals positive stain for SYNAPTOPHYSIN (negative for glial fibrillary acidic protein) the neoplastic cells likely originated from same precursor tissue as what?

Neurons! Synaptophysin is a transmembrane glycoprotein found in the preysnaptic vesicles of neurons, neuroectodermal, & neuroendocrine cells. CNS tumors of neuronal origin often stain positively for synaptophysin on immunohistology. [glial neoplasms will stain for GFAP]

24-72 hrs after ischemic stroke what is seen on histology?

Neutrophillic Infiltration

HIV medicine that is NOT phosphorylated intracellularly and acts in cytoplasm to inhibit synthesis of viral DNA from RNA template?

Nevirapine/efavirenz ~ NNRTIs

Why are newer generation antihistamines ok for elderly patients?

New gen antihistimines (Loratidine, cetirizidine) have reduced side-efx and are safer in elderly patients (less lipophilic and do not readily cross BBB and are usually non-sedating)

Patient prescribed appropriate medication for elevated triglycerides and doctor warns the patient may feel some "skin flushing" or "warmth" what agent mediates the side effect?

Niacin (Vitamin B3) used to tx Hyperlipidemia, Prostaglandins mediate cutaneous flushing caused by Niacin. Prostaglandin release! This will increase renal blood flow and increase GFR & drug delivery. However, using this with NSAIDs can result in decreased diuretic response.

What does Niacin help with? What can be an adverse side effect?

Niacin decreases hepatic synthesis of triglycerides & VLDL and reduces clearance of HDL...however Niacin can also decrease renal excretion of uric acid (precipitating gouty arthritis)

What disease is common to the Ashkenazi Jew family and how does it present clinically?

Niemann-Pick Disease...sphingomyelinase deficiency = NEURODEGENERATION & HEPATOSPLENOMEGALY

Man looking for drug that will lower blood pressure without worstening ECG?

Nifedipine (dihydropyridines: nifedipine, amlodipine, felodipine) primarily affect arterial smooth muscle causing vasodilation with little or no effect on cardiac conduction or contractility.

Where does ADH act in kidney and what is its purpose?

No ADH = DILUTE URINE. ADH acts on medullary segment of collecting duct to increase urea & water reabsorption, allowing for production of maximally concentrated urine. Acts on V1(vasoconstriction & increased prostaglandin release) & V2(antidiuretic response)

Are physicians required to provide medical services against their personal beliefs? What happens in these cases?

No...but physician is required to refer patient to another physician who will.

Sickle cell, what AA change what causes sickling?

Nonpolar AA Valine replaces Charged AA Glutamate at position 6 of the Beta Globulin chain. The sickling is caused by OXYGEN UNLOADING [CONDITIONS ASSOCIATED WITH LOW OXYGEN = low pH; high 2,3BPG]

Difference between normal greif and major depressive episode?

Normal greif there is no suicidality, worthlessness or pervasive anhedonia

What is the most common cause of viral gastroenteritis?

Norovirus [diarrhea, vomtting, fever, malaise & headache] ...watery diarrhea without blood or mucous. Fecal-oral spread.

What is the primary site of ribosome synthesis and assembly? (on electron microscopy)

Nucleolous! (will appear black and inside of nucleus)

Man gets renal transplant and 4 years later the patient develops hypertension labs show increased creatinine and urinalysis is within normal limits. Kidney is reduced in size on ultrasound. Biopsy of the graft is most likely to show:

OBLITERATIVE VASCULAR FIBROSIS ~ chronic renal allograft rejection manifests months to years after transplantation and presents with worsening hypertension and a slowly progressive rise in serum creatinine. It is mediated by a chronic, indirect immune response against donor alloantigens and results in obliterative intimal thickening, tubular atrophy & interstitial fibrosis

3 year old brought to ED with fever and stiff neck ~ bacterial meningitis, dr starts IV antibiotics and on the second day of tx, the mom wants to take kid home and give baby homeopathic meds...dr explains risk of lethality but mother is unconvinced, what is the most appropriate course of action?

OBTAIN A COURT INJUNCTION TO MANDATE CONTINUED TREATMENT ~ parent's authority to make medical decisions for their children can be challenged in cases where a kid is at risk for harm. Physicians are justified in obtaining a court injunction to proceed with life-saving tx

If a deaf patient can speak via writing or sign language what is best?

OBTAIN ASSISTANCE FROM A QUALIFIED SIGN LANGUAGE INTERPRETER

In case-control what kind of measurement can you use?

ODDS RATIO ~ Case-control study is sued to compares exposure of cases to exposures of controls. ODDS RATIO

What is the formula for odds ratio?

OF = ad/bc

Woman with diplopia, complains of clumsiness of left hand and numbness of right foot....she had numbness in right hand 6 months ago, resolved without treatment...what will Mri show?

OLIGODENDROCYTE DEPLETION ~ suspect MS in a patient with neurological deficits that cannot be explained by a single lesion. MS is an autoimmune disease. Within the plaques, loss of myelin sheaths and depletion of oligodendrocytes is seen. Oligoclonal bands of IgG may be detected in CSF.

Immigrant has 3 month productive cough, night sweats, low-grade fever...sputum shows budding yeast that form germ tubes,the most likely site of this organism before entering the sputum is what?

ORAL CAVITY ~ candida albicans is a normal inhabitant of the GI tract (including oral cavity) in up to 40% of the population. It is a common contaminant of sputum culutres. Presence of Candida does NOT indicate disease.

2 year old fever, vomit, sleeepiness. Multiple illness similar since birth, increased blood ammonia levels and increased orotic acid excretion in urine...tahcypneic and unresponsive to stimuli...what is likely deficient?

ORNITHINE TRANSCARBAMYLASE deficiency is the most common disorder in the urea cycle. HYPERAMMONEMIA & ELEVATED URINARY OROTIC ACID

23 year old white male reccurent sever nose bleeds, pink spider-like lesions on his oral & nasal mucosa face and arms

OSLER-WEBER-RENDU syndrome (hereditary hemorrhagic telangiectasia) autosomal dominant condition marked by presence of telangiectasias in skin and mucous membranes.

What is a feared complication of correciton of rapid chronic hyponatremia?

OSMOTIC DEMYELINATION SYNDROME ~ rapid correction of chronic hyponatremia may lead to osmotic demyelination of the axons in the central part of the pons. This condition is called central pontine myelinolysis [spastic quadriplegia & pseudobulbar palsy]

Picture of a hand wants to you decide between osteoarthritis and rheumatoid arthritis (there is 10-15 minute morning stillness and the joint pain is activity-related)

OSTEOPOROSIS ~ characterized by osteophyte formation leading to hard bony enlargement of the distal interphalangeal joints (Heberden nodes) and proximal interphalangeal joints (Bouchard nodes).

Patient dies from a pneumonia and on gram staining there is a lancet-shaped gram-positive diplococci, death could have been prevented by a vaccine of what type?

OUTER POLYSACCHARIDE COVERING

Research study where pathologists who knew the status of research subjects were 3 times likley to label results "diabetic neuropathy" what kind of bias?

Observer bias - blinding prevents patients/researchers from interfering with an outcome.

Research study, neither the patients or physicians are aware who takes drug & who takes placebo...what bias is this study best at preventing?

Observer bias - blinding prevents patients/researchers from interfering with an outcome.

What is the mf'in Hawthorne Effect?

Observer effect = tendency for a subject to change their behavior when they know they are being watched

Which drug used in the stomach INHIBITS PRIMARY ACTIVE TRANSPORT?

Omeprazole, parietal cells release hydrogen ions into the gastric lumen via the H+/K-ATPase, which requires hydrolysis of ATP and is therefore an ACTIVE TRANSPORT MECHANISM. Omeprazole and other PPIs suppress the activity of gastric parietal cell H+/K-ATPase leading to an increase in the pH of the gastric lumen

Where do ACE inhibitors work?

On the EFFERENT ARTERIOLE. ACE inhibitors dilate the EFFERENT arteriole decreasing glomerular pressure and filtration rate

Woman keeps vomitting what specific serotonin blocker could be used

Ondansetron = serotonin 5HT-3 antagonist tx nausea and vomitting following chemotherapy.

What is clozapine used to treat? What should be monitored while a patient is taking Clozapine?

Only antipsychotic that has shown consistent efficacy in tx of treatment resistant SCHIZOPHRENIA. Neutropenia and agranulocytosis are the two things to watch for via ABSOLUTE NEUTROPHIL COUNT. Due to risk of life threatening agranulocytosis.

What are the ONLY substances that have shown clear therapeutic benefit for pharmagologic receptor antagonists in Allergic (atpoic) asthma?

Only leukotrienes, LTC4, LTD4, LTE4 & acetylcholine have pharmacologic receptor antagonists that offer clear therapeutic benefit

Majority of overdose deaths in the united states are relatd to what drugs?

Opiods...abuse/addiction to opiods including both prescription pain medications and heroin is a growing problem

Contralateral homonymous hemianopia, where is the lesion?

Optic tract. (also there will be an afferent pupillary defect [marcus gunn pupil] in the pupil contralateral to the tract lesion.

Genetic shift (reassortment) involves exchange of genomic segments...what two viruses can do this?

Orthomyxovirus & Rotavirus are capable of genetic shifts through reassortment. They are both SEGMENTED.

Osteoblastic, Mixed, Osteolytic bone metastases?

Osteoblastic(prostate cancer, small cell lung cancer, hodgkin lymphoma). Osteolytic(multiple myeloma, non-small cell lung cancer, non-hodgkin lymphoma, renal cell carcinoma, melanoma)

Woman with 10 pound weight loss, hypothyroidism muscle weakness gradually declining with new heliotrope rash in peri-orbital area and cheeks +Gottron's papules...what is likely to be seen?

Ovarian Adenocarcinoma ...dermatomyositis is an autoimmune inflammatory disease affecting muscles and skin. Has strong association with malignancy (particularily lung colorectal and ovarian)

What can happen when MANNITOL is used in the management of cerebral edema & increased intracranial pressure

Overaggressive tx with osmotic diuretics can lead to excessive volume depletion and eventual hypernatremia in certain patients.

Topical preparations of alpha-adrenergic agonists cause vasocontriction of nasal mucosa vessels, what can happen with overuse?

Overuse of topic a-adrenergic agonists causes negative feedback, resulting in decreased norepinephrine synthesis and release from nerve endings, which diminishes their effect.

What is the most important UROPATHOGENIC virulence factor for the #1 cause of UTIs?

P FIMBRIAE....E COLI (without P fimbriae, E coli would not be able to bind to uroepithelial cells and infect the bladder, ureters and kidneys...it would just get peed out

hardy weinberg: allele frequency =

P(normal) + Q(mutant frequency) = 1

What is the ultimate fate of C-peptide?

PACKED IN SECRETORY GRANULES until they are secreted in equimolar amounts

Man is having frequent accidents bumping into people and things and past medical history is significant for renal stones, physical exam shows bitemporal visual field defects. In addition to brain imaging, the patient should also be screened for what kind of tumor?

PANCREATIC ~ MEN1 ~ multiple endocrine neoplasia type 1 (3 p's: parathyroid gland, pituiatry, pancreas) usually presents with asymptomatic hypercalcemia or renal stones.

Bloody nipple discharge in a patient with no mass or metastasis?

PAPILLARY CELLS WITH FIBROVASCULAR CORE. Intraductual papilloma is a proliferation of papillary cells in a cyst wall or duct that may contain focal atypia (#1 bloody nipple discharge)

Fine-needle aspiration shows large cells with nuclei containing finely dispersed chromatin with a ground-glass appearance in the thyroid?

PAPILLARY THYROID CANCER

20 year old man has right-sided testicular swelling, ith no pain or trauma...ultrasound shows solid testicular mass, if patient's tumor spreads where is it likely to go?

PARA-AORTIC lymph from the testes drains through lymph channels directly back to the paraaortic lymph nodes. [scrotum goes to superficial inguinal lymph nodes]

Infant born with facial dysmorphia & cleft palate, right to left shunt, sinopulmonary infections...examination of lymph nodes will show poor what?

PARACORTEX REGION. This is DiGeorge syndrome (maldevelopment of the 3rd and 4th pharyngeal pouch derivatives) EXTREME deficiency in number of T cells leading to poor development of the PARACORTEX

18 month old with fever, runny nose, sore throat...patient says go home and drink plenty fluids. Two days later ED with fever, brassy cough & difficulty breathing. PE:Stridor. What pathogen is causing the condition?

PARAMYXOVIRUS ~ brassy, barking cough; dyspnea; and recent history of upper respiratory infection in a child are suggestive of viral laryngotracheitis (croup). The most common cause of croup is parainfluenza virus.

BPH, how are the patients kidneys going to react?

PARENCHYMAL PRESSURE ATROPHY (urinary retenteion results in increased pressure in the urinary tract system & resultant reflux nephropathy)

Patient presents with vaginal bleeding, prior miscarriage and there is fetal tissue with some edematous villi with focal trophoblastic proliferation & normal appearing villi...what's up

PARTIAL MOLE ~ begnign, focally hyperplastic trophoblasts, focally enlarged/hydropic villi, fetal/embryonic tissue present. [difference between partial and complete...PARTIAL = FETAL TISSUE PRESENT]

Patient develops a SEVERE ANEMIA following a minor febrile illness, what virus and is it a DNA or RNA virus?

PARVOVIRUS B19 NONENVELOPED SINGLE-STAND DNA VIRUS = MOST COMMON VIRAL CAUSE OF APLASTIC CRISIS

Patient is salty doctor is running late, when given forms regarding his history the patient only signs his name and says to dr "you're smart figure it out" when asked if everything is okay patient says "oh nothing, I'm fine" defense mechanism?

PASSIVE AGGRESSION: avoding conflict by expressing hostility covertly

6 year old immigrant difficulty breathing, head and neck exam reveals neck swelling, palatal paralysis, gray pharyngeal exudate. What (specific) intervention is most likely to improve patient's prognosis?

PASSIVE IMMUNIZATION The diptheria antitoxin(passive) in activates all circulating toxin. TX: Diptheria Antitoxin(inactive),Antibiotics, DPT(active) to prevent future diptheria [corynebacterium diptheriae]

What is the biggest requirement for referring a patient to hospice care?

PATIENT HAS A PROGNOSIS OF LESS THAN 6 MONTHS (doesn't even have to have decision making capacity)

Describe murmur associated with patent ductus arteriosus

PDA is characterized by a continuous murmur best heard in the left infraclavicular reion with maximal intensity at S2

what is the mechanism of action of Milrinone?

PDE-3 inhibitor = VASODILATION...increased intracellular cAMP which promotes intracellular calcium influx & increases cardiac contractility

Dermatitis, dementia & Diarrhea...what condition/deficiency and what is a precursor to the thing that is deficient?

PELLAGRA ~ DERMATITIS, DEMENTIA, DIARRHEA ~ NIACIN DEFICIENCY B3 can be synthasized endogenously from tryptophan and is an essential component of NAD and NAPD

Man in hospital, post-MI is now experiencing SHARP chest pain that increases with swallowing and coughing and radiates to his neck. Why?

PERICARDIAL INFLAMMATION OVERLYING THE NECROTIC SEGMENT OF MYOCARDIUM...PERICARDITIS IS SHARP & PLEURITIC (can sometimes be relieved by leaning forward)

Kid ODs on mushrooms the mushrooms stimulate muscarinic receptors what is the direct effect of the poison?

PERIPHERAL VASODILATION activation of muscarininc receptors by acetylcholine or cholinergic agonists results in peripheral vasodilation due to synthesis of NO in endothelial cells,which leads to vascular smooth muscle relaxation (hypotension)

Patient is suffering from hypotonia/seizures...has an inability to OXIDIZE very long chain fatty acids (VLCFA) & phytanic acid...defect in what cellular comparement?

PEROXISOMES ~ peroxisomal diseases are rare inborn errors of metabolism where peroxisomes are either absent or nonfunctional. VLCFA or Fas with branch points at odd-numbered carbons cannot undergo mitochondrial beta-oxisdation so they are metabolized by a special form of beta or alpha oxidation within PEROXISOMES

3 week old boy presents with discharge from his umbilicus, small reducible umbilical hernia, minimal clear-to-straw discharge from umbilicus, what's up?

PERSISTENCE OF ALLANTOIS REMNANT...patients present with straw colored urine discharge [urachus is a remnant of the allantois]

Man is dysarthric with a right lower faical droop & ride-sided hemiparesis...MRI reveals acute subcortical infact in the left internal capsule, how will the findings of the pyramidal tract look in 6 months?

PERSISTENT MYELIN DEBRIS ~ WALLERIAN DEGENERATION IN CNS = process that occurs when an axon is damaged, resulting in aconal deegeneration and breakdown of thr myelin sheath DISTAL to the site of injury. No regeneration due to persistence of myelin debris & glial scarring

Diptheria is a toxin-mediated disease but not all strains of C. Diptheria express disease causing exotoxin, what makes them turn into disease causing?

PHAGE CONVERSION PERMITTING EXOTOXIN PRODUCTION. Acquring the TOX gene via lysogenization

Lung biopsy reveals fine carbon particals in patient's respiratory bronchioles and alveolar ducts, what defense mechanism is responsible for clearing particles from this portion of respiratory tract?

PHAGOCYTOSIS...patient had coal work pneumonconiosis; engulfment of inorganic dust causes macrophage activation and release of cytokines inducing pulmonary inflammation.

Gram stain of CSF of a army recruit with fever & headache has BEAN-SHAPED GRAM-NEGATIVE cocci in pairs, what is the route of contamination?

PHARYNX -> BLOOD -> CHOROID PLEXUS -> MENINGES. Neisseria Meningitidis gains access to CNS by colonizing nasophraynx and subsequently invading the mucosal epithelium and gaining access to the bloodstream. Through the blood, it spreads to the choroid plexus, gains access to the CNS through the blood-brain barrier and initiates an inflammatory process

What is it called when there is coinfection of a host by 2 viral strains resulting in progeny virions that contain nucleocapsid proteins from 1 strain and unchanged genome of the other strain?

PHENOTYPE MIXING ~ question might reference influenza/sialic acid and pigs

Medication prescibed tha: INCREASES PERIPHERAL VASCULAR RESISTANCE, INCREASES SYSTOLIC BLOOD PRESSURE, DECREASES PULSE PRESSURE & DECREASES HEART RATE

PHENYLEPHRINE ~ selective alpha-1 adrenergiv receptor agonist that increases peripheral vascular resistance and systolic blood pressure and decreases pulse pressure and ehart rate

decreased activity of what enzyme is most likely responsible for decreased production of epinephrine?

PHENYLETHANOLAMINE-N-METHYLTRANSFERASE

Image of gingival hyperplasia is shown, what drug is responsible for the patient's teeth and gums?

PHENYTOIN ~ GINGIVAL HYPERPLASIA is a common side effect of phenytoin and is sometimes reversible when phenytoin is withdrawn. Phenytoin causes increased expression of platelet-derived growth factor (PDGF).

Man gets infection (gas, tissue necrosis + gram positive rods) what is the mechanism of action of toxin responsible/

PHOSPHOLIPID SPLITTING ~ LECITHINASE(ALPHA TOXIN) is the main toxin produced by CLOSTRIDIUM PERFRINGENS. Functions to degrade lecithin, part of cell membrane and leads to membrane destruction cell death and widespread necrosis & hemolysis

Doc#1 finishes shift and does handoff to Doc#2(who is coming off a shift from a different job) explaining that there is a patient who needs to be checked on and have his anion gap calculated; doc#2 forgets to do that...what is the cause?

PHYSICIAN FATIGUE...sleep deprivation in physicians often causes cognitive impairment, resulting in medical errors. Although mandated resident work-hour limitations are in place, it is the responsibility of all physicians to self-regulate their workloads to promote patient safety.

Glycogen, lipid, protein synthesis follow what pathway (thanks to initial binding of insulin)

PI3K stimulates translocation of GLUT-4 to membrane, GLYCOGEN SYNTHESIS & fat synthesis.

What is another name for PICA (posterior inferior cerebellar artery) inclusion? Where does PICA branch from?

PICA occlusion (Wallenburg syndrome) causes: vertigo/nystagmus, ipsilateral cerebellar signs, loss of pain/temperature sensation in ipsilateral face & contralateral body, bulbar weakness & ipsilateral horner syndrome. Can be seen in cervial spinal trauma (with dissection of vertebral artery).

Man with rash of two months (hyperpigmented/hypopigmented macule/patches on upper body) what is the rash called and what fungi causes it?

PITYRIASIS VERSICOLOR/TINEA VERSICOLOR ~ caused by malassezia species. Forms spores & hyphae producing "spaghetti & meatballs" appearance on KOH prep LM

Woman with previous uncomplicated pregnancy (via c-section) delivers a baby and profuse vaginal bleeding occurs, the bleeding continues respite uterine massage and administration of uterotonic medicines....why?

PLACENTA ACCRETA - postpartum hemorrhage is an obstetric emergency and can result from abnormal placentation. Prior uterine surgery can impair decidualization, resulting in myometrial invasion by villous tissue and a placenta that is abnormally adherent to the myometrium (placenta accreta)

Kid born with ambiguous genitalia & clitoromegaly...lab studies show elevated testosterone & androstendione [46,XX] normal size uterus, mom experienced facial hair growth & voice deepening during pregnancy, what enzyme?

PLACENTAL AROMATASE deficiency. Aromatase converts androgens into estrogens in: ovaries, testes, placenta, other peripheral tissues

Genetics Explanation? 10 year old new swelling in right leg(caved in chest, and intellectual diasbilit, missense mutation in gene coding for cystathionine beta-synthase enzyme) what is the most likely explanation?

PLEIOTROPY = describes instances where multiple phenotypic manifestations result from a single genetic mutation. Most SYNDROMIC genetic illness exhibit PLEIOTROPY

Woman with pain during sex and urination has multiple painful shallow ulcers with an erythematous base on the left labie...bilateral tender inguinal lymphadenopathy....what is best diagnostic test?

POLYMERASE CHAIN REACTION FOR VIRAL DNA she has genital herpes (could also use a tzank smear or direct flourescence antibody testing)

Study done showed people with lower levels of Tamoxifen had higher chance of relapse...what could cause the drug's ineffectiveness in only some of the patients?

POLYMORPHISM OF CYTOCHROME P450 ENZYME ~ Cytochrome P450 enzymes found in the liver are responsible for the majority of drug metabolism. Polymorphisms occuring in the genes coding for these enzymes result in various phenotypes that differ in their rates of metabolism; individua differences in phenotype alter treatment efficacy and drug toxicity

Question stem describes gram-positive diplococci with small alpha-hemolytic colonies and lung consolidation in the left upper lobe. What is the major virulence factor?

POLYSACCHARIDE CAPSULE ~ the primary virulence factor of Streptococcus pneumonia (without it, it cannot cause disease) is a polysaccharide capsule that inhibits phagocytosis.

What is the relationship between Beta-Endorphins & ACTH & MSH?

POMC! Beta-endorphins and ACTH & MSH are all derived from the POMC (proopiomelanocortin) a polypeptide precursor that goes through enzymatic cleavage and modification to produce B-endorphins, ACTH, MSH.

Woman in ED with acute-onset abdominal pain, nausea & confusion. Has no past medical hx, sample of urine is reddish in color and IV dextrose is given and her symptoms improve significantly. Dextrose infusion is improved by affecting what pathway?

PORPHYRIN SYNTHESIS~ acute intermittent porphyria is an autosomal dominant condition caused by PORPHOBILINOGEN DEAMINASE DEFICIENCY. Most patients remain asymptomatic, but a minority present with acute attacks. TX: IV glucose/heme to downregulate ALA synthase activity

Cross section of abdomen/liver is shown and patient has alcoholic cirrhosis and image is asking what will have high pressure before tx?

PORTAL VEIN (arrow closest to the liver that appears to be going inside of the liver) lays medial or just within the right lobe of the liver and anterior to the IVC

Woman has significant sleep distburbance, 3 months ago got raped and now she has nightmares and feels like she is back in the parking lot. Dx?

POST-TRAUMATIC STRESS DISORDER

Cross section of leg, kid was hurt in soccer game and now tibia moves posteriorly identify on image where the lesion occurred

POSTERIOR CRUCIATE LIGAMENT

Question stem sounds like hemorrhoids but says there is an actual rectal tear on examination and asks where it is?

POSTERIOR MIDLINE DISTAL TO THE DENTATE LINE = anal fissue...POSTERIOR MIDLINE, likely due to decreased blood flow to the area

Old man with difficulty swallowing develops pneumonia, what lung region is likely to be affected

POSTERIOR SEGMENT OF THE RIGHT UPPER LOBE ~ due to gravity, supine patients typically aspirate into the posterior segments of the upper lobes and superior segments of the lower lobes.

Little kid with hematuria, periorbital edema and hypertension...rbcs casts and elevated serum creatinine?

POSTSTREPTOCOCCAL GLOMERULONPEHRITIS

What is given prophylactically in the event of a iodine-131 nuclear release/outbreak?

POTASSIUM IODIDE = competitively inhibits thyroid uptake of radioactive iodine isotopes and is often administered following nuclear accidents to protect the thyroid and reduce development of radiation-induced thyroid carcinoma

What epi/bio formulas are influenced by disease prevalence?

PPV & NPV

What represents the probability of truly having a disease given a positive test result? How does it change based on prevalence?

PPV = (a)/(a+b) = the probability of truly having a disease given a positive result. It increases with increasing disease prevalence and decreases with decreasing disease prevalence.

What is plama renin activity (PRA) a good measure of? If a patients PRA does not increase after being prescribed valsartan and hydrocholorthiazide, whats uP?

PRA (plasma renin activity) is a good measure of amount of angiontensin I generated per unit of time and is useful in assessing function of RAAS axis. No change in PRA after being prescibed valsartan and hydrocholorothiazide = noncompliance of medicine

35 year old man comes to ED with coffee-ground emesis and lightheadedness. Had back pain and was started on naproxen, he has HIV/Hep C/ he's allergic to penecillin....UP Gi endoscopy shows gastric mucosal erythema and erosions what best explains the patient's current symptoms?

PREDICTABLE DRUG RXN ~ nsaids cause Gi bleeding

Man ejacts too quickly during sex, has type 1 diabetes and takes medicine for that...has no problem ejecting fast during masturbation....dx?

PREMATURE EJACULATION...recurrent episodes of early ejaculation accompanied by sense of lack of control. (gotta check meds/stressors)

Man with upper abdominal pain relieved by food, endoscopy shows a small ulcer with a clean base in the duodenal bulb, what site is likely going to demonstrate the infectious agent responsible?

PREPYLORIC AREA ~ h pylori can cause duodenal ulcers and is typically found in greatest concentration in the PREPYLORIC AREA of the gastric antrum.

Man experiences fever and muscle rigidty soon after major surgical procedure +cyanotic skin mottling, what is MOA of appropriate tx?

PREVENTION OF INTRACELLULAR CALCIUM RELEASE IN SKELETAL MUSCLE = DANTROLENE (patient is experiencing malignant hyperthermia. Dantrolene blocks the ryanodine receptors and prevents release of Ca into the cytoplasm of skeletal muscle fibers

What is the single strongest predictive factor of commiting suicide?

PREVIOUS SUICIDE ATTEMPT

Man with severe fatigue, hepatomegaly, splenomegaly...has gain of function mutation in non-receptor tyrosine kinase protein leading to persistent activation of signal transducers and activators of transcription STAT proteins...what disorder?

PRIMARY MYELOFIBROSIS ~ chronic myeloproliferative disorders often have a JAK2 mutation.

What does PCR specifically need in order to amplify small fragments of DNA?

PRIMERS THAT ARE COMPLEMENTARY TO THE REGIONS OF DNA FLANKING THE SEGMENTS OF INTEREST

Man takes medicine for CAD, CHF and cardiac rhythm abnormalities and now how positive anti-ANA and joint pains/fevers....what should the patient be questions about (what drug)??

PROCAINAMINDE...procainaminde and hydralazine have the highest risk of causing drug-induced lupus [positive ANA/histone antibodies...negative ds-DNA]

Patient presents with substernal chest pain & ST elevation ...a few days later dies suddenly, what is the likely reason?

PROFOUND HYPOTENSION...patient initially likely suffered ruptured left ventricular free wall can lead to cardiac tamponade and death due to hypotension.

Woman has found out giving birth would probably kill her and the baby, decides to terminate pregnancy using mifepristone & misoprostol regimen...what mechanism of action?

PROGESTERONE ANTAGONIST...progesterone is necessary for implantation and maintenance of pregnancy. Mifepristone (progesterone agonist) + misoprostol (prostaglandin E1 agonist) used to terminate 1st trimester pregnancy.

14 year old boy brought in by his parents, due to his actions since they are getting a divorce...the patient has been quiet and avoids parents because he can "sense" that they are mad at him even though they arent...what psych pathology?

PROJECTION = immature defense mechanism misattributing one's own feelings to another person who does not actually have them

32 year old man with visual problems had several near misses and has headaches but cranial nerves are intact, what hormone is likely to be elevated?

PROLACTIN~ pituitary tumors can present with headaches, bitemporal hemianopsia, hypopituitarism. Additional physiologic effects depend on secretion of specific hormones. (prolactin in women =galactorrrhea/amenorrhea. Men = hypogonadism)

How does the carotid sinus massage work?

PROLONGED ATRIOVENTRICULAR NODE REFRACTORY PERIOD. Leads to an increase in parasympathetic tone causing temporary inhibition of SA node activity, slowing of conduction through the AV node & prolongation of the AV node refractory period.

What are the TATA and CAAT boxes essential for

PROMOTERS OF TRANSCRIPTION in eukaryotic cells [25-75 base pairs upstream from transcription start site]. They promotoe initiation of transcription by serving as binding sites for transcription factors & RNA pol ii

What (specifically) does the romberg test, test for?

PROPRIOCEPTION ~ the romberg test is a test of proprioception in which patients are observed for unsteadiness as they stand with their feet close together, arms to sides and eyes closed. (failure = ataxia possibly due to defects in posterior column or peripheral nerves)

400 women, check-up regarding smoking status....over next ten years some smokes get breast cancer some do not. Study design type?

PROSPECTIVE COHORT STUDY - prospective cohort studies are organized by selecting a group of individuals (cohort) determining their exposure status and then following them

12 day old has feeding intolerance that worsened over past 2 days, was born at 27 weeks, only intubated for 2 days. Well-appearing/viogorous. Continuous murmur with systolic accentuation is best heard at the left heart border. Production of what is is causing the infant's condition?

PROSTAGLANDIN ~ PDA...patent due to prostaglandin E2 production by the placenta. Continous murmur (left to right shunting) Cardiovascular strain. INDOMETHACIN OR IBUPROFEN therapy can inhibit prostaglandin E2 synthesis and acelerate closure

What is responsible for the intracellular effects of G protein mediated adenylate cyclase second messenger system?

PROTEIN KINASE A

If a drug (phenylephrine) were administered that blocks IP3, what enzyme would likely be decreased?

PROTEIN KINASE C ~ after a ligang binds to a GPCR that activates phospholipase C, membrane phospholipids are broken down into DAG & IP3. PROTEIN KINASE C is subsequently activated by DAG & calcium; calcium is released from ER under the influence of IP3

What is the major virulence factor for Group A Strep?

PROTEIN M is the major virulence factor for streptococcus pyogenes...it inhibits the activation of complement, mediates bacterial adherence, and is the target of type-specific humoral immunity to S. pyogenes

What should be monitored to see the effectiveness of Warfarin?

PROTHROMBIN TIME ~ warfarin is an oral anticoagulant that inhibits carboxylation of Vitamin K dependent (2,7,9,10) used in afib, dvt, and PE. PT (prothrombin time) should be monitored. Warfarin = coumadin = vitamin K antagonist

Two curves, transport across membrane vs. solute concentration...what best explans the difference in curve shape?

PROTIEN CARRIER..graphs are showing simple diffusion vs. facilitated diffusion(requires carrier proteins (this method of diffusion can result in saturation).

As you move up and to the right on the oxy-hemoglobin dissociation curve...what do hemoglobin molecules release?

PROTONS In the lungs, the binding of oxygen to hemoglobin drives the release of H+ & CO2 from hemoglobin (HALDANE EFFECT). In the peripheral tissues, high concentrations of CO2 & H+ facilitate oxygen unloading from hemoglobin (BOHR EFFECT)

40 year old disholved woman smells like alcohol has stomach pain but cannot point to where it is or accurately describe it, does not have health insurance. What is the next best step in handling the situation?

PROVIDE APPROPRIATE MEDICAL SCREENING EXAM AND STABILIZE HER CONDITION ~ emergency medical treatment & active labor act was enacted by congress to prevent hospitals from inappropriately transferring, discharing or refusing to tx indigent patients.

Where do most renal cell carcinomas (clear cell carcinomas) originate?

PROXIMAL RENAL TUBULES. Epitherlial cells of the promiximal renal tubules....has high lipid content

Woman comes to ED with fevers and chills of 24hrs, 3 weeks ago she had bowel resection for Crohn disease and since has had a catheter to get nutrition. Blood cultures return positive for yeast, what is the characteristic of the likely organism?

PSEUDOHYPHAE WITH BLASTOCONIDA ~ Candida [risk factors: central vascular catheter & parenteral nutrition]

When you see facial impetigo [honey-crusted, on face] in the question stem and it is talking about something with the kidneys what is the disease?

PSGN (post-streptococcal glomerulonephritis) = most common acute nephritis in children. Type THREE (IMMUNE COMPLEX) hypersensitivity. Previous B-hemolytic strep infection in stem might be worded as: impetigo, cellulitis, pharyngitis...(serum sickness & lupu

Electron-dense deposits on the epithelial side of the glomerular basement membrane are seen in patients with what?

PSGN (post-streptococcal glomerulonephritis) = most common acute nephritis in children. Type THREE (IMMUNE COMPLEX) hypersensitivity. Previous B-hemolytic strep infection in stem might be worded as: impetigo, cellulitis, pharyngitis...(serum sickness & lupus). IMMUNE COMPLEX DEPOSITS: IgG, IgM, C3

Kid with edema, hematuria, strep infection history...disease and type of hypersensitivity txn?

PSGN (post-streptococcal glomerulonephritis) = most common acute nephritis in children. Type THREE (IMMUNE COMPLEX) hypersensitivity. Previous B-hemolytic strep infection in stem might be worded as: impetigo[honey-crusted usually on face], cellulitis, pharyngitis...(serum sickness & lupus are also type 3)

How does PSGN (post-strep glomerulonephritis) look on IF?

PSGN occurs frequently in children, NEPHRITIC (oliguric renal failure, HTN, red cell casts, mild proteinuria & edema) following recent skin infection. IF granular deposits of: C3, IgG & IgM in mesangium and basement membranes.

Patient presents with fever, back/flank pain, inguinal mass & difficulty walking. Patient lies supine with his right knee flexed and externally rotated. He resists simultaneous extension of the leg & thigh, particularly at the hip. The pathological process occuring in this patient most likely involves what muscle?

PSOAS MAJOR ~ psoas abscess can occur as the result of hematogenous or lymphatic seeding from a distant site of by spread form an adjacent site. Patients can present with fever, back/flank pain, inguinal mass & difficulty walking. Inflammation of the psoas muscle causes pain when the hip is extended (psoas sign)

Man has hemmroids, one of which is below the denate line and is extremely tender, excision under local anesthesia is planned....the anesthetic agent blocks sensory impulse carried by what nerve?

PUDENDAL NERVE (cutaneous/somatic nervous innervation from the inferior rectal nerve, a branch of the pudendal nerve)

Stretch injury to what nerve may occur due to stress placed on the pelvic floor during labor?

PUDENDAL NERVE (ventral rami S2-S4) Pudendal nerve injury leads to weakness of the perineal musculature causing fecal and urinary incontinence, perineal pain & sexual dysfunction.

What arteries are typically spared by Polyarteritis Nodosa?

PULMONARY ARTERIES ....PAN involves medium to small sized arteries impacting most organ systems with the exception of the pulmonary arteries

Question stem describes sleep apnea (tired, snores, narrow oropharynx & large neck circumference) what is patietn at risk for developing?

PULMONARY HYPERTENSION ~obstructive sleep apnea presents in obese individuals with excessive daytime sleepiness and signs of noctural upper airway obstruction (snoring/gasping). The condition is associated with systemic hypertension.

Where is pulmonary vascular resistance the lowest?

PVR is lowest at functional residual capacity. Increased lung volumes increase PVR due to longitudinal stretching of alveolar capillaries by expanding alveoli

5 year old has trouble "seeing the board" tall, thin, MCA thrombosis and dies from cerebrovascular accident...what supplement could have been given to prevent this?

PYRIDOXINE Patient had homocystinuria= cystathione deficiency [most common inborn error of methionine metabolism]. Many homocystinuria patients respond dramatically to pyridoxine (vitamin B6)

Researcher investigates rxns in liver, cultures glutatamate with nitrogen isotopes and after some time the nitrogen isotopes are transferred to oxaloacetate, forming aspartate in the process...what vitamin is he studying?

PYRIDOXINE ~ Vitamin B6 ~ is necessary for the transamination and decarboxylation of amino acids, for gluconeogenesis, and for other essential biochemical processes. TRANSAMINATION RXNS OCCUR BETWEEN AN AMINO ACID & A ALPHA-KETOACID (becomes an amino acid) B6/PYRIDOXINE serves as a cofactor in amino acid transformation and decarboxylation rxns

Bisphosphate drugs (ie alendronate) have a chemical structure similar to what drug?

PYROPHOSPHATE (diphosphate) - bisphosphonates have a chemical structure similar to pyrophosphate and disrupt osteoclast function by attaching to hydroxyapatite binding sites on bony surfaces

Person comes in with severe abdominal pain, gets appendectomy then questions whether it was necessary...works alone for fear of being "setup" disorder?

Paranoid Personality Disorder

What is the major cause (80%) of malignancy induced HYPERcalcemia?

Parathyroid hormone-related protein (PTHrP causes bone resorption and decreased renal excretion of calcium)

Caput meduse portal vs. systemic

Paraumbilical Veins(portal) Superficial/Inferior Epigastrics(systemic)

Patient has -CD55 & -CD59 (deficiencies in both) what is disease and cause?

Paroxysmal Nocturnal Hemoglobinuria (PNH) from COMPLEMENT ACTIVATION. PNH = hemolytic anemia + hypercoagulability + pancytopenia. PNH = acquired mutation in PIGA gene which causes the absense of the GPI anchor and deficiency of CD55 & CD59 complement inhibitor proteins

Patellar fracture, what kind of bone is the patella? What movement is inhibited after a fracture?

Patella = sesamoid bone. Patient will be unable to extend the knee against gravity. (also will have swollen knee and patellar tenderness) & palpable gap in EXTENSOR mechanism.

4 year old, failure to thrive and exercise intolerance, has continous murmur best heard over upper left sternal border. What does he have and what embryo derivative?

Patent ductus arterious from the 6th AORTIC ARCH (answer) - 6god

Best test to offer new husband in a relationship with a woman who previously birthed a sickle cell child?

Paternal hemoglobin electrophoresis

What is the disulfram-like reaction? Give an example.

Patient becomes sick (nausea, headache, vomitting, facial flushing & abdominal cramps) after drinking while taking certain medication (like metronidazole in the treatment of trichomatis vaginitis)

Old man from china several months of epigastric pain, mucosal defect in duodenum...whats up?

Patient has DUODENAL PEPTIC ULCER DISEASE...96% of duodenal peptic ulcer disease patients also have H. Pylori...needs an antiobiotic triple therapy (PPI(OMEPRAZOLE), AMOXICILLIN & ERYTHROMYCIN)

Dizziness, mild confusion, low serum sodium concentration What is patient experiecing? What site is the hormone secreted at?

Patient has SIADH (syndrome of inappropriate ADH secretion. ADH/Vasopressin & Oxytocin are synthesized in magnocellular neurons found in SUPRAOPTIC & PARAVENTRICULAR NUCLEI OF HYPOTHALAMUS...once synthesized that travel anterograde to and are released from posterior pituatry. NEUROPHYSINS are involved in the posttranslational processing & stabilization of oxytocin & vasopressin within neurosecretory vesicles during transport to the posterior pituitary.

Man comes to ER with SHARP, mid-chest pain that hurts on INSPIRATION and decreases when he sits up. What else would you see?

Patient has acute pericarditis you would also see FRICTION RUB (pericardial friction rub).

patient with hypercholesterolemia & hypertension suffers a sudden-onset neurological deficit (right arm weakness/difficulty speaking) what happened? What is the most common side effect for the drug that will be needed daily?

Patient suffered a TIA (transient ischemic attack). Low-dose aspirin may prevent ischemic stroke by irreversibly binding COX-1 and inhibiting it. But a common side effect is GI mucosal injury & bleeding

What are lacunar infarctions?

Patient with uncontrolled diabetes/hypertension dies of heart attack and on autopsy her brain shows small cavities in brain structure filled with clear fluid

Explain relationship between visual accomodation & 1st gen antihistamines?

Patients visual symptoms are most likely due to a blockade of Acetylcholine. Patient's impaired visual accomodation due to anti-cholinergic (anti-muscarinic) effects of 1st gen antihistamines. Anti-cholinergic effects on ciliary muscles impair accomodation & cause blurring of vision for close objects.

Patient with acute hepatitis due to severe hypertriglyceridemia...what is most effective tx?

Patients with hypertriglyceridemia have pancreatic lipases that can cause toxic levels of free fatty acids to be released within the pancreatic tissue leading to acute pancreatitis. FIBRATES attack triglycerides

Woman has cervicitis, what can it progress to and what are two likely bugs?

Pelvic inflammatory disease ~ INFERTILITY ~ PID can cause scarring of the fallopian tubes, leading to ecotpic pregnancy and infertility.

Pernicous anemia (Vit B12 deficiency) presents how? What layer of gastric is effected?

Pernicous anemia (Vit B12 def) presents as fatigue and lower-extremity paresthesia & megaloblastic RBCs. ANTIBODY-MEDIATED destruction of the parietal cells in gastric body and fundus. Parietal cells are oxyntic (pink) and are predominantly found in the upper glandular area (2nd from top) the parietal cells normally secrete: Intrinsic Factor & Gastric Acid

Incomplete/Abnormal Omphalomesenteric (vitelline) duct [7th week of embryo development] what 4 abnormalities can result?

Persistent vitelline duct, Meckel diverticulum, Vitelline sius, Vitelline duct cyst

Man has DKA how will his pH, HCO3-, 2PO4- look?

Ph(down); HCO3-(down); 2PO4-(UP)~ urinary acid excretion occurs primarily in the form of NH4+ and titratable acids (H2PO4-)...in metabolic acidosis, urinary pH decreases due to increased excretion of free H+, NH4+, H2PO-...bicarb is completely reabsorbed from tubular fluid in acidotic states.

What is the irreversible a1 & a2 adrenergic antagonist?

Phenoxybenzamine (reduces the arterial vasoconstriction induced by norepinephrine) since it is irreversible even high concentrations of norepinephrine cannot overcome its effects

Intellectual disability, gait/posture abnormality, eczema, musty body odor?

Phenylketonuria (Autosomal Recessive). The chance that the couple's next kid will have it is 1/4

What ratio is commonly used to measure fetal lung maturity?

Phosphatidyl Choline(lethicin)/Sphingomyelin the L/S ratio...Sphingomyelin is high early in pregnancy, Lethicin/phosphatidyl choline skyrockets close to 40 weeks gestation.

Phosphorylation of what leads to insulin resistance (easy point, know it)

Phosphorylation of Serine & threonine residues of insulin receptor and insulin receptor substrate by serine kinase leads to insulin resistance. This type of phosphorylation can be induced by TNF-alpha, catecholamines, glucocorticoids & glucagon.

What is photoaging and what kind of light is responsible for it?

Photoaging is a product of too much exposure to UVA LIGHT and is characterized by epidermal atrophy with flattening of the RETE RIDGES. DECREASED COLLAGEN FIBRIL PRODUCTION & INCREASED DEGRADATION OF COLLAGEN & ELASTIN

Pleuric chest pain from lower lobe pneumonia (sharp pain on inspiration) is carried via what nerve?

Phrenic (c3-5) it is referred to the shoulder/neck.

Old man, blood cultures grow beta-lactamase producing Bacterioides...what is appropriate tx?

Piperacillin-Tazobactam [extended-spectrum penicillin + B-lactamase inhibitor] It is effective against most gram negative enteric rods (including pseudomonas and bacterioides fragilis)

What can cause bitemporal hemianopsia & galactorrhea leading to oligo/amenorrhea?

Pituitary Tumor from the LACTOTROPH CELLS

30 year old woman only menstrates 2-3 times per year, BMI 35, mild acne & hair growth on upper lip and chin...what does patient have and what is she at risk of?

Polycystic Ovary Syndrome ~ risk of Endometrial carcinoma. PCOS is associated with hirituism, oligomenorrhea, polycystic ovaries

Androgenetic alopecia (male pattern baldness) follows what inheritance?

Polygenetic...hair loss is drive by hormonal & genetic factors.

Relationship between mean and median for a positively skewed graph

Positively skewed = MEAN IS GREATER THAN THE MEDIAN

What Strep Gallolyticus (strep bovis) is cultured, what else should be worked up immediately?

Possible colonic neoplasia..Strep gallolyticus is part of the normal flora of the colon, COLONOSCOPY is essential.

A patient presents with signs of left (bibasilar crackles) and right (jugular venous distenstion, peripheral edema) decompensated heart failure, what else is expected to be seen?

Possibly dilated cardiomyopathy caused by viral myocarditis. Myocardial inflammation leads to eccentric hypertrophy (thinning out) w/decreased ventricular contractility (systolic dysfunction)

Patient presents with right sided numbness/tingling-> full right side paresthesia -> seisures...select part of brain responsible for ORIGIN os seizure?

Post-central gyrus (primary somatosensory) the ORIGIN is there because stroke presented with initial numbness

What are post-partum blues?

Post-partum blues are a normal (usually self-limiting) condition that occurs within a few days post-partum [sadness, insomnia, anxiety] symptoms peak at 5 disappear around 10 days usually without intervention. "POSTPARTUM MOOD CHANGES ARE COMMON, PLEASE CALL IF YOU DO NOT IMPROVE IN A FEW DAYS"

What kind of lesion can produce homonymous hemianopia with macular sparing?

Posterior Cerebral Artery occlusion (occipital cortex damage)

Where does the posterior cerebral artery come from and what does it supply?

Posterior Cerebral artery branches off basilar artery...supplies CN 3 & 4 and other structures in the midbrain. Most common finding with a PCA stroke is contralateral homoymous hemianopia, often with macular sparing.

Patient has rupture of POSTEROMEDIAL papillary muscle, what blood flow would be compromised?

Posterior descending artery.

What part of the urethra is commonly injured with pelvic fractures?

Posterior urethra (and if urethral injury is suspected foley catheter is contraindicated)

When hematuria, proteinuria & urine RBC casts are present what is likely diagnosis?

Poststreptococcal glomerulonephritis (PSGN) Lab findings: LOW C3 CONCENTRATION(c4 is normal), elevated anti-DNAase B titers, elevated anti-ASO

Physio question about ion flow, when this ion channel is open it's the only one that flows out?

Potassium (potassium, sodium, chloride) determine electrical potential

Resting membrane permeability: Ligand-gated ion channel permeability: Voltage-gated ion channel permeability:

Potassium, Sodium, Chloride....Changed in membrane potential occur in response to changes in the neuronal membrane permeability to different ions. The more permeable the membrane becomes for a cellular ion, the more that ion's equilibrium potential contributes to the total membrane potential.

Pulmonary hypoplasia + facial/lower limb deformities...whats wrong?

Potter sequence (renal anomoly/renal agenesis) decreases fetal urine output leading to oligohydramnios ...lack of amnionic fluid causes compression of the fetus (characteristic facies and limb abnormalities) & pulmonary hypoplasia leading to death.

What is the probability of rejecting a null hypothesis when it is truly false?

Power = (1-B)

Behavioral change often happens in 5 distinct stages what are they:

Pre-contemplation (denial of problem); Contemplation(acceptance of problem, thinking of change); Preparation(planning to make change in near future); action(putting active changes into place); maintenance(maintaining change over the long term).

Acute trauma or chronic repetitive pressure on what BURSA in the knee can cause anterior knee pain from kneeling?

Prepatellar bursa (synovial, fluid-filled sac)...supposed to alleviate pressure from bony prominences and reduce friction between muscles and tendons. PREPATELLAR BURSITIS causes anterior knee pain usually due to kneeling.

What two conditions (one eye one skin) are specifically age-related?

Presbyopia(denaturation of lens proteins & changes in lens curvature) Wrinkles(decreased synthesis & increased breakdown of collagen and elastin)

What is the nutcracker syndrome?

Pressure in left renal vein may become elevated due to compression where the vein crosses the aorta beneath the superior mesenteric artery [hematuria & flank pain]. Elevated pressure in left gonadal vein can form a varicoecele

Patient with ovarian cancer (CA-125) showing high telomerase activity, how does telomerase directly impact growth and malignancy?

Prevents chromosomal shortening. Telomerase is an RNA-dependent DNA polymerase that synthesizes telomeric DNA sequences that can replace the lost chromosomal ends of telomeres. Cancer cells typically contain increased telomerase activity.

Small fibrotic focus in the LOWER lobe of the right lung and a calcified lymph node in the right lung hilus...what kind of TB exposure?

Primary Tb (Ghon complex) in lower lobe & ipsilateral hilar adenopathy....Secondary TB ~ upper lobe, cough, hemoptysis and weight loss...acid fast sputum

Difference between primary and secondary TB infection

Primary Tb (Ghon complex) in lower lobe....Secondary TB ~ upper lobe, cough, hemoptysis and weight loss...acid fast sputum

Herpes simplex primary versus reccurent (movement)

Primary infection moves retrograde to trigeminal ganglion (via DYENINS-microtubule associated motor protein). Reactivation (stress/triggers) uses anterograde movement from trigeminal ganglion to skin via KINESINS-microtubule associated motor protein.

Primary, Secondary, Tertiary prevention

Primary(prevention-health promotion). Secondary(detection before disease causes SYMPTOMS-case finding, screening). Tertiary(tx disease to prevent further probs-rehab)

The question asks about bacterial DNA and Uracil (normally only found in RNA) and what enzymes is responsible for formation of the RNA strands?

Primase = DNA-dependent RNA polymerase that incorporates short RNA primers into replicating DNA.

Presentation of an adrenalCORTICAL adenoma?

Produces a lot of aldosterone which would present clincially as PARESTHESIAS & MUSCLE WEAKNESS.

What hormone in women increases the most in concentration following ovulation?

Progesterone. During latter half of the menstrual cycle, the corpus luteum secretes high levels of progesterone which thickens the endometrium and prepares it to receive/nourish blastocyts

All hormone-containg contraceptives prevent pregnancy through actions of ___? How do they work?

Progestins (which DO thin the uterine lining but more importantly they stop ovulation), the main mechanism of contraceptives with systemtically active progestins [combined hormonal oral contraceptives] is inhibiting ovulation by decreasing FSH & LH synthesis in the anterior pituitary

Classic triad of normal-pressure hydrocephalus?

Progressive gait difficulties, cognitive disturbances & urinary incontenence. Brain imaging reveals ventricular enlargement that is out of proportion to sulci enlargement.

What is propionic acidemia? How does it present?

Propionyl Co-A is derived from the metabolism of Valine, Isoleucine, methionine, threonine & odd-chain fatty acids. Congenital deficiency of proprionyl-CoA-carboxylase = Proprionic Acidemia = lethargy, poor feeding, vomitting, hypotonia

Furosemide is a Na-K-2Cl symporter in the loop of henle what else do loop diuretics stimulate the release of?

Prostaglandin release! This will increase renal blood flow and increase GFR & drug delivery. However, using this with NSAIDs can result in decreased diuretic response.

What is the most common cause of malignant otitis externa (MOE)...serious ear infection seen in old diabetic patients?

Pseudomonas Aeruginosa [nonlactose-ferminting, oxidase-positive, motile, gram-negative rod] MOE = ear pain & drainage and granulation tissue seen in ear canal

Ecthyma Gangrenosum [cutaneous necrotic disease] is associated with what?

Pseudomonas Aeruginosa bacteremia (common in hospitalized patients, who are neutropenic/have burns/have endwelling catheters. Pseudomonas virulence factors include: exotoxin A(protein synthesis inhibition), elastase (destroys elastin in blood vessels) phospholipase C (degrades cellular membranes) & procyanin ( generates ROS).

What is necessary for diagnosis of schizoaffective disorder?

Psychosis must occur in the absence of major mood episodes. But mood symptoms must be present for most of the illness.

Intravaginal anesthetic, medial to ischial spine, through sacrospinous ligament...what nerve is blocked what happens after nerve is blocked?

Pudendal nerve S2-S4 (Sensory: perineum. Motor: urethral & anal sphincters) important landmarks are: ischial spines & sacrospinous ligament

Filamentous, branching, gram-positive anaerobic bacteria & sulfur granules?

Pulmonary actinomycosis

Reduced pulmonary parenchymal compliance (less lung volume & increased transpulmonary pressure) is the hallmark of what?

Pulmonary fibrosis (reduced lung volume, increased transpulmonary pressure)

What is pulsus paradoxis and what 4 illnesses can it be seen in?

Pulsus paradoxus is defined by a decrease in systolic blood pressure for greater than 10 mmHg with inspiration. Seen in patients with: Cardiac Tamponade, Severe Asthma, COPD & Constrictive PERICARDITIS

Order of speed of conduction via electrical properties of the heart (fun mneumonic)

Purkinje, Atrial muscle, Ventricular muscle, AV node "Park At Venture Ave" ...fastest at purkinje system, slowest at AV node

14 year old girl comes to physician with dark urine and facial puffyness, 4 weeks earlier she had pustular skin lesions that broke down over a few days to form thick scabs...it's sensitive to what?

Pyrrolidonyl arylamidase positive (PYR has replaced bacitracin in most labs) She had impetigo now has PSGN. Strep A = catalase nega, beta-hemolytic, bacitracin-susceptible, PYR-positive

Kid visits grandparents farm over summer break cleaning animal poop, shortness of breath, fatigue, myalgias, debilitating retroorbital headaches. LFT: injury. What bug?

Q fever from Coxiella Burnetti [zoonotic infection] increased liver enzymes.

Benzodiazepines, Antihistamine, Sedating antidepressants should be AVOIDED in treatment of insomnia in the elderly due to their adverse effects. What drug has demonstrated safety and efficacy in older adults?

RAMELTEON, a melatonin agonist, is safe and effective in reducing time to sleep and onset of sleep in the elderly

patient ingests rat poison, what is rat poison the "superdrug" version of and what can be used to fix this?

RAT POISON = SUPERWARFARIN ...YOU GIVE VITAMIN K + FFP

Man has complete thyroid ectomy performed (had papillary thyroid cancer) surgeon accidentally damages a structure near the inferior thyroid artery, what nerve was injured?

RECCURENT LARYNGEAL NERVE ~ travels in close proximity to the inferior thyroid artery and can be injured during thyroid surgery, resulting in vocal cord paralysis.

22 year old woman: dysuria, vulvar pain, itching...inguinal lymphadenopathy & tender vesicular lesions covering both labia majora & perineum...patient's infection can lead to what condition?

RECURRENT GENITAL ULCERS HSV-2 (sacral DRG and can be reactivated to cause reccurent genital lesions)

Immigrant kid has required several blood product transfusions due to anemia, on PE he has moderate spenomegaly. Labs on circulating RBCs show low pyruvate kinase activity. What is the most likely cause?

RED PULP HYPERPLASIA ~ PYRUVATE KINASE DEFICIENCY causes hemolytic anemia due to failure of glycolysis and resultant failure to generate sufficient ATP to maintain erythrocyte structure. SPLENIC HYPERPLASIA results from increased work of the splenic parenchyma, which must remove these deformed erythrocytes from the circulation

Kidney stones in someone with Crohn ileocolitism what is the likely underlying causes?

REDUCED INTESTINAL CALCIUM OXALATE BINDING ~ enteric oxaluria ~ impaired bile acid absorption in the terminal ileum leads toloss of bile acids in feces with subsequent fat malabsorption. Lipids then bind alcium ions, and the resulting soap complex is excreted. Free oalate is absorbed and forms calculi

Woman has double vision (new) and complains about difficulty chewing (no extremity weakness or sensory complaints)....what is going on at the cellular level?

REDUCED MOTOR END-PLATE POTENTIAL ~ myasthenia gravis is an autoimmune disease that results in decrease in functional acetychooline receptors on the postsynaptic terminal of the NMJ

Fat sendentary dude is started on Fenofibrate, what is mechanism of action?

REDUCING HEPATIC VLDL PRODUCTION...Fibrates lower the triglyceride levels by activating peroxisome proliferator-activated receptor alpha (leads to decreased hepatic VLDL production & increased lipoprotein lipase activity).

New test availible, Lab results show: 11.2, 13.4,17.6 what concerns should be raised?

RELIABILITY (reliability = reproducible...similar results on repeat measurements) Reliability is maximal when random error is minimal.

7 year old boy being evaluated for growth retardation, has multiloculated, cystic, suprasellar lesion bulging into the floor of the 3rd ventricle and base of the brain. From what structure is the third mass likely derived?

REMNANTS OF THE RATHKE POUCH ~ Craniopharyngiomas are tumors arising from remnants of Rathke's pouch. The ANTERIOR PITUITARY IS FORMED FROM AN OUT-POUCHING OF THE PHARYNGEAL ROOF = RATHKE'S POUCH

What about getting old requires an old man to take a "weight-adjusted" adult dose of digoxin?

RENAL CLEARANCE ~ elderly patients typically exhibit age-related renal insufficiency, even in the presence of normal creatinine levels. (dose reduced to prevent toxicity)

Patient with flank pain, hematuria, elevated lactate dehydrogenosa and wedge-shaped right kidney lesion on ct likely has what?

RENAL INFARCTION DUE TO THROMBOEMBOLISM. SYSTEMIC THROMBOEMBOLISM OCCURS WITH ATRIAL FIBRILLATION

Abrupt-onset gross hematuria in an otherwise healthy patient with family history of sickle cell?

RENAL PAPILLARY NECROSIS ~ presents with gross hematuria, acute flank pan, and passage of tissue fragments in urine. Commonly seen in patients with sickle cell disease or train, diabetes mellitus, analgesic, or obstructive pyelonephiritis

Bone marrow biopsy is performed and there is a balanced translocation between chromosomes 15 & 17, what proteins is most likely to be abnormal in the hematopoietic cells of the patient?

RETINOIC ACID RECEPTOR - AML t(15;17) ACUTE PROMYELOCYTIC LEUKEMIA (APML = M3 variant of AML) involves RARA = Retinoic Acid Receptor Alpha from chromosome 17 and promyelocytic leukemia (PML) gene on chromosome 15

Loss of speech and motor skills, deceleration of head growth, and wringing hand movements AFTER a period of normal development?

RETT SYNDROME mainly effects girls and is associated with MECP2 gene mutation. [development starts normal then drastic downfall]

Biggest difference between a dysplatic tissue and a carcinoma?

REVERSIBILITY OF CHANGES OF DYSPLASTIC...dysplasia is reversible, epithelial malignancies progress through the sequence of low-grade dysplasia -> high-grade dysplasia/carcinoma in situ -> invasive carcinoma. Once dysplastic cells have breached the basement membrane process is no longer reversible (carcinoma).

What stool findings would be most suggestive of strongyloides stercoralis?

RHABDITIFORM LARVAE IN THE STOOL (can also result in a hyperinfection syndrome with massive dissemination of the organism, leading to shock)

ST elevation in leads II, III & aVF as well as sinus node dysfunction

RIGHT CORONARY ARTERY OBSTRUCTION

Where would you auscultate in order to listen to the endocarditis commonly seen in IV drug users?

RIGHT HEART TRICUSPID ENDOCARDITIS ~ STAPH IV DRUG USERS

Man presents with left-sided weakness, left arm more weak than left leg...left face weakness...everything on right is fine. What artery in image occluded?

RIGHT MIDDLE CEREBRAL...hemiparesis & hemi-sensory loss

Woman stabbed adjacent to the left setnal border in the 4th intercostal space, what structure was likely injured?

RIGHT VENTRICLE [skin, pec major, external intercostal membrane, internal intercostal muscle, internal thoracic artery & veins, transversus thoracis muscle, parietal pleura, pericardium, RV myocardium

Image of a cell with basophillic nucleoli is shown and the question is asking, what cellular enzyme works here?

RNA POLYMERASE I...the nucleolys is the site of ribosomal subunit maturation & assembly. RNA pol I functions exclusively within the nucleolus to transcribe the 45S pre-rRNA gene, which codes for most of the ribosomal RNA components (18S, 5.8S & 28S rRNAs)

Relationship between Gene Silencing & RNA interference?

RNA interference is when short non-coding RNA sequences induce POSTTRANSCRIPTIONAL GENE SILENCING (non-coding regions base pair with complementary sequences of targets and they are silenced. mRNA TRANSLATION is direclty interrupted

Patient bleeding while brushing teeth, has coiled hairs and perifollicular hemorrhages...disease and hypoactive enzyme compartment?

ROUGH ENDOPLASMIC RETICULUM ~ SCRUVY (VITAMIN C DEFICIENCY) hydroxylation of proline and lysine residues in collagen helps attain its maximum tensile strength (this process occurs in the rough endoplasmic reticulum)

A quick way to figure out RPF from hematocrit and RBF?

RPF = RBF*(1-Hematocrit)

FDA will approve drug if it reduces reccurence by 40%; Recurrence rate of therapy alone = 8% in order for FDA to approve what is the maximal incidence of recurrent disease in new drug?

RRR = (Arcontrol - Artreatment)/Arcontrol 60% of 8% = 4.8%

Ct with a crescent shaped mass, what vessels respondible?

RUPTURE OF CORTICAL BRIDGIN VEINS = SUBDURAL HEMATOMA

55 yo man with chest pain, does not respond to aspirn or sublingual nitroglycerin...ST elevation in leads I, aVL, V1-V3 with deep Q wave development over next several hours. Cardiac cath would show what?

RUPTURED ATHEROSCLEROTIC PLAQUE WITH FULLY OBSTRUCTIVE THROMBUS ~ an acute transmural myocardial infarction marked by ST-elevation and subsequent Qwave formation is most likely the result of a fully obstructive thrombus superimposed on a ruptured atherosclorotic coronary artery plaque

Hypotension, elevated JVP and clear lungs, what kind of MI? how will CO, Pulmonary Capillary Wedge Pressure and CVP look?

RV MI~ CO(down), Pulmonary Capillary Wedge Pressure(down), CVP(UP)

lateral Supracondylar humeral fracture of pediatric elbow, from hyperextension injury..what is likely damaged?

Radial Nerve

What kind of nerve injury can result from axillary pressure (being on crutches)? What movement will be impaired and what reflex will be absent?

Radial nerve injury can occur with reptitive trauma/pressure at the level of the axilla. Findings include: weakness of forearm, hand, finger extensors [wrist drop/absent triceps reflex] & sensory loss over posterior arm & forearm, dorsolateral hand and dorsal thumb...a more distal lesion would spare triceps brachii.

Patient with weakness in finger and thumb extenion (finger drop)..everything above elbow is fine, what nerve and where was nerve injured?

Radial nerve...injured upon entrance into the SUPINATOR CANAL

How does Raltegravir work in HIV?

Raltegravir is an integrase inhibitor that disrupts HIV genome integration into the host cell's chromosomes, thereby preventing host cellular machinery from being used to synthesize HIV mRNA.

Order of events for the cardiac myocyte action potential:

Rapid depolarization(phase 0), initial rapid repolarization(phase 1) plateau (phase 2), late rapid repolarization (phase 3) & resting potential (phase 4). Action potential is associated with increased membane permeability to Na+ & Ca++ and decreased permeability to K+

Crescent formation on light microscopy is diagnostic for what?

Rapidly progressive glomerulonephritis. Crescents are made up of FIBRIN, glomerula parietal cells, monocytes, macrophages,

Woman hates kids but is overly concerned when her stepson hurts himself,what kind of defense mechanism?

Reaction formation: responding in a manner opposite to one's actual feelings

What is the classic triad of reactive arthritis and what HLA is It associated with?

Reactive arthritis (HLA-B27 arthropathy): nongonoccal urethritis, conjunctivitis, arthritis. [there can also be SACROILITIS ~20%]

painful arthritis when using stairs, but ok with weight bearing & after a diarrheal illness,

Reactive arthritis = Positive HLA-B27 [histocompatibility antigen]...reactive arthritis in campylobacter, shigella, salmonella, yersina, chlymidia or bartonella

What is described by the mixing of genome segments in segmented viruses that infect the host chromosome

Reassortment, genetic SHIFT involves the reassortment of genetic segments of the human strain with the gneomic segments of the animal

What is the process of mixing genomic sequences in segmented viruses that infect the same host cell? Give an example

Reassortment, genetic SHIFT involves the reassortment of genetic segments of the human strain with the gneomic segments of the animal

Microscopic examination shows spherules packed with endospores, patient's history is likely to reveal what?

Recent travel to Arizona [coccidioides immitis is a dimorphic fungus endemic to southwestern US, it exists in the environment as a mold (w/hyphae) that forms spores....spores are inhaled and turn into spherules in the lungs

Man has lump on bone and is losing his hearing, what factor is essential for differentiation of the numerous multinucleated cells see on pathology?

Receptor activator of nuclear factor kappa-B ligand. Osteoclasts originate from hematopoietic progenitor cells, macrophage colony-stimulating factor and receptor for actived NFK-B play an important role in osteoclast differentiation. [Paget = increased abnormal osteoclasts -> excessive bone turnover and disorganization]

What is it called when there is gene exchange that occurs through the crossing over of 2 double stranded DNA molecules?

Recombination

Neural crest cells (precursors of ganglion cells of intestinal wall plexi) migrate caudally, what will also be effect by hirschprung diease?

Rectum

Kid hit with baseball in right eye, orbital floor damage, in addition to inferior rectus muscle becoming entrapped and limiting vertial gaze....what othe impairments can be seen?

Reduced sensation over right upper lip. V2 from trigeminal (infraorbital nerve) exits a foramen adjancent to the orbital floor resulting in paresthesia of upper cheek/lip & upper gingiva.

New measuring device gives a reading of 200 on 3 separate occasions, use of the gold-standard gives a reading of 260...describe the device:

Reliable but not accurate. Precision = reliability

Man is a vegetable and his wife says remove feeding tubes because he didn't want to live like this, children do not agree...what do you do?

Remove the feeding tube in accordance with the patient's wishes as described by his wife

Acute interstitial nephritis from starting a beta-lactam antibiotic or other specific drug will do its damage on what part of kidney?

Renal INTERSTITIUM

How will levels of renin, ang 1, ang 2, aldosterone and bradykinin change with use of Losartan?

Renin increases, Ang 1 increases, Ang 2 increases, Aldosterone decreases. No change in bradykinin.

What is the most common occular tumor of childhood? How does it present?

Retinoblastoma, presents with a white pupillary reflex (leukocoria) in children under 5

Woman comes in with right weakness/difficulty speaking, history of diastolic murmur, strokes out and dies autopsy shows diffuse fibrous thickening and distortion of mitral valce leaflets & narrowing of mitral valve orfice, what condition did she likely have

Rheumatic fever ...rheumatic mitral stenosis is characterized by diffuse fibrous thickening and distortion of the mitral valve leaflets along commissural fusion at the leaflet edges.

Sore throat, runny nose, fever, cough...Naked RNA virus

Rhinovirus...acts as mRNA and uses host cell machinery for translation. (single strand positive sense RNA is infectious)

Definition of RISK?

Rick is the probability of developing a disease over a certain period of time. To calculate: divide the number of affected subjects by the total number of subjects in the corresponding exposure group.

Patient takes phenytoin for her tonic -clonic seizures, claims to be compliant but blood test shows her phenytoin level is low, what else (kind of drug) is she taking?

Rifampin (or any other P450 inducers: barbiturates, rifampin, carbamazepine, griseofulvin, chronic alcohol) enhances the metabolism of phenytoin...decreasing its serum concentration.

On posterior anterior (PA) chest x-ray projections what composes most of the right side of the cardiac silhoutte?

Right atrium

Relationship between right renal vein and artery? Gonads?

Right renal vein is short and runs anteriorly to right renal artery before joining IVC. Right gonadal vein also drains directly into the IVC

What is risedronate and how does it work?

Risedronate is a bisphosphate which treats osteoporosis by inhibiting mature osteoclast-mediated bone resorption. Chemical structure similar to pyrophosphate and attach to hydroxyapetite by binding sites on bony surfaces to INHIBIT BONE REABSORPTION BY MATURE OSTEOCLASTS

Man with severe low back pain, PE: pain with flexing the back & raising the legs. Pinprick periana does NOT cause rapid contraction of the anal sphincter, what nerve root is most likely involved?

S4 - CAUDA EQUINA SYNDROME ~ saddle anesthesia & loss of anocutaneous reflex are symptoms of cauda equina syndrome which is associated with damage to the S2 - S4 nerve roots.

Missionary returned from latin america with fever, headache, abdominal pain, 1 week history of watery diarrhea that has recently become bloody, PE: hepatosplenomegaly & faint, erythematous maculopapular lesions on chest/abdomen...what bug?

SALMONELLA TYPHI = typhoid fever, fecal-oral route,penetrates gut mucosa via M CELLS and bacteria-mediated endocytosis by enterocytes. Bacteria are able to survive and proliferate within macrophages fond in peyer's patches..ab pain, salmon-colored rose spots on chest, HEPATOSPLENOMEGALY, GI bleeding

An african american man presenting with bilateral hilar adenopathy and pulmonary complains who works in urban hospital is concerning for what?

SARCOIDOSIS

Findings of hilar adenopathy, pulmonary infiltrates & non-caseating lung granulomas in african american female point to what? What cells contribute to pathological finding?

SARCOIDOSIS, Th1, IL-2, IFN-gamma

Woman who works in a day care has intensely pruitic rash (worse at night) on wrist, fingers, finger webs...

SARCOPTES SCABIEI mite & eggs = highly contagious disease that presents with an intensely pruritic rash (worse at night) in the flexor surfaces of the wrist, lateral surfaces of fingers & finger webs. Diagnosis is confirmed by skin scraping from excoriated lesions that show mivtes, ova and feces under LM.

Relationship between thiazide diuretics and calcium?

SAVE CALCIUM. Thiazide diuretics work at the distal convoluted tubule and can cause HYPERcalcemia, HYPERuricemia, hyponatremia & hypokalemia. They work by blocking Na-Cl symporters in the distal convoluted tubules [causing sodium, chloride & water excretion]

baseball kid with extra rib has right arm numbness + dull pain and decreased sensation over medial two fingers and hypothenar eminence..what structure is contributing to the patient's condition?

SCALENE MUSCLES (thoracic outlet syndrome is due to compression of the lower trunk of the brachial plexus in the scalene triangle, the space borded by the anterior and middle scalene muscles and the first rib. (symptoms: upper extremity numbness, tingling & weakness)

Mass removed from brain positive for S-100 immunoreactivity, what is the diagnosis/

SCHWANNOMA...normally present with a biphasic pattern of cellularity (Antioni A & B arease) and S-100 positivity (along with melanomas due to their neural crest origin).

13 month old baby tahcypneic, periorbital cyanosis reccurrent otitis media, pnuemonia, thrush +chronic diarrhea +failure to thrive + Pneumocystis jiroveci...what disease?

SCIDs (severe combined immune deficiency) = defective T cell development & B cell dysfunction. (loss of cellular and humoral immunity) Patients present with reccurent bacterial and viral and fungal opportunistic infections

Patient undergoing treatment for myasthenia gravis has new cramping, nausea, sweating diarrhea give an example of a selectictive muscarinic acetylcholine receptor antagonist that will reduce effects of cholinesterase inhibitors in gut?

SCOPOLAMINE ~ muscarinic overstimulation can be overcome by an antimuscarinic agent such as scopolamine

What is responsible for the release of bicarb-rich secretions from the exocrine pancrease?

SECRETIN ~ S CELLS in duodenal mucosa in response to stimulation by intraluminal acidity

Woman gets spleen lacerated in car accident, she already had celiac disease is given O-negative blood and has reaction inluding hives what is the diagnosis?

SELECTIVE IgA DEFICIENCY [her body formed IgE antibodies directed against IgA] patients will have sinopulmonary & GI infections as well as autoimmune disease (might have anaphylaxis during blood transfusion)

Patient with acid-fast bacilli and 15 pound weight loss, Isoniazid monotherapy in the patient would most likely result in what?

SELECTIVE SURVIVAL OF BACTERIAL CELLS SECONDARY TO GENE MUTATION...active tuberculosis is never treated with drug monotherapy due to fast emergence of mycobacterial antibiotic resistance from gene mutation.

Cross section of notmal myocardial cells with other areas of myocardium inflitrated by an amorphous and acellular pink material in someone with progressive exertional dyspnea, edema, ascites, elevated JVP with S4)

SENILE AMYLOIDOSIS ~ restrictive cardiomyopathy can be caused by infiltrative diseases and often results in diastolic heart failure due to ventricular hypertrophy with impaired ventricular filling. [areas of myocardium infiltrated by an AMORPHOUS & ACELLULAR PINK MATERIAL(AMYLOID)]

Rare disease in newborns can have all the bad things reversed if caught early, would you want a specific test or a sensitive test?

SENSITIVE (ability to correctly identify those WITH the disease). It is essential to have a HIGH sensitivity in screening tests. [negative result in highly sensitive test would rule a diagnosis out SnNout)

9 year old girl, occasional stomachaches, has friends but hates sleepovers...gets stomach aches during sleepover but feels better when mom picks her up. Also has nightmares of her parents dieing a horrible death

SEPARATION ANXIETY DISORDER (4weeks+ in kids....6weeks+ in adults)

Baby has deficiency in producting TETRAHYDROBIOPTERIN what is likely deficient in the baby?

SEROTONIN

16 year old soccer girl newly vegetarian, has fatigue and loss of endurance. She also has pale conjunctivae& her hemoglobin is 9.2....what would her labs look like?

SERUM FERRITIN(LOW); CIRCULATING TRANSFERRIN(HIGH), MCV(LOW), HYPERSEGMENTED NEUTROPHILS(NONE), SERUM FOLATE(NORMAL) ~ anemia in women of childbearing age is most commonly caused by iron deficiency secondary to menstrual blood loss. Iron deficiency = decreased serum ferritin, increased total iron-binding capacity (transferrin) and microcytic, hypochromic red blood cells.

1 year old black kid comes in with 3-hour history of severe swelling and tenderness of hands and feet...PE: bilateral severe swelling of hands & feet, what is likely to be abnormal?

SERUM HAPTOGLOBIN...dactylitis(painful swelling of the hands and feet) common presentation of sickle cell in young children. 1 of many vasoocclusive manifestations of SCD. Sickling episodes result in hemolysis leading to increased indirect bilirubin and lactate dehydrogenase and DECREASED HAPTOGLOBIN

Man gets MUCORMYCOSIS from using too much decongestant and is prescribed an antifungal drug that binds to ergosterol and alters the cell membrane permeability, what drug and what should be monitored?

SERUM POTASSIUM & MAGNESIUM LEVELS ~ Amphotericin B (binds ergosterol in fungal cell membranes to form holes) RENAL TOXICITY IS THE MOST NOTORIOUS SIDE EFX ~ SEVERE HYPOKALEMIA & HYPOMAGNESEMIA

It is determined that a patient needs long-term amiodarone therapy, what should be checked before this is started?

SERUM TSH LEVELS; it is 40% IODINE BY WEIGHT...amiodarone can cause hyperthyroidism due to increased thyroid hormone synthesis OR destructive thyroiditis with release of preformed thyroid hormone.

Combination of HYPOnatremia, osmolality, inappropriately concentrated urine and a lung mass is suggestive of what?

SIADH...an important cause of SAIDH is a paraneoplastic effect secondary to small cell carcinoma of the lung

SIGECAPS what is it and what does it stand for?

SIGECAPS = Sleep disturbance, loss of Interest, excessive Guilt, low Energy, impaired Concentration, Appetite disturbance, Psychomotor agitation/retardation, Suicidal ideation. (5+ = depressed)

What is a Kruckenburg tumor?

SIGNET. A gastric tumor that has metastasized to the ovaries and can present with unintended weight loss, epigastric pain, & adenexal masses. Histologically the metastatic tumor has large amounts of MUCIN and apically displaced nuclei SIGNET appearance.

Middle-aged adult with dyspnea on exertion, nodular densities on x-ray & CALCIFIED HILAR LYMPH NODES due to occupation...?

SILICA ~ SILICOSIS is a pneumoconioses that is frequently asymptomatic but can present with dyspnea on exertion and productive cough

What are the rotatory cuff muscles and what is their primary action?

SITS (supraspinatus, infraspinatus, teres minor, subscapularis)...contribute to shoulder stability. Supraspinatus injury = inability to Abduct

Graph is showing administration of propophol over time: concentration is high in the blood, eventually high in the brain then one more system is extremely low for a long time and eventually rises toward maximal drug administration what organ is this?

SKELETAL MUSCLE...poorly vascularized peripheral compartments like Skeletal muscle, Bone, fat...accounts for short duration of action of many anesthetics

Researchers looking at glucose transporters and insulin concentration, what cell types are represented by increasing glucose transporter expression as insulin concentration increases?

SKELETAL MUSCLE/adipose tissue (GLUT-4 insulin mediated). Insulin independent (brain, kidney, liver, intestine, RBCs = GLUT1,2,3,5

Acute pericarditis, facial rash, proteinuria?

SLE...serosal inflammation and/or PERICARDITIS/PLEURITIS painful during INSPIRATION relieved by leaning forward. PERICARDITIS IS THE MOST COMMON CARDIOVASCULAR MANIFESTATION ASSOCIATED WITH SLE (sharp pleuritic chest pain that is relieved by sitting up and leaning forward).

What is Buspirone associated with?

SLOW ONSET OF ACTION, buspirone is a nonbenzodiazepine anxiolytic used to tx generalized anxiety disorder (no risk of dependence)

Woman has mild intellectual disability, 20% of the cells have 45, X and the remaining 80% have 46, XX genotype. What is the explanation?

SOMATIC MOSAICISM ~ woman likely has Turner mosaicism, genetically different celllines within the body. Can result from several processes including chromosomal nondisjuntion or a mutation during the first stages of embryo development. 45,X/46,XX IS THE MOST COMMON MOSAICISM AFFECTING SEX CHROMOSOMES.

Man with angina, has prolonged QT interval on ECG...what heart medication is most likely responible?

SOTALOL ~ torsades de pointes = polymorphic ventricular tachycardia that occurs in the setting of a prolonged QT interval. [Antiarrhythmics(sotalol, quinidine); Antipsychotics(haloperidol); Antibiotics(macrolides; fluoroquinolones)]

18 month old with neurologic regression, hepatosplenomegaly, cherry-red macular spot? (5th percentile for height & weight); what substrate does the patient have an accumulation of?

SPHINGOMYELIN ~ Niemann-Pick disease, sphingomyelinase deficiency causes accumulation of the lipid sphingomyelin. [hepatosplenomegaly, neurologic regression & cherry-red macular spot in infancy]

Alcoholic patient with caput medusa, gynecomastia, sweet breath and disoreintation and spider angiomata...what findings in the patient have the same pathogenesis as his gynecomastia?

SPIDER ANGIOMATA ~ liver cirrhosis ~ HYPERESTRINISM (spider angiomata, gynecomastia, loss of sexual hair, palmar erythema

32 year old man comes to ER complaing of fever, malaise, maculopapular rash...when patient's serum is mixed with CARDIOLIPIN, lecithin and cholesterol there is extensive flocculation...what is the next step in management?

SPIROCHETAL ANTIBODIES = TREPONEMA PALLIDUM....[screen: RPR/VDRL; confirm: FTA-ABS]

Man with nonischemic cardiomyopathy, hx of heart failure, new shortness of breath on mild exertion. Hx HTN and HyperCholesterolemia....left ventricular ejection fraction is 30% what diuretics would most likely help improve survival?

SPIRONOLACTONE ~~ Mineralocorticoid receptor antagonists (spironolactone, eplerenone) imporve survival in patients with CHF and reduced LV ejection fraction. They should NOT be used in patients with hyperkalemia or renal failure

Man gets hurt injury involves an organ that is supplied mainly by an atery of the foregut even though the organ is NOT a foregut derivative...what organ on CT?

SPLEEN = mesodermal origin (dorsal mesentery). Although it is supplied by the splenic artery (a branch of celiac trunk/foregut)

Patient vomitting blood and passing black (guaiac-positive feces) Upper scope reveals a bleeding spot within a cluster of enlarged toruous veins in the gastric fundus, where is problem?

SPLENIC VEIN. The short gastric veins drain blood from the gastric fundus into the splenic vein. Pancreatic inflammation (pancreatitis/cancer) can cause a blood clot within the splenic vein, which can increase pressure in the short gastric veins and lead to gastric varices only in the fundus

Patient's forehead has actinic keratoses (hyperkeratosis and parakeratosis) what is he at greatest risk for developing?

SQUAMOUS CELL CARCINOMA

What types of cells play a pivotal role in the cell-mediated immune response to primary TB>

STIMULATION BY CD4+ T LYMPHOCYTES ...langerhans giant cells are characteristic of granulomatous conditions, including the caseating granulomas associated with mycobacterium TB. Horseshoe appearance...the macrophages that form these giant cells are activated by CD4+ TH1 LYMPHOCYTES

Patient is clearly now lactose intolerant what has decreased (not an enzyme)

STOOL pH...patient likely has secondary lactase deficiency, an acquired condition resulting from inflammation/infection. Studies can show: INCREASED BREATH HYDROGEN CONTENT, REDUCED STOOL pH, ELEVATED STOOL OSMOLALITY

What is the most common benign vascular tumor in children?

STRAWBERRY (CAPILLARY) HEMANGIOMA. They occur during first weeks of life and grow very rapidly and typically regress by 5-8 years...composed of capillaries separated by connective tissue.

Patient comes in with numbness/tingling in both legs and difficulty walking....conjuctival pallor & loss of vibration and position sensation in bilateral lower extremities w/associated gait ataxia. What is a likely change in his life?

STRICT VEGAN DIET FOR THE PAST 6 MONTHS = vitamin B12 deficiency = COBALAMIN deficiency usually found in animal sources.

College kid brings his roommate to doctor office says he was fine until recently when he began staying up all night to study for finals (roommate thinks cops are after him and is sweating profusely) what's up?

SUBSTANCE-INDUCED PSYCHOLOGIC DISORDER most likely due to stimulant intoxication (paranoid, hypertensive...no history of psych issues)

A product that when injected into muscle of dystonia patients causes dramatic (yet temporary) effect...substance produced by bacteria that demonstrate what?

SUBTERMINAL SPORE FORMATION local botox injection, Clostridia are gram+ spore-forming anaerobic rods. C. Botulinum is the bacteria responsible for botulism a toxin-mediated disease.

Man has "pins&needles" sensation in legs + painful lesions lips/mouth. History of alcohol & IV use, hepatomegaly. Labs: LOW riboflavin, what enzyme activity is impaired?

SUCCINATE DEHYDROGENASE (man has riboflavin deficiency; riboflavin is a precursor of the coenzymes FMN & FAD. FAD participates in the TCA cycle and ETC by acting as an electron acceptor for succinate dehydrogenase (complex II), which converts succinate into fumarate.

Anesthia does a depolarizing block, phase 1 looks like normal....phase 2 is declining...what drug?

SUCCINYLCHOLINE ~ train-of-four stimulation is used furing anestheia to assess degree of paralysis induced by NMJ-blocking agents

Image of hypertrophic cardiomyopathy, question askes which activity would most likely increase the intensity of the patient's murmur?

SUDDEN STANDING, valsava(bearing down), or nitroglycerin administration decreases preload and will result in INCREASED MURMUR INTENSITY. manuevers that increase preload or afterload will decrease murmur intensity by increasing LVEDV and lessening the outflow tract.

What can be given to reverse cyanide toxicity (in the setting of too much nitroprusside given to a patient)

SULFUR DONOR(SODIUM THIOSULFATE), direct binding of cyanide ions (hydroxocobalmin), induction of methemoglobinemia(sodium nitrate)

Patient has IMA ligated, collateral circulation from what blood vessels are responsible for preventing left colon ischemia?

SUPERIOR MESENTERIC ARTERY via marginal artery of drummond

Patient has diplopia due to inability to abduct the eye and has lost corneal reflex, what anatomic structure (opening in skull) is to blame?

SUPERIOR ORBITAL FISSURE ~ The oculomotor nerve (CNIII) and opthamic nerve (CNV1)amonths, 4 & 6 and superior opthalmic vein venter the orbit via SUPERIOR ORBITAL FISSURE

Man has progressive pain beneath his right scapula and right arm for several months, associated with numbness in his right forearm extending into tips of 4th & 5th fingers. +Cough...what is the cause of his symptoms?

SUPERIOR SULCUS TUMOR = PANCOAST TUMOR: Shoulder pain, horner syndrome, spinal cord compression C8-T2 pain

Woman with a DVT has an embolectomy performed and post-procedure develops reperfusion injury due to reactive oxygen species, what enzyme is most likely to help neutralize the toxic molecules?

SUPEROXIDE DISMUTASE ~ acute compartment secondary to reperfusion injury...SUPEROXIDE DISMUTASE is an antioxidant and it neutralizes reactive oxygen species, preventing cell injury.

Student finished step 1 and is annoyed by everyone talking about difficulty and decides not to think about the exam or her performance until she gets her score. What defense mechanism?

SUPPRESSION ~ suppression is a mture defense mechanism involving a conscious choice not to dwell on a particular thought or feeling

Changes in host range are most commonly caused by mutations in what?

SURFACT GLYCOPROTEINS that mediate virion attachment to target host cell plasmalemma receptors.

woman as skin dimpling on the right breast and an irregular immobile mass in the right upper quadrant of the outer breast, where is the malignancy FROM?

SUSPENSORY LIGAMENT...invasive breast carcinoma typically presents as an irregularly shaped adherent breast mass, most commonly in the upper outer quadrant. Malignant infiltration of suspensory ligaments of the breast causes dimpling of the overlying skin.

What is the sinoatrial node closest to?

SVC....SA node specialized pacemaker cells, earliest site of electrical activation in patients with sinus rhythm

Patient has acute decompensated heart failure (reduced cardiac output and excessive ventricular filling pressures) what is most likely contributing causes

SYMPATHETIC NERVOUS SYSTEM INCREASED ACTIVITY

Ischemic hepatic cells in mice are studied, cells that have been ischemic for longer have reduced numbers of ribosomes attached to the ER...this structural change is most likley to impair what cellular functions?

SYNTHESIS OF CELL MEMBRANE PROTEINS ("attached ribosomes" bind to the RER after protein translation begins they synthesis most secretory proteins: proteins of the NUCLEUS & CELL MEMBRANE, proteins within the ER, GOLGI & LYSOSOMES). Ribosomes attach to the RER via the translocon

Murmur between S1 & S2, goes up then down...what's up?

SYSTOLIC crescendo-decrescendo murmur (aortic or pulmonic stenosis) #1 reason is degenerative calcification of aortic valve leaflets

Black patient with sickle cell pain crisis, blood culture shows: nonlactose-fermenting, oxidase-negative, motile organisms. Bug and virulence factor?

Salmonella (osteomyelitis) special CAPSULE = Vi antigen...Vi stands for virulence and it protects the bug from OPSONIZATION

What bone is commonly injured in falling on an outstretched hand...what is a common result of that fracture?

Scaphoid bone fracture ~ AVASCULAR NECROSIS (tenderness in anatomical snuffbox)

Relationship between lunate/scaphoid and falling on outstretched hand?

Scaphoid is more medial than lunate...Scaphoid (medial) is often fractured, lunate(lateral) is often dislocated.

Strawberry tongue...what SERIOUS complication can result from the disease.

Scarlet fever = strawberry tongue. Group A strep. Complication = RHEUMATIC FEVER & glomerulonephritis

Schistocytes (helmet cells) seen on histology are indicative of what? Levels of serum ____ will be low?

Schistocytes (helmet cells) are fragmented erythrocytes usually secondary to mechanical trauma from microangiopathic hemolytic anemias or prosthetic cardiac valves. INTRAVASCULAR HEMOLYTIC ANEMIAS ARE CHARACTERIZED BY DECREASED SERUM HAPTOGLOBIN LEVELS AS WELL AS INCREASED LDH & BILIRUBIN

Difference in drainage between scrotum & testes

Scrotum(superficial inguinal). Testes(PARA-AORTIC...testes started in abdomen and descended)

what tan/brown skin markings have a "stuck on" appearance?

Seborrheic keratosis (benign epidermal tumor that presents as a tan or brown, round lesion with a well-demarcated border & stuck-on appearance). Hyperkeratosis (thickening of stratum corneum) & keratin-containing cysts.

2 year old boy seen with giardia given metronidazole and improves from home, returns to doctor with more frothy loose stools...what is wrong? Enzyme?

Secondary lactase deficiency (can occur after inflammatory[celiacl] or infectious [giardia]) processess damage the microvilli of the small intestines.

Endometrial sampling reveals coiled glands filled with carbohydrate-rich mucus, edematous stroma, tortuous spiral arteries...at which point in cycle/graph was sample taken?

Secretory phase (second half of menstrual cycle days 15-28; progesterone released by the corpus luteum causes the uterine glands to coil and secrete glycogen-rich mucus in preparation for embryo implantation.

What does acute theophyline intoxication result in?

Seizures = major cause of death in acute theophyline toxicity [tachyarrthymias are another major concern]. Nauseua/Vomitting/Abdominal Pain/Diarrhea/Cardiac Arrthymias/ Seisures

What is a potential side effect of buproprion therapy (especially when given at high doses)

Seizures! [buproprion is an antidepressant with a structure similar to that of amphetamines....inhibiting reuptake of norepinephrine & dopamine] ...it is contraindicated in patients with seisure disorders, anorexia & bullemia nervosa.

An anti-biotic containing medium is a _____ type of medium?

Selective!

What is septic abortion and what is a common bug involved?

Septic abortion is any type of abortion that results in retained products of conception (fever, abdominal pain, foul-smelling vaginal discharge). Common pathogens are STAPH AUREUS OR E. COLI

Generalized anxiety disorder (excessive/chornic worry, feeling on edge, muscle tension, insomnia) best pharm option?

Serotonin Reuptake Inhibitors & Serotonin-Norepinephrine reuptake inhibitors are first-line medications for GAD (citalopram is an option)

Patient has OCD (fear of touching keyboards shared by colleagues) what neurotransmitter is the target for therapy?

Serotonin. Patient has OCD and will need an SSRI (works by blocking the reuptake of serotonin into the presynaptic neuron

What two characteristics are MAJOR in neoplastic poylps?

Serrated & adenomatous (tubular, villous, tubulovillous) . VILLOUS ARE MORE LIKLEY TO UNDERGO MALIGNANT TRANSFORMATION THAN TUBULAR ADENOMAS

Midaxillary line, 5th intercostal space...what muscle must be pierced?

Serratus Anterior

What do sertoli cells do, how would a male without them present?

Sertoli cells produce inhibin in response to FSH from the anterior pituitary...inhibin suppresses FSH production in the pituiatry, impaired sertoli cell function would lead to decreased production of inhibin, increased FSH levels & impaired fertility

Paitent having liver problems and unprotected sex with multiple female parters and never got vaccinations?

Serum HBsAG (sexual activity and IV drug use) ...risk of HepC transmission via sex is VERY LOW

What is the most sensitive marker for diagnosis in hypothyroidism?

Serum TSH (although TSH is not elevated in patients with central hypothyroidism, this form of hypothyroidism is uncommon and usually occus in patients with hypothalamic or pituitary dysfunction

Research: knocked out leptin receptor what effect will it have on BMI and serum leptin levels?

Serum leptin levels will increase, BMI will increase. Leptin decreases neuropeptide Y (which stimulates appetitie)...Leptin stimulates POMC (fullness). Leptin acts on the ARCUATE NUCLEUS OF THE HYPOTHALAMUS...mutations in leptin gene or receptor = HYPERPHAGIA & OBESITY

What serum marker reflects the activity of osteoblasts?

Serum level of bone-specific alkaline phosphatase. Bone-specific alkaline phosphatase reflects osteoblastic activity

What is the SPIKES protocol for delivering bad news?

Set up stage, Perception (open-ended questions), Invitation (ask what they want to know), Knowledge (warn before dropping bad news), Empathy, Summary/Strategy

How do serum calcium and parathyroid hormones look in someone who has primary osteoporosis?

Seurm calcium, phosphorus & PTH levels are typically normal in patients with primary osteoporosis.

What is major depressive disorder with psychotic features?

Severe subtype of unipolar major depression (MDD symptoms + presence of delusions (responsible for all evil in the world) and/or auditory hallucinations)

Relationship between sex hormones and bone? (prepuberty)

Sex hormones promote BOTH bone growth & epiphyseal plate closure...precocious puberty may result in a shorter stature (despite an initial growth spurt)

Major side effect of SSRIs?

Sexual dysfunction

Depending on age of patient and condition, what bacteria only needs about 10-500 bacteria present to cause disease?

Shigella

What are meglitinides?

Short-acting glucose lowering medications...functionally similar to sulfanureas and act by binding to and closing ATP-dependent K+ channels in the pancreatic beta cell membrane including depolarization and L-type calcium channel opening [increased Ca2+ influx stimulates beta cell insulin release]

Triad of ECG abnormalities seen in Wolf-Parkinson-White syndrome?

Shortened OR interval, a delta wave at the start of the QRS complex and a widened QRS interval. Wolff-Parkinson-White syndrome = Accessory AV conduction pathway. On graph it will be from p wave into qrs

Cardiac catheritization requires either the femoral artery or the radial artery, what happens when there is arterial puncture above the inguinal ligament?

Significant increase in risk of retroperitoneal hemorrhage

How do simvastatin & cholestyramine work together to reduce plasma LDL?

Simvastin inhibits HMG-CoA reductase (inhibiting cholestol synthesis), while cholestyramine INCREASES hepatic cholesterol & bile acid synthesis..but the effect can be blocked by adding a stain drug and net rxn = reduced LDL

Man gets new kidney from his sister, takes medication that blocks IL-2 signal transduction. What drug?

Sirolimus (binds FK-506 binding protein, inhibits mTOR, this leads to an interruption of IL-2 signal transduction...preventing Gi to S phase progression and lymphocyte proliferation)

Why is the effect on verapamil not as significant for skeletal muscle as it is smooth/cardiac?

Skeletal muscle has LITTLE DEPENDENCE ON EXTRACELLULAR CALCIUM INFLUX. The L-type calcium channels are directly in contact with the RyR calcium channels. So skeletal muscle is resistant to CCB like verapamil because for skeletal muscle the calcium release is from a MECHANICAL interaction, not ca-induced-ca-released like smooth and cardiac.

What are the two most significant risk factors for developing esophageal squamous cell carcinomas in US?

Smoking and Drinking alcohol (keratin pearls = squamous cell carcinomas)

What is by far the most effective preventative intervention in almost all patients?

Smoking cessation (especially those with diabetes)

38 year old woman wants to take birth control, what would be most important factor is making the decision?

Smoking status [risk of endometrial cancer due to unopposed effect of estrogen]

patient preoccupied with unexplained medical symptoms and excessive health care usage. What is wrong? What is best approach?

Somatic symptom disorder - patient is preoccupied with unexplained medical symptoms & excessive health care use. Best approach is regular visits with PCP with the goal of functional improvement. UNNECCESARY DIAGNOSTIC TESTS & SPECIALIST REFERRALS SHOULD BE AVOIDED

Explain severe allergic asthma and what is the best tx?

Some asthmatics have frequent allergies as a trigger due to a high IgE response/sensitivity in the body. ANTI IGE ANTIBODIES can be given as a subcutaneous injection OMALIZUMAB.

Patient with paroxysmal A fib, treated via rate/rhythm control...drug gives patient torsades de point..what drug is being taken?

Sotalol [class 3 antiarrythmic = K+ channel blocking]. Torsades due to QT interval prolongation.

Informed consent process: It is essential that the patient FULLY understands what they are agreeing to. If a physician tries to vaguely explain procedure what should resident do?

Speak privately to the physcian about using a trained Viatnemese-English translator

True Negatives =

Specificity * (number of patients without the disease)

Patient in motor vehicle accident, had spleen removed then got strep pneumo and died, why?

Spleen acts as both a FILTER and a major site of opsonizing antibody synthesis...asplenic patients are prone to infection by: Strep Pneumo, H. Influenzae, N. Meningitidis

6 month old ery weak/delayed motor development, he can barely lift his head up when prone and unable to roll to side. Mutation of a protein involved in assembly of snRNPs, what cellular element is impaired?

Spliceosomes ~Spinal muscular atrophy~SMN1 gene(survival motor neuron 1) = defective snRNP = impaired SPLICEOSOME FUNCTION

Man comes in with lesions on his arm they are NOT PAINFUL, no fever and testing confirms fungal origin...whats up?

Sporothrix Schnekki (rose-gardeners disease) - dimorphic fungi that causes subcutaneous mycosis (often transmitted via thorne prick) [nodules spread along lymphatics]

Warfarin can have a significant interaction with what drug potentially leading to an thromboembolic stroke via inhibition of Warfarin's actions?

St. John's Wort (CytP450 INDUCER) makes liver work harder breaking down Warfarin faster = clotting = stroke. St. John's Wort (over the counter medicincal herb best known for anti-inflammatory and antidepressant properties but it INDUCES cytp450 and shouldn't be given with Warfarin

Kid gets a ventriculoperitoneal shunt places, coagulase negative staph grows...what virulence mechanism does this bug use?

Staph Epidermitis, uses SYNTHESIS OF AN EXTRACELLULAR POLYSACCHARIDE MATRIX = biofilm = common cause of foreign body infections.

10 year old dull pain above knee, point tenderness with soft tissue swelling & periosteal reaction over the low end of the femur, what organism is responsible?

Staph aureus = HEMATOGENOUS OSTEOMYELITIS IS PREDOMINANTLY A DISEASE OF CHILDREN EFFECTING LONGBONES #1staph aureus...#2groupAstrep

What bug causes right-sided endocarditis in IV drug users?

Staph aureus = acute right sided bacterial endocarditis

Woman hospitalized with high-grade fevers, chills & right upper abdominal pain...elevated alkaline phosphatase, fluid-filled cavity within right lobe of liver...what bug?

Staph aureus, by hematogenous route (presence of a fluid-fileld cavity in liver + fevers, chills, right upper abdominal pain is suggestive of hepatic abcess

Relationship between statin drugs and decreased cholesterol level?

Statins inhibit HMG-CoA reductase, the rate limiting step in cholesterol synthesis...this then requires hepatocytes to increase their surface expression of LDL receptors and take up more LDL = huge reduction in cholesterol and LDL concentration.

What is status epilepticus and what is firstline?

Status epilepticus: tonic-clonic seizures, fever/flyu-like symptoms. Firstline: GABA enhancers like benzodiazepines given IV (lorazepam). They work by acting on GABA-A receptor causing in influx of negatively charged chloride ions in response to GABA binding. =hyperpolariziton/no more firing

What is the most common cause of community acquired pneumonia (fever, chills, productive cough) in the immunocompromised?

Strep Pneumo

Patients over 65 are at risk of developing seconardary bacterial pneumonia after influenza, list the order of pathogens most often responsible:

Strep Pneumo, Staph Aureus, Haemophilus Influenzae.

Man with severe leg pain after minor laceration, leg is swollen, firm, erythema, draining blood-tinged exudate. Bug is Gram+/wide zone of hemolysis/-coag/-catalase/+PYR pyrrolidonyl arylamidase what bug?

Strep pyogenes (pyr + = bacitracin sensitive, wide zone = beta hemolysis)

Pressure-volume curve moves to the left and up, what parameter is likely to increase?

Stroke volume

Painful thyroid enlargement following a viral illness with lymphocytic infiltrate with macrophages & multinucleated giant cells?

Subacute Granulomatous (de quervain) thyroiditis

Clinically, an abrupt/severe headache is characteristic of what?

Subarachnoid Hemorrhage "worst headache of my life" = SACCULAR (BERRY) ANEURYSM.

Patient found relief with capsaicin cream, decreased activity of what neurotransmitter is most likely associated with pain relief?

Substance P

What muscle relaxer can cause signficant potassium release and life-threatening arrhythmias in patients at risk for hyperkalemia?

Succinylcholine can cause signficant posttium release and life-threatening arrhythmieas in patients at risk for hyperkalemia, including those with burns,myopathies, crush injuries & denervating injuries or disease.

Lethargic and vomitting (otherwise healthy) 6 month old has ALDOLASE B deficiency [hereditary fructose intolerance], what should be avoided in diet?

Sucrose/Fructose...patients will be hypoglycemic and vomit in presence of either...tx strict removal of sucrose & fructose from diet

Emergency cricothyrotomy...what layers will incision pass through?

Superficial cervical fascia and cricothyroid membrane...cricothyrotomy is indicated when an emergency airway is required. Passes thru: SUPERFICIAL CERVICAL FASCIA, PRETRACHEAL FASCIA, CRICOTHYROID MEMBRANE

most of the cutaneous lymph from the umbilicus down, including the anus below the dentate line, drains where?

Superficial inguinal lymph nodes (exceptions: glans penis & posterior calf they go to deep inguinal lymph nodes)

The spermatic cord enters through the deep inguinal ring an opening in the transversalis fascia, what does it exit through?

Superficial inguinal ring, which is an opening in the external abdominal oblique muscle aponeurosis. Surgical repair involves reducing testes through the superficial inguinal ring.

Hemorrhoids, portal vs systemic contribution?

Superior Rectal Vein(portal) Middle/Inferior rectal veins(systemic)

Kid has seizure (previously had rough, fever 2 days); appears tired but is alert and cooperative...best next step?

Supportive care only (since he had a fever seizure and not a heat stroke, there is no need for cooling...if he had head stroke then we would want cooling).

What hypothalamic nucleus regulates circadian rhythm?

Suprachiastmic hypothalamic nucleus regulates circadian rhythm & pineal gland function

Synthesis of daughter strands occurs simultaneously but since dna synthesis occurs in the 5'->3' direction there will be a leading (same direction) and lagging (opposite direction) what is unique about lagging daughter strand

Synthesis of multiple, short DNA fragments...Okazaki Fragments, that are separated by many RNA primers.

Fixed segmental loss of upper extrimity pain & temperature sensations, upper extremity lower motor neuron signs and/or lower extremity upper motor neuron signs + Scoliosis =

Syringomyelia

Patient is HIV negative with CMV antibody IgG positive without clinical evident illness...how?

System MONONUCLEOSIS-like SYNDROME...it will be monospot negative [if patient was immunocompromised (HIV) then there would be normal: retinitis, pneumonia, esophagitis, colitis, hepatitis]

Candida extract is injected intradermally into a kid and 48 hours later has has firm nodule measuring 16mm, what kind of cell is responsible for the rxn?

T-LYMPHOCYTES: Contact dermatitis, granulomatous inflammation, tuberculin skin test, candida extract skin rxn are all examples of delayed-type hypersensitivity (DTH) cell that mediate DTH are Th1-lymphocytes that release INF-gamma to cause recruitment and stimulation of macrophages. DTH rxns take days to reach peak activity

What kind of reaction happens after poison ivy/oak/sumac URUSHIOL contact?

T-LYMPHOCYTES: POISON IVY(linear patterns); Contact dermatitis, granulomatous inflammation, tuberculin skin test, candida extract skin rxn are all examples of delayed-type hypersensitivity (DTH) cell that mediate DTH are Th1-lymphocytes that release INF-gamma to cause recruitment and

Large amount of IFN-gamma, what cell types are responsible for this finding?

T-lymphocytes...IFN-gamma activates macrophages, increases MHC, promotes Thelper1 lymphocyte differentiation. IFN-gamma release assays test for latent tuberculosis infection by measuring the amount of IFN-gamma released by T lymphocytes when exposed to antigens unique to Mycobacterium Tuberculosis

What are the 3 forms of circulating Thyroid Hormone?

T4 (thyROXine =major secretory product of thyroid gland). T3(triiodothyronine = MOST ACTIVE form of thyroid hormone arises from T4 deiodination) rT3(reverse T3 inactive form from T4)

man with advanced prostate cancer with bone metastases has midly tender enlargement of breast tissue behind nipple-areolar complex, what med could have PREVENTED this patient's problem?

TAMOXIFEN = selective estrogen receptor modulator inhibits the effects of estrogen on breast tissue and is effective for prevention and treatment of gynecomastia in patients on ADT

What is the TATA box and where is it

TATA box is -25 (after CAAT box) it is a transCRIPTION promotor (caat tata crip)

What two factors are the big dogs in wound healing?

TGF-B [critical for fibroblast migration, proliferation & connective tissue synthesis] & PDGF (platelet derived growth factor). KELOID = INCREASED TGF-B

Woman comes in for annual check-up breaks down and cries during sexual history portion because she was abused, what is the appropriate response?

THAT SOUNDS LIKE AN AWFUL EXPERIENCE, ARE YOU COMFORTABLE TALKING ABOUT IT?

What structure contains the ovarian artery, vein, nerve and lymphatics and should be ligated during te removal of one of the ovaries?

THE SUSPENSATORY LIGAMENT OF THE OVARY = fold of peritoneum that attaches the ovary to the pelvic sidewall and contains: OVARIAN ARTERY, OVERIAN VEIN, LYMPHATICS, NERVES

E coli is grown on lactose-containing medium, upregulating the production of B-galactosidase & galactosidase permease....how/why both?

THERE IS ONE MRNA CODING FOR BOTH ENZYMES ...bacterial mRNA can be POLYCISTRONIC (where 1 mRNA codes for several proteins...lac operon is an example)

Woman with small bluish lesion nder the nail of her right index finger, lesion is extremily tender to touch...if it is a tumor what function do its cells of origin serve?

THERMOREGULATION...a bluish neoplasm under the nail bed is either a glomus/glomangioma tumor or a subungual melanoma...both pretty rare. Turmor originates from the SMOOTH MUSCLE CELLS THAT CONTROL THERMOREGULATORY FUNCTIONS OF DERMAL GLOMUS BODIES.

Infant with "carmelizing sugar" smelling diapers, tonic posturing poor feeding, vomitting and irritability...what supplementation may improve the condition?

THIAMINE ...branched chain alpha ketoacid dehydrogenase requires several coenzymes: THIAMINE, lipoate, coenzyme a, FAD, NAD (tender loving care for nancy). Kid has maple syrup urine disease (defective breakdown of leucine, isoleucine, valine)

Image of nephron, which part is IMPERMEABLE to water?

THICK/THINK ASCENDING LOOP...impermeable to water regardless of serum vasopressin levels.

Fever & sore throat in a patient with Grave's disease(hyperthyroidism) treated with medical therapy should raise concern for what?

THIONAMIDE-INDUCED AGRANULOCYTOSIS [next step in management = WHITE BLOOD CELL COUNT WITH DIFFERENTIAL]...sudden onset fever and sore throat should raise concern for agranulocytosis

Lab animals deprived of folic acid experience increased erythocyte precursor cells, the immature cells die...what could be given to reduce this apoptosis?

THYMIDINE. Thymidine supplementation can moderately increase dTMP levels so it can reduce the erythrocyte precursor cell apoptosis.

What is the best description of the function of thyroid peroxidase enzyme?

THYROGLOBULIN IODINATION ~ thyroid peroxidase is responsible for the oxidation of iodide to iodine, the iodination of thyroglobuln tyrosine residues and the iodotyrosine couplinf reaction that forms T3 & T4. Autoantibodies against thyroid peroxidase are common in autoimmune hashimoto thyroiditis

Zinc fingers made up of mostly CYSTEINE (sometimes HISTIDINE) serve as a receptor for what hormone?

THYROID HORMONE ~ zinc finger motifs are chains of AA (cysteine/histidine) around a zinc atom that recognize specific DNA sequences and are used by many transcription factors to bind DNA and alter activity of target genes. Intracellular receptors that bind steroids, THYROID HORMONE, and fat-soluble vitamins act directly as transcription factors and contain zinc-finger binding domains.

Man comes to er for painful muscle cramps, had tingling sensation around his mouth since earlier that day with intermittent sensation of choking in throat....on exam appears comfortable but ancious, light tapping anterior to ear iliits twitching of PERIORAL MUSCLES what is the cause?

THYROID SURGERY = HYPOCALCEMIA...muscle cramps/parasthesias...neuromuscular hyperexcitability. Chvostek sign (facial muscle contraction elicited by tapping on facial nerve anterior to the ear). Trousseau sign (triggered by BP cufff)

Teen boy presents with right knee pain worsened by extending leg, where is the likely insertion side of the involved muscle tendon?

TIBIAL TUBEROSITY [the quadriceps muscle group is connected to the patella, which in turn is attached to the tibial tubercle by the patellar ligament. Repetitive quad contraction can lead to Osgood-Schlatter = focal pain and swelling at tibial tuberosity

Patient takes propofol during procedure and he becomes clinically alert several minutes after administration what mechanism explains how this happens?

TISSUE REDISTRIBUTION OF THE DRUG. Propofol and other highly lipophillic drugs readily diffuse across membranes quickly accumulating in tissues receiving high blood flow...short duration of action.

When injury occurs local neutrophils & macrophages release what? (3)

TNF-alpha, IL-1, IL-6. Erythrocyte sedimentation rate (ESR) is a non-specific marker of inflammation,

Immigrant 8 year old has low-grade fever and skin rash...rash started on face and spread down body, PE: pinkish maculopapular rash and tender lymphadenopathy bilaterally behind the ears....what is the cause?

TOGAVIRUS...RUBELLA...POSTAURICULAR AND OCCIPITALLYMPHADENOPATHY

Why do patients taking daily maintenance nitrates only do 2 doses a day and not 3+

TOLERANCE DEVELOPMENT ...they need to have a nitrate-free period every day to avoid tolerance to the drug.

A mutation in WHAT causes malginant transformation of adenoma into a carcinoma?

TP53 ~ tumor suppressor gene. Normal mucosa -> polyp/adenoma[APC]. Adenoma/polyp increasing in size[KRAS]. Malignant transofmration of adenoma into carcinoma[p53]

Pregnant mom has type 1 diabetes, baby delivered by c-section what metabolic disturbance is most likely to develop in the neonate?

TRANSIENT HYPOGLYCEMIA ~ neonates born to mothers with poorly controlled diabetes during pregnancy are exposed to high maternal glucoselevels in utero resilting in excessive detal insulin production and islet hyperplasia. [ONLY LASTS FOR SEVERAL DAYS]

What is usually the last step in signal transduction systems (ie MAP-kinase, P13K/Akt/mTOR pathway)...

TRANSLOCATION TO THE NUCLEUS AND GENE TRANSCRIPTION...mutations in growth factor receptors (Akt, mTOR, PTEN) that enhance the pathway contribute to cancer pathogenesis

What is the responsibility of the PECAM-1 gene in neutrophil function

TRANSMIGRATION (PECAM-1 = TRANSMIGRATION; ICAM-1 = tight adhesion)

Abdominal pain, periodic diarrhea,low-grade fever. ENTEROENTERIC FISTULA, colonoscopy shows "cobblestone" mucosa that has linear ulcerations with "skip areas" of normal bowel. Why is there a FISTULA?

TRANSMURAL INFLAMMATION ~ Crohn's disease causes TRANSMURAL INFLAMMATION of any area of the GI tract. The involvement of all layers of the interstinal wall explains the most common complications of Crohn's disease: strictures and fistula formation.

Pressure-volume curve has both axis increased, in a 72 year old man with fatigue and exertional dyspnea, PE: bibasilar crackles, elevated JVP, edema...what is the likely cause?

TRANSTHYRETIN DEPOSITION ~ Diastolic heart failure, caused by decreased ventricular compliance...normal LV ejection fraction, normal LV end diastolic volume....elevated LV filling pressures. Caused by: Hypertension, obesity, infiltrative disorders (transthyretin amyloidosis, sarcoidosis)

Left shoulder droop and weakness of abduction above horizonal position, in a patient who recently underwent posterior triangle surgery. WHAT MUSCLE IS PARALYZED?

TRAPEZIUS ~ spinal accessory nerve innervates trapezius and in vulnerable to injury in the posterior triangle of te neck. Injury can result in weakness and winging of the scapula

Chronic alcoholic admitted to hospital for chest pain (obviously cannot get alcohol in hospital) what will be the first sign of his withdrawal?

TREMULOUSNESS is typically the first sign of alcohol withdrawal (also GI, anxiety, agitation, autonomic problems...delirium is most severe and begins 48-96 hours after last drink)

IV drug use + wedge-shaped hemorrhagic lesions on lung...how did person die?

TRICUSPID VALVE ENDOCARDITIS

What are LOW serum alpha-fetoprotein levels associated with

TRISOMY 21 ~ low AFP, High B-hCG, High Inhibin A

Man has vision problems walking DOWN the stairs, seems to see double. No history of head trauma or other eye issues...only difficulty with stairs and reading sometimes. What nerve?

TROCHLEAR NERVE PALSY ~ Superior oblique palsy, patients may compensate by tucking the chin and tilting the head away from the affected eye

The contractile mechanism in skeletal muscle depends on what proteins?

TROPONIN, myosin II, actin, tropomyosin

man has vomitting and abdominal pain that radiates to the back and improves by bending forward...tenderness over epigastrium and decreased bowel sounds. Ab CT shows pancreatic enlargment with areas of necrosis. What is being inappropriately activated?

TRYPSINOGEN ~ acute pancreatitis...Trypsin normally converts zymogens to their more active form in the duodenal lumen. Premature cleavage of trypsinogen to trypsin within the pancreas leads to uncontrolled activation of these zymogens causing pancreatic autodigestion and acute pancreatitis

Explain relationship between T3, T4 & TSH

TSH produces T4, which is converted to more active T3...presence of T3 inhibits TSH/T4 production

Woman is being started on entanercept what test should be performed before tx begins?

TUBERCULIN SKIN TEST TNF-alpha inhibitors impair cell-mediated immunity. All patients beginning tx with TNF-alpha inhibitors should be evaluated for latent TB

Old man with progressive weakness, fatigue, anorexia & 22lb weight loss over past 4 months. What is most likely contributing to:anorexia, malaise, anemia, weight lost and general wasting?

TUMOR NECROSIS FACTOR-alpha (TNF-a) = CACHEXIA. TNF-a is thought to mediate paraneoplastic cachexia in humans by suppressing appetite and increasing basal metabolic rate.

What is the best statistical method to compare two groups of subjects?

TWO SAMPLE T TEST = compare means of 2 groups of subjects

Adopted kid has MUSTY ODOR, intellectual disability and suffers from seizures what amino acid is essential?

TYROSINE ~ PKU results from inability to convert phenylalanine to tyrosine by the phenylalanine hydroxylase system...making tyrosine an essential amino acid in these patients. Classical clinical features of untreated pPKU = intellectual disability, seizures, light pigment & musty odor

Strep pneuoniae is able to undergo transformation, allowing bacterium to do what?

Take up exogenous DNA fragments and express the encoded proteins. This is how strains of S Pneumoniae that do not form a capsule can acquire the genes that code for the capsule and therefore gain virulence.

Patient with appendicitis, what anatomical structure can be used to guide surgery?

Tenai Coli band [3 separate smooth muscle ribbons that travel longitudinally on the outside of the colon and converge at the root of the veriform appendix...look for appendix at cecal base = tenaei coli origin

When would a male have a high level of hCG?

Testicular malignancy that can secrete hCG (peptide hormone normally produced by the placenta). hCG is structurally very similar to TSH...patients with testicular germ cell tumors or gestational trophoblastic disease may develop high serum hCG concentrations, which can stimulate TSH receptors causing hyperthyroidism.

Sexual development is controlled by 3 hormones what are they?

Testosterone: internal male genitalia, spermatogenesis, male sexual differentiation at puberty (muscle/libido). Dihydrotestosterone: external male genitalia, growth of prostate, male-pattern hair growth (also amplifies testosterone. Estrogen: endometrial proliferation, development of ovarian granulosa cells, breast development.

Cyanotic spells that improve with squatting, RV impulse, systolic murmur...what is the this? What is deviated in this?

Tetralogy of Fallot, anterior & cephaladad deviation of the infundibular septum during embryo development [VSD w/overriding aorta] = RV outflow obstruction (SQUATTING INCREASES PVR [AFTERLOAD] DECREAES R->L SHUNTING)

Right-sided pure hemisensory loss, location of stroke?

Thalamic Stroke (VentralPosterior thalamic stroke) the vpl recieves input from the spinothalamic tract and dorsal columns. The vpm recieves input from the trigeminal pathway and sends somatosensory projections to the cortex. Damage to these may result in complete contralateral sensory loss and problems with gait.

What does it mean if the 95% confidence interval does not contain 0?

That the results were statistically significant. So a meta-analysis with results showing a 95% confidence interval of [-2.7, -1.3] that does NOT contain zero or the Null Hypothesis means that the results were statisticlaly significant.

What is thayer-martin used to isolate? What is thayer-martin made up of?

Thayer-martin is used to isolate Neisseria species from culture. It is a chocolate (heated blood) agar that contains vancomycin(to kill gram +), collistin(to kill gram -), nystatin(to kill fungi), trimethoprim(to inhibit growth of other gram -)

What is the significance of the "mode" in biostats?

The MODE is the most frequently seen value. So adding an outlier to a data set is less likely to impact the mode (modes tend to be only thing resistant to outliers)

Professor creates a model with two balloons one small and one big but they both have identical surface tension, what will happen when the clamp (closed between them) opens?

The SMALLER BALOON WILL COLLAPSE

Explain the candidal antigen skin test, what are the key cells involved?

The candidal antigen skin test is used to determine the presence of cellular, or T cell-mediated, immunity through detection of delayed-type hypersensitivity rxn (TyPE 4). Key Cells: MACROPHAGES, CD4+T CELLS, CD8+T CYTOTOXIC T CELLS).

Where is the cell attachment site for the immunoglobulin molecule?

The carboxyl terminal of the Fc portion of the heavy Ig chain...antibody bound to antigen leads to signal for apoptosis (see picture In phone)

Which concentrations increase and which decrease as you move along the proximal tubule?

The concentrations of PAH, creatinine, inulin & urea increase as fluid runs along the proximal tubule. Concentrations of bicarbonate, glucose & amino acids decrease

If a substance is reabsorbed from the nephron lumen then the net renal excretion rate...

The net renal excretion rate of the substance will be equal to its filtration rate minus the total tubular reabsorption rate

Given an image where the aorta pressure is lower than normal during systole compared to left atrium, where on picture corresponds to peak murmur intensity?

The point where th left atrium pressure is the highest and where aorta pressure should be

Different types of Wernicke-Korsakoff & what vitamin is deficient?

Thiamine deficiency causes beriberi as well as Wernicke-Korsakoff syndrome (confusion, ataxia, oculomotor abnormalities & permanent memory deficits) Dry beriberi = symmetrical peripheral neuropathy; wet beriberi includes addition of high-output congestive heart failure.

Old man taking a diuretic, hypercalcemia is seen shortly after...what kind of diuretic? Where does it work?

Thiazide diuretics work at the distal convoluted tubule and can cause HYPERcalcemia, HYPERuricemia, hyponatremia & hypokalemia. They work by blocking Na-Cl symporters in the distal convoluted tubules [causing sodium, chloride & water excretion]

What effect to thiazide diuretics have on metabolites, where do thiazide diuretics work?

Thiazide diuretics work at the distal convoluted tubule and can cause HYPERcalcemia, HYPERuricemia, hyponatremia & hypokalemia. They work by blocking Na-Cl symporters in the distal convoluted tubules [causing sodium, chloride & water excretion]. HYPERcalcemia leads to suppression of parathyroid hormone

Acanthosis Nigrans (commonly occurs in flexural areas) presents how (visibly) and what is the cause?

Thickening and hyperpigmentation (underarms other flexural areas) ...classic "velvetey" texture. Often associated with insulin-resistant states (diabetes mellitus, acromegaly, obesity) could also be sign of GI MALIGNANCY

Acute heptatits in postoperative period?

Think ingested halogenated inhaled anesthestics (halothane hepatitis) ~ DESFLURANE

Patient wih recurrent vaginal candida infection, weight loss, lymphadenopathy + thrush ...likely has what? What malignant B-lymphocyte proliferation is associated?

This patient with recurrent vaginal candida infection likely has HIV. Latent EPSTEIN-BARR VIRUS infection is present in up to 90% of normal individuals...reactivation is common in immunosuppressed.

Clostridium Dificile (normal 2% in gut flora) has two toxins, explain their actions

ToxinA(enterotoxin...interstinal inflammation/fluid secretion) & Toxin B(cytotoxin-cytotoxic) both toxins inactivate Rho-regulatory proteins involved in signal transduction & actin cytoskeletal maintenance

Infant presenting (after seemingly normal birth) with excessive secretions & choking/cyanosis during feeds....

Tracheoesophageal fistula with esophageal atresia = failure of the primitive foregut to appropriately divide into separate trachea and esophageal structures.

Gross hematuria in an elderly man who worked at a rubber company for 30+ years

Transitional cell carcinoma of the bladder. [occupation exposures or tobacco smoke]

An echogram showing the aorta lying anterior and to the right of the pulmonary artery is diagnostic for what? What part of formation got messed up?

Transposition of the Great Arteries (TGA incompatible with life unless there is a PDA) Defective SPIRALING

24 year old man comes into the emergency room with a painful erection for the past 6 hours, PMH significant for depression, OCD. What med caused this>

Trazadone (a sedating antidepressant) often used to treat insomnia, can rarely result in PRIAPISM (persistent erection for 4+ hours)

Primary -> Tertiary Syphillis

Treponema pallidum...primary syphillis will have a PAINLESS ulceration with raised borders (CHANCRE), secondary syphillis has a bacteremic stage 5 weeks later with a diffuse macular rash (CONDYLOMATA LATA) large gray wartlike growths on genitals. Late(tertiary) GUMMAS

What 3 heart things are associated with holosytolic murmurs?

Tricuspid regurg, Mitral regurg & VSD

How does tricyclic antidepressent toxicity present how is it reversed?

Tricyclic depressant overdose can present with mental status changes, seizures, prolonged QRS duration, ventricular arrythymias & anticholinergic findings. SODIUM BICARB IS USED TO REVERSE = INCREASE SERUM PH & EXTRACELLULAR SODIM

Patient with episodic facial pain triggered by shaving or washing face, how does medicine given help?

Trigeminal Neuralgia 1st line is CARBAMAZEPINE a neuroleptic that inhibits neuronal high-frequency firing by REDUCING ABILITY OF SODIUM CHANNELS TO RECOVER FROM INACTIVATION. Carbamazepine can also cause bone marrow suppression

Patient has serotonin syndrome, what is the amino acid serotonin comes from?

Tryptophan

Neonate with diminshed femoral pulses, lymphedema & cystic hygromas..what disease? What happened?

Turner Syndrome 45,X...loss of PATERNAL X CHROMOSOME

Type 1 vs type 2 muscle, which type is found on paraspinal muscles?

Type 1 = slow twitch (low level sustained force, postural maintenance...aerobic metabolism, high myoglobin (oxygen storage) & mitochondria. Type 2 = rapid/forceful pulses of movement (2b = anaerobic glycogenolysis & subsequent glycolysis). 2a = fast twitch (atp via aerobic metabolism)

Type of collagen seen in the heart (scarred/post-MI) is what type? Where else is it seen?

Type I (bone, tendon, dermis, ligaments, dentin, cornea, blood vessels, SCAR TISSUE)

What type of interferons are synthesized by most human cells in response to viral infections?

Type I interferons (alpha & beta) are produced by most human cells in response to virus, they help suppress viral replication by HALTING PROTEIN SYNTHESIS

Experiment where lung alveoli are exposed to NO2 there is massive necrosis but eventually some regeneration, what cells are responsible for regeneration?

Type II Pneumocytes

Patient is found to have cogwheel ridigity, resting tremor, bradykinesia...loss of function gene that leads to accumulation of misfolded proteins, what biochemical process is messed up?

UBIQUITINATION [ubiquitin is a protein found in eukaryotic cells that undergoes ATP dependent attachment to other proteins labeling them for degradation...accumulation of misfolded proteins can lead to parkinson's or alzheimer's].

Patient has all sign and symptoms of potential syphillus infection (painless ulcer) what diagnostic test is most useful in diagnosing Treponema Pallidum?

ULCER EXUDATE MICRSCOPY WITH DARK-FIELD ILLUMINATION will reveal HELICAL MOTILE SPIROCHETES.

Baby born with flaccid lower extremities and absent bilateral ankle reflexes...x-ray shows poorly developed lumbar spine and sacrum..mom blames herself, what factor related to baby's condition?

UNCONTROLLED DIABETES...CAUDAL REGRESSION SYNDROME (rare) sacral agenesis causing lower extremity paralysis and urincary incontinence...poorly controlled maternal diabetes

What is above and below the uterine artery as it makes its course?

UNDER = INTERNAL ILLAC ARTERY, above = uterine artery (uterine artery crosses over the anterior surface of the ureter "water under the bridge"

How is the hepatitis commonly transferred in pregnant women spread?

UNENVELOPED RNA VIRUS ~ Hep E virus is spread fecal-oral and the most concerning feature of Hep is HIGH MORTALITY RATE OBSERVED IN INFECTED PREGNANT WOMEN

Woman with polyuria is given desmopressin and urine output decreases and urine osmolality increases...renal clearance of what substance would decrease the most after the injection?

UREA ~ vasopressin and desmopressin cause a V2 receptor-mediated increase in water & urea permeability at the inner medullary collecting duct. The resulting rise in urea REABSORPTION (decreased urea clearance) enhances the medullary osmotic gradient, allowing the production of maximally concentrated urine.

What is the most common cause of unilateral hydronephrosis? (what site is the cause of the obstruction)

URETEOPELVIC JUNCTION. Inadequate canalization of the ureteropelvic junction, the connection site between the kidney and the ureter, it is the most common cause of unilateral fetal hydronephrosis

What specific anatomy isnt working correctly during stress incontinence?

URETHRAL SPHINCTER DYSFUNCTION ~ stress incontinence is due to a loss of pelvic floor support and incompetence of the urethral sphincter.

Patient suspected to be illiterate what is the best course of action trying to get through to the patient?

USE A VISUAL AID TO EDUCATE THE PATIENT ABOUT THE PROCEDURE (alternate learning methods such as visual resources should be used to address the problem.

Old lady comes to busy hospital with pneumonia and is started on empiric therapy, she declines and has to be sent to ICU. Her sputum sample was taken early in her admission but was never analyzed by team (to get more specific meds) what could have prevented this?

USING STANDARDIZED PATIENT HANDOFFS ~ communication failures a major cause of medical errors and can be reduced by use of a structured process.

What is treacher-collins syndrome?

Underdevelopment of 1st (trigeminal nerve, mandible, maxilla, zygoma, incus, malleus) & 2nd (facial nerve, stapes, styloid process, lesser horn of hyoid) pharyngeal arches....resulting in hypoplasia of mandibular & zygomatic bones.

What anticoagulant is most effective in inactivating thrombin?

Unfractionated heparin. Both unfractionated heparin & LMWH can bind to antithrombin to increase its activity against factor 10a. But ONLY UNFRACTIONATED HEPARIN is able to bind to both antithrombin and thrombin to to allow antithrombin to inactivate thrombin,

What is a mallory weiss tear?

Upper GI mucosal tear caused by forceful vomitting can result in a METABOLIC ALKALOSIS

What is a rare but serious complication of pelvic surgery (hysterectomy) that presents with FLANK PAIN & FEVER

Ureter injury...the ureter is most vulnerable to injury during a hysterectomy...damage can occur at the point of ligation of the uterine vessels because the ureter passes inferior and lateral to the uterine arteries at the level of the INTERNAL CERVICAL OS

Patient with really bad acne is interested in isoretinonin treatment, what is the next step in deciding if its contraindicated?

Urine B-hCG test (isoretinoin is contraindicated in pregnancy - category X)

22 year old male reccurent blistering on back of hands and forearms, drinks 3 beers a day...what enzyme is deficient?

Uroporphyrinogen DECARBOXYLASE (UROD) - Patient has porphyria cutanea tarda (PCT) most common disorder of porphyrin (heme) synthesis. Enzyme deficiencies in EARLY steps of heme synthesis manifest as neuro disorders. Late in steps of heme synthesis: photosensitivity

What are potential side effects of tricyclic antidepressants, like Amitriptyline

Used occasionally for pain management or insomnia. TCA's have strong anticholinergic properties, potential side efx include: confusion, constipation & urinary retention. Meds should be used with caution in ELDERLY

Baby born with neonatal tetanus, what is the most effective strategy to prevent the condition?

VACCINATION OF PREGNANT WOMEN

What should the patient do who is experiencing paroxysmal supraventricular tachycardia?

VAGAL stimulation (such as valsalva maneuver or carotid sinus massage) RECTUS ABDOMINUS is the most important muscle in achieving increased pressure in valsava maneuver!!

African american kid "runs out of breath quickly" missed several pediatric vaccinations and has been hospitalized twice for "chest infection" reports his bones hurt. What mutation?

VALINE SUBSTITUTION FOR GLUTAMIC ACID = sickle cell = exertional dyspnea, pneumonia resulting ina life-threatening acute chest syndrome and recurrent abdominal and bone pain are clinical features of sickle cell anemia. (SINGLE POINT MUTATION: VALINE -> GLUTAMIC ACID)

Patient with new rapid/irregular jerking movements involving face arm & legs. No previous illness except sore throat 3 months ago, what is he at risk for developing

VALVULAR HEART DISEASE ~ RHEUMATIC FEVER (Sydenham chorea) Occurs months after group A streptococcal infection...carry high risk of chronic valvular disease.

Coagulase negative Staph (epidermitis/saphroliticus)...what is the EMPIRIC tx that should be given?

VANCOMYCIN

Blood cultures grow enterococcus and the strains isolated are known to substitute D-lactate for D-alanine in the synthesis of pentapeptide proteoglycan precursors, this decreases binding for what drug?

VANCOMYCIN ...this is how resistance is conferred in organisms such as VRE.

What drug can assist patients with cessation of tobacco use by reducing withdrawal cravings and attentuating the rewarding effects of nicotine?

VARENICLINE = partial agonist of nicotinic acetylcholine receptors.

3day old girl brought to ED persistent vomiting and refusal to feed, vomit is greenish-yellow color (no blood) normal duodenum, absent jejunum and ileum and remainder of distal ileum winding around a thing vascular stalk what process is responsible?

VASCULAR OCCLUSION ~ Intestinal atresia distal to the duodenum occurs due to vascular accidents in utero. "apple-peel" atresia occurs when the superior mesenteric artery is obstructed = blind-ending proximal jejunum; a length of absent bowerl and mesentary and finally a terminal ileum spiraled around an ileocolic vessel

Woman with progressive exertional dyspnea for past 6 months, patient's mom died of pulomonary arterial hypertension...if patient's condition is inherited what is the most likely cause of her disease?

VASCULAR SMOOTH MUSCLE PROLIFERATION (vascular PAH = pulmonary arterial hypertension) PAH > 25 due to an INACTIVATING MUTATION BMPR2 (two hit hypothesis)

In msot cases of angiogenesis in tumors or otherwise, what are the two likely suspects?

VEGF & FGF ~ Vascular endothelial growth factor (VEGF) stimulates angiogenesis in a variety of tissues as VEGF increases endothelial cell motility and proliferation. FGF(Fibroblast Growth Factor) FGF-2 is involved in endothelial cell proliferation, migration and differentiation. also plays an important role in embryogenesis by stimulating angioblast production

67 year old smoker comes in with 2-week history of decreased vision in right eye (blurry) needs extra light to read paper. Exam shows grayish discoloration of macula w/areas of adjacent hemorrhage...what should be target of therapy?

VEGF (vascular endothelial growth factor is increased and the patient is experiences WET AGE-RELATED MACULAR DEGENERATION) Acute vision loss + metamorphopsia w/fundoscope showing a grayish-green subretinal membrane. TX: VEGF inhibitor(ranibizumab, bevacizumab)

Patient has congenital long QT syndrome described in stem as family hx of sudden death & ion channel defect (outward potassium flow with prolonged action potential) what is the most likely consequence of the patient's disease?

VENTRICULAR TACHYCARDIA AND SUDDEN DEATH ~ congenital long QT syndrome is most often caused by genetic mutations in a K+ channel protein that contributes to the outward-rectifying potassium current. A decrease in the outward K+ current leads to prolongation of action potential duration and QT interval.

Man taking atenolol & aspirin gets new medication that causes dizziness and reduced BP, what med?

VERAPAMIL...combined use of non-dihydropyridine CCBs and Bblockers can have additive negative chronotropic effects...yielding sever bradycardia and hypotension.

Chromosome 3p deletion, what gene/disease?

VHL (von Hippel-lindau disease) ~ common in patients with SPORADIC RENAL CELL CARCINOMA

Man gets Vasectomy, what should he be warned about following the procedure?

VIABLE SPERM IN THE EJACULATE ~After vasectomy, viable sperm remain in the portion of the vas deferens distal to the ligation. Patients can still have viable sperm in the ejaculate for 3 months and at least 20 ejaculations

What happens with excessive Vasoactive Intestinal peptide (VIP) or VIPoma?

VIP inhibits gastric secretion. Somatostatin inhibits VIP. Excessive ViP secretion can be due to a pacreatic islet tumor (VIPoma) resulting in watery diarrhea, hypokalemia & achlorhydria [WDHA]syndrome = pancreatic cholera. Excess chloride loss in the stool.

Man with abdominal discomfort and black stools PMHx: asthma and hypertriglyceridemia. Transmural inflammation of mid-sized arteries with areas of amorphous, eosin-staining arterial wall necrosis. Areas of internal elastic lamina disruption are also present. What is the predisposing factor?

VIRAL HEPATITIS B ~ POLYARTERITIS NODOSA = multisystem vasculitis characterized by episodic ischemic symptoms in various organs with sparing of the lungs. Biopsy shows segmental, transmural inflammation with fibrinoid necrosis. PAN is COMMONLY ASSOCIATED WITH HEPATITIS B.

What is inhibited by oseltamivir?

VIRION PARTICLE RELEASE oseltamivir(tamiflu) is a neuraminidase inhibitor useful in tx and prevention of influenza A & B, the medication impairs the release of newly formed virions from infected host cells and impairs viral penetration of mucous secretions that overlie the respiratory epithelium'

Since breastfeeding is the gold standard for infant nutrition, what other 2 vitamins should be given as a supplement?

VITAMIN D & VITAMIN K

16 year old boy progressive gait instability, dysmetria and dysarthria, brother died of neurological disease...MRI shows degeneration, what conditions has clinical manifestations that closely resembler this patient's disease?

VITAMIN E DEFICIENCY ~ FRIEDREICH ATAXIA(hypertrophic cardiomyopathy + kyphosis and hair foot arches)

Low-pitched holosystolic murmur at the left sternal border that accentuates during manuevers that increase afterload (handgrip manuever)

VSD [small VSD may be assymptomatic and found on routine screening]

15 year old girl (healthy) has amennorrhea (no period) ultrasound reveals shortened vaginal canal with rudimentary uterus, whats up?

Vaginal Agenesis (MUELLERIAN AGENESIS) Mayer-Rokitansky-Kuster-Hauser syndrome have no upper vagina, and variable uterine development.

27 year old man brought to ER by wife, he's been "acting crazy" for the last 2 weeks and hasn't slept in 4 days...patient has history of depressive episodes, what medication is most appropriate for long-term use in the patient?

Valproate (valproate, lithium, or quetiapine can be used for mood stabilization in people with BIPOLAR.) Valproate is most commonly used for seizure prophylaxis. Ethosuximide is also effective against absence seizures but does not suppress tonic-clonic seizures

What bipolar medicine would also help prevent generalized tonic-clonic & myoclonic seizures?

Valproate...valproate is most commonly used for seizure prophylaxis

What medication is preferred in patients with combined absence and tonic-clonic seizures?

Valproate...valproate is most commonly used for seizure prophylaxis. Ethosuximide is also effective against absence seizures but does not suppress tonic-clonic seizures

Patient has some dermatomal pain + vesicular rash, how can you tell if this is a primary or reactivation VZV?

Varicella IgG antibodies suggest an antecedant primary infection, which means that this would be a reactivation of VZV...the IgG Varicella antibodies confer immunity against chicken pox reinfection but not against herpes zoster (=reactivation of VZV)

What are the only cells within the atherosclerotic plaque capable of synthesizing structurally important collagen isoforms and other matrix components?

Vascular smooth muscle cells! Guy dies and have plaques on heart, atheromas have fibrous cap..what cells are responsible for fibrous cap? Vascular SMOOTH MUSCLE CELLS

What part of the thalamus mediates satiety?

Ventromedial [hypothalamus is located in the diencephalon]

What is always necessary for development of acute pyelonephritis following sexual intercourse?

Vesicouterine urine reflx: fecal flora or gram negative rods (E coli, Klebsiella, Proteus) & Enterococcus ascend up the short urethra in women into bladder. When flow is disrupted from virulence factors cystitis occurs.

How is oxygenated blood from the placent delivered to the fetus?

Via Ductus Venosus (umbilical vein = highest oxygen concentration in fetal circulation) enters directly into the IVC (INFERIOR VENA CAVA)

Patient with diffuse B cell lymphoma (most common Non-Hodgkin Lymphoma) has peripheral neuropathy due to what drug? What phase of cell cycle does it work?

Vincristine - M Phase. Vinca alkaloids inhibit microtubule formation (specifically agents binding to Beta-Tubulin) [neurotoxicity - stocking glove neuropathy]

What produces dextrans?

Viridins Streptococci (strep mutans/sanguinis) are normally present in mouth and can account for tooth decay and initiation of dental carries. Same bug causes bacterial endocarditis

What is scurvy and what are the clinical signs?

Vit C deficiency...severly malnourished or homeless/alcoholic...peridontal disease, poor wound healing, hyperkeratotic hair follicles. Vit C is necessary for hydroxylation of Lysine & Proline

How does Vitamin A deficiency present clinically...how is it related to BILIARY OBSTRUCTION?

Vitamin A deficiency (night blindness, eye dryness/corneal ulceration, hyperkeratosis, growth retardation). Deficiency of this fat-soluble vitamin can develop in patients with biliary disorders, exocrine pancreas insufficiency, interstinal malabsorption.

Unvaccinated 20 month old girl comes to ED with a rash, she developed fever with cough, congestion & red eyes...she is ill/lethargic +Rash. Deficiency of what vitamin is associated with her high rate of complications?

Vitamin A ~ Measles. Vitamin A can be beneficial in the tx of measles infection by reducing comorbities (keratitis & corneal ulceration), recovery time and length of hospital stay.

What is wernicke encephalopathy? What deficiency is it associated with?

Vitamin B1 (THIAMINE) deficiency ~ acute is Wernicke encephalopathy which is associated with the MAMMILARY BODY (under hypothalamus & part of the papez circuit). MUST GIVE THIAMINE BEFORE DEXTROSE OR YOU CAN PRECIPITATE WERNICKE ENCEPHALOPATHY

Anemia (exertional dyspnea, fatigue, pallor) + neuro deficits, what kind of deficiency? What is the confirmatory test?

Vitamin B12 deficiency (impaired myelin synthesis [dorsal column & lateral corticospinal tract] & megaloblastic anemia) VitB12 is a cofactor for methylmalonyl-CoA mutase (methylmalonyl-CoA -> Succinyl CoA) & methionine synthase (homocystiene & folic acid -> methionine)

What vitamin intoxication shows: Altered mental status, muscle weakness, constipation, polyuria/polydipsia?

Vitamin D intoxication: due to hypercalcemia. Can result in SARCOIDOSIS

SIBO, small intestine bacterial overgrowth is characterized by an overproduction of what?

Vitamin K and Folate overproduction = SIBO = small intestinal bacterial overgrowth. (defiency in: B12, A, D, E & iron) (overproduction: Vitamin K & Folate)

Patients with what disease often present with a lifelong history of mucosal bleeding (gingival bleeding, epistaxis, menorrhagia)?

Von Williebrand factor deficiency...patients have normal platelets but prolonged bleeding time due to impaired platelet functioning

12 days after time of death from MI what is likely to be seen?

WELL-DEVELOPED GRANULATION TISSUE WITH NEOVASCULARIZATION

Man with right hand pain, has "pins and needles" sensation & PhysExam shows a loss of sensation over the right 5th digit...patient is most likely to have weakness in what movement of the wrist?

WRIST ADDUCTION due to ulnar nerve injury...(most commonly occurs at the level of the elbow, may cause sensory loss in the medial 1.5 digits of the hand (look up ulnar claw pic)

Image of a person getting a shot on the lateral epicondyle, what muscle attaches there and what is it responsible for?

WRIST EXTENSION ~ EXTENSOR MUSCLES ~ LATERAL EPICONDYLITIS (angiofibroblastic tendinosis at its origin on the lateral epicondyle)

Occlusion of the anterior cerebral artery, what function is likely to be impaired?

Walking up stairs.

What is the main side effect of Thiazolidinediones?

Weight Gain + Edema. They exert their glucose-lowering effects by reducing insulin resistance...binds to PPAR-gamma (peroxisome proliferator-activated receptor-gamma)

When would you see warfarin-induced skin necrosis?

When a patient has a protein C deficiency or protein S deficiency. Patients started on warfarin develop a transiet hypercoagulable state due to the short half-life of protein c (this is made worse in a patient who is protein c deficient and can result in thrombotic occlusion of microvasculature with skin necrosis).

Autosomal recessive condition of excess copper...disease and tx? (3rd question, know this)

Wilson's disease = autosomal recessive condition of excess copper...leads to toxic accumulation in the liver basal ganglia & cornea. TX: Chelation therapy with D-penicillamine is indicated to remove excess loosely bound serum copper.

Eczema + Reccurent infections + Thrombocytopenia

Wiskcott-Aldridge syndrome = eczema + thrombocytopenia + combined B & T lymphocyte deficiency

Explain the normal increase in maternal insulin resistance...

Woman comes into baby doc office and has a 50g glucose containing drink and an hour later her lab findings are abnormal. HUMAN PLACENTAL LACTOGEN increases maternal insuling resistance during 2nd & 3rd trimesters --> rise in serum glucose [gestational diabetes comes when the compensatory rise in maternal insulin secretion is inadequate to prevent sodium glucose levels from getting super high].

Difference between medicare and medicaid...who is covered by medicare under 65?

Woman's disabled son! Medicare is a federal insurance program for individuals 65+ who have a work history! OR younger individuals with disabilities

Migrant ranch workes comes to ED with paroxysmal involuntary muscle contractions involving: jaw, neck & trunk...how did the causitive substance travel?

Wound -> Motor neuron axons -> spinal cord. Clostridium Tetani is responsible for tetanus, a toxin-mediated disease that causes uncontrolled muscle spasms and respiratory failure. Toxin travels within the motor neuron by RETROGRADE TRANSPORT into the spinal cord where it causes inhibition of inhibitory interneurons and unregulated firing of primary motor neurons

28 year old man is given prophylatic medications and vaccinations before a business trip to Africa, 5 days later presents with jaundice & dark urine. Smear shows RBCs with dark inclusions....what is disease and inheritance?

X-LINKED RECESSIVE (G6PD deficiency) dark inclusions = Heinz bodies

For patients who have frequent or semi-frequent gouty arthritis attacks, what is recommended for chronic use?

XANTHINE OXIDASE INHIBITORS

5 year old boy, persistent facial ulcer of 2 months (squamous cell carcinoma) ...what disease and what enzyme is deficient?

XERODERMA PIGMENTOSUM ~ ENDONUCLEASE DEFICIENT ..failed nucleotide excision repair so THYMINE DIMERS...autosomal recessive, hyperpigmentation in sun-exposed areas.

What enzyme inactivates 6-mercaptopurine?

Xanthine Oxidase [6-MP is mainly degraded in the liver by xanthine oxidase...Allopurinol[XO inhibitor] can increase the concentration of 6-MP significantly. (6-MP & 6-thioguanine is a PRODRUG that requires HGPRT for activation)

Woman brings her daughter in because she sunburns too easily (thin & hyperpigmented skin) + Nevi. What is disease/process?

Xeroderma Pigmentosum - defect in Nucleotide Excision Repair (increased sensitivity to UV light)

Do patients have the right to have information withheld from them? What is appropriate response?

YES. "TELL ME MORE ABOUT YOUR CONCERNS SO I CAN BETTER UNDERSTAND WHY YOU FEEL THIS WAY"...autonomy!

Single sarcomere is from what to what? How does it look on EM

Z line to Z line (BLACK thick lines surrounded by WHITE)...thin/actin is bound to the Z lines

What is Zollinger ellison Syndrome?

ZES is caused by gastrin-secreting neuroendocrine tumors (gastrinomas) involving small intestine or pancreas. Patients typically have peptic ulcers, abdominal pain, reflux, diarrhea [associated with multiple endocrine neoplasia type 1]

Man comes in with muscle weakness, Labs: low plasma renin activity, changes in BP standing and laying down.....overactivity of what structure is responsible?

ZONA GLOMERULOSA OF ADRENALS ~ primary hyperaldosteronism

Calculation for maintenance dose

[Cpss X CL]/[bioavailibility fraction]

Where does the C1 molecule bind to IgG?

[IgM circulates in a pentameric form and is a better activator of compleMent) The C1 molecule binds to the Fc region of the heavy immunoglobulin chain in the region near the hinge point.

What is gestational choriocarcinoma?

a malignant tumor that arises from the TROPHOBLAST (can be preceded by any type of pregnancy). Tumor casues: vaginal bleeding, uterine enlargement, significantly increased B-hCG levels! PROLIFERATION OF CYTOTROPHOBLASTS & SYNCYTIOTROPHOBLASTS

Hypertension at a young age (30) is suggestive of what?

a secondary cause such as hyperaldosteronism. Most common cause of primary hyperaldosteronism is unilateral adrenal adenoma. (conn syndrome)

What is tardive dyskinesia?

abnormal involuntary movements of mouth, tongue, face, extremities, trunk ...associated with prolonged exposure to ANTIPSYCHOTICS

What action does norepinephrine have on cAMP in the heart?

cAMP INCREASE IN CARDIAC MUSCLE CELLS ~ Norepinephrine stimulates B1 adrenoreceptors which utilize the cAMP signal transfuction pathway. Stimulation of these receptors by norpepinephrine causes cAMP concentration within cardiac myocytes.

What is confounding?

confounding occurs when there is an extraneous factor that is associated with both exposure and disease

elevated sweat chloride concentrations are found in most patients with

cystic fibrosis

Sympathetic nervous system regulates visceral function via a 2 neuron system (cholinergic pre-ganglionic & adrenergic post-ganglionic)../what are the two exceptions?

eccirine sweat glands & adrenal medulla are exceptions as they are both innervated by cholinergic neurons

Woman gets in car accident did not see car coming from front-side (bitemporal hemianopsia) MRI shows enlarged structure, what is it and what embryonic layer is it derived from>

enlargement of the PITUITARY GLAND is suggestive of a prolactin-secreting pituitary adenoma. ECTODERM gives rise to surface ectoderm, neural tube & NEURAL CREST\

Man with history of heartburn comes to office with increased substernal bleeding (especially at night)...what are his symptoms the result of?

erosive esophagitis (new onset odynophagia = painful swallowing) ULCERATION=symptoms. [barret esophagus = no symptoms

Absolute risk reduction =

event rate in control group - event rate in treatment group

What is akathisia?

extrapyramindal side effect associated with antipsychotic treatment (ranges from subjective feelings of tension to marked physical restlessness). If antipsychotic dose is increased the akathisia will increase.

4th pharyngeal arch

fourth aortic arch + superior laryngeal branch of the vagus nerve

What is the primary cause of morbidity in acute rheumatic fever?

heart failure from severe pancarditis. Acute rheumatic fever is a complication of untreated group a strep.

A patient who recently received systemic chemotherapy for breast cancer and now how progressive hematuria & suprapubic tenderness most likely has what, caused by what?

hemorrhagic cystitis caused by a nitrogen-mustard based chemotherapeutic agent such as cyclosphosphamide

Kid has jaundice and is sick and has palpable spleen. Small red blood cells that lack a zone of central palor...name of disease and why?

hereditary spherocytosis...spectrin and ankyrin are messed up. So RBC plasma membrane scaffolding is messed up...leads to destruction of spleen and as a result hemolytic anemia and jaundice result. Scleral icterus and palpable spleen

What is hereditary spherocytosis and what confirms the diagnosis?

hereditary spherocytosis...spectrin and ankyrin are messed up. So RBC plasma membrane scaffolding is messed up...leads to destruction of spleen and as a result hemolytic anemia and jaundice result. Scleral icterus and palpable spleen [young man + osmostic fragility]. PIGMENTED GALLSTONES ARE A COMPLICATION OF ANY HEMOLYTIC ANEMIA.

Woman has family members with the same inherted disease but they all show it differently, what genetic principle describes this?

heteroplasmy !!!!! Describes the condition where different mitochondrial genomes within a single cell...

Super high LDL level, normal serum triglycerides...what disease? What is defective?

heterozygous familial hypercholesterolemia (AUTOSOMAL DOMINANT) LDL receptor defect that results in high LDL levels and increases risk of premature atherosclerosis. Southern blot can be used to ID DNA (uses restriction endonuclease digestion of sample DNA and gel electrophoresis & gene identification with a labeled DNA probe)

28 year old male presents with involuntary movements, depression and a family history of movement disorders likely has what? Why did his disease presentation show sooner than his fathers?

huntington's disease CAG repeats. ANTICIPATION is the genetics term...there are increasing numbers of trinucleotide repeates (CAG for HUNTINGTONS) each successive generation, the larger number of repeats the earlier the onset of the disease...this occurs more frequently during PATERNAL TRANSMISSION causing anticipation.

High altititude exposure results in what kind of response?

hypoxemia (low O2) with respiratory alkalosis...after a few days, chronic respiratory alkalosis sets in with a corresponding decrease in serum bicarb level reflecting renal compensation,

What is a crossover study?

in a crossover study subjects are randomly allocated to a sequence of 2 or more treatments given consecutively [a washout period (time with no tx) is often added between treatment intervals to limit confounding].

Nitrates a primary venodilators that do what?

increase peripheral venous capacitance = reduced cardiac preload & LVEDP&volume. Decrease SVR & cardiac afterload.

Baby is too sleepy to feed & has been vomitting (a brother who died from low sugar in his blood) Urine test reveals elevated levels of methylmalonic acid what would lab values look like?

increased urine propionic acid, decreased serum glucose, increased urine ketones, increased serum ammonia. Methylmalonic Acidemia from complete or partial DEFICIENCY OF METHYLMALONYL-CoA MUTASE

In the treamtent of hypovolemic shock, how do IV fluids help?

intravascular volume & ventricular preload can be increased rapidly. The increase in preload stretches the myocardium and increases VENTRICULAR END-DIASTOLIC SARCOMERE LENGTH --> leads to increased stroke volume & cardiac output.

Stem will be talking about a man with reccurent syncope, who is given a drug and EEG...the drug will INCREASE CARDIC CONTRACTILITY and at the same time DECREASE VASCULAR RESISTANCE?

isoproterenol = nonselective B-adrenergic agonist increases cardiac contractility (positive ionotropic effect) via B1 & decreased vascular resistance by binding to B2. B1/B2 Agonist.

What is the purpose of the copper reduction test?

it detects reducing sugars (fructose, glucose, galactose) FRUCTOKINASE DEFICIENCY = POSITIVE COPPER REDUCTION TEST WITH NEGATIVE GLUCOSE OXIDASE TEST

How does autosomal DOMINANT polycystic kidney disease manifest in life?

later in life...40-50 years old: enlarged kidneys, hypertension, and renal failure. Look normal as newborns/young adults. But the DOMINANT form of polycystic kidney disease manifests later in life. [mutations in PKD-1 or PKD-2] can be clincally silent.

What is the triad seen in aplastic anemia?

low hemoglobin, thrombocytopenia, absent hematopoietic cells in bone marrow is consistent with aplastic anemia.

Relationship between lung absess and aspiration pneumonia

lung absess is a common complication of aspiration pneumonia, which occurs in individuals with impaired consciouness or inability to swallow (alcoholics, dementia) [anaerobic bacteria from the gingivodental sulcus are the most common cause of lung absesses]

The vomiting reflex is mainly mediated by 5 receptors, what are they?

m1Muscarinic, d2Dopaminergic, H1histaminic, 5-HT3serotinergic & NK!neurokinin-1

patient with beta-thalassemia minor is described in question stem...what is the initialy step in the pathogenesis of the disorder?

mRNA FORMATION ~ beta-thalassemia is caused by mutations that result in defective transcription, processing, and translation of beta-globin mRNA. This leads to deficiency of the beta-globulin chains required for normal hemoglobin synthesis

Woman has enterococcus, given IV drug and develops tinnitis and hearing loss...how does the drug work?

mRNA GENETIC CODE READING ~ patient has enterococcal endocarditis and likely received an AMINOGLYCOSIDE ~ gentamicin a RIBOSOME-TARGETING ANTIBIOTIC...Aminoglycosides inhibit genetic code and protein synthesis by BINDING TO THE PROKARYOTIC 30S RIBSOMAL SUBUNIT.

What is the formula for 95% confidence interval?

mean +/- 1.96*SD/sqrt(n)

Aprocrine glands release what?

membrane-bound vesicles containing the secretory product.

New drug activates GABA-A in CNS, nerve (resting membrane is -70) is introduced to the drug what will the new resting membrane be?

minus 75 mV ~ this is because GABA-A receptors are iontotropic and regulate the flow of negatively charged Cl- ions across the neuronal cell membrane. More hyperpolarized due to reaching equilibrium potential for chloride

Frequently encountered diseases (such as spina bifida & hypertension) what kind of inheritance?

multifactorial (complex interaction of genetic and environmental)

Elevated alpha fetoprotein is associated with:

multiple gestation, open neural tube defects & abdominal wall defects

22 year old guy keeps waking up from dull headaches, physical exam shows multiple pigmented spots on his neck and trunk. What cell layer are the cells forming the tumors from?

neurofibromatosis type 1 - von reckinghausing autosomal dominant NF1 tumor suppressor mutation. Tumors are derived from Schwann Cells which come from NEURAL CREST CELLS

Bilateral acoustic neuromas are associated with what?

neurofibromatosis type 2

Why would a strain of H influenzae be classified as "nontypable" how would this reflect clinically?

nontypable strains of H influenzae do NOT produce a capsule and are referred to as nontypable. Which would explain how a kid who got the influenzae vaccine could still get H flu from a nontypable strain.

Anion gap metabolic acidosis + DKA what is the treatment of choice?

normal saline & insulin

5th pharyngeal arch

obliterated

What do lacunar infarcts result from?

occlusion of small penetrating arteries that supply the brain...resulting in lipohyalinosis w/small vessel occlusion.

What do organophosphates do to ACh in the synpatic cleft?

organophosphates prevent the degradation of ACh in the synpatic cleft

A 99% confidence interval that does not cross the null value corresponds to a p value of?

p=.01 (statistically significant)

A 95% confidence interval that does NOT cross the null value has a p-value of what?

p=.05 (statistically significant)

hardy weinberg phenotypic =

p^2(freq of normal) + 2pq(carrier freq) + q^2(freq of diseased individuals) = 1

What is polymyalgia rheumatica?

polymyalgia rheumatica occurs in more than half of the patients with temporal arteritis (neck/torso/shoulder/pelvic girdle pain/morning stiffness) Fatigue, Fever, weight loss may also occur

Kid has skin infection, not responsive to nafcillin but IS responsive to vancomycin...why?

poor interaction with binding proteins [MRSA is resistant to all Beta-Lactam antibiotics (including b-lactamase resistant antibiotics...as it has an altered PBP)]

39 year old nulliparous woman pelvic pressure and constipation exam shows irregularly enlarged uterus and normal rectal tone, whats up?

posterior subserosal uterine leiomyoma. Subserosal leiomyomas (fibroids) often cause irregular uterine enlargement resulting in pelvic pressure '

Which HIV antiviral drug class has hyperglycemia, lipodystrophy and drug-drug interactions due to inhibition of cytP450?

protease inhibitors like LOPINAVIR

How do proton pump inhibitors reduce HCl output?

proton pump inhibitors (omeprazole, LANSOPRAZOLE) block the final common pathway of gastric acid secretion from parietal cells, which is stimulated by acetylcholine, histamine and gastrin

In order to maintain blood flow through the body, what two circulations must match

pulmonary circulation & systemic circulation [volume of output of RV and LV must match balance is necessary to maintain continuous flow during rest and exercise]

1-2 weeks after ischemic stroke what is seen on histology?

reactive gliosis & vascular proliferation around necrotic area.

6th pharyngeal arch

reccurent laryngeal + pulmonary arteries + ductus arteriosus

Spirometry findings in restrictive lung disease

reduced total lung capacity, vital capacity, inspiratory capacity, functional reserve capacity, and residual volume

What does the highest point on the jugular venous tracing curve indicated?

right atrial contraction...the 3 positive waves (A,c,V) & 2 negative waves (x, y)....the "a" wave (ATRIAL CONTRACTION) as absent in patients with atrial fibrillation

What is a left atrial appendage (LAA)?

small sack-like structure in left atrium that is particularily susceptible to thrombus formation. 90% of left atrial thrombi are found in LAA in patients with nonvalvular atrial fibrillation

hardy weinberg (calculating mutant allele frequency from disease prevalence) =

sqrt(q^2)[disease prevalence] = q(mutant allele frequency)

What part of ECG do Beta blockers actually effect?

the PR INTERVAL ...beta blockers decrease AV nodal conduction, leading to an increased AV nodal refractory period. Correlates to PR prolongation on an ECG.

Serotonin syndrome

triad of autonomic instability (hyperthermia, hypetention, tachycardia), altered mental status, neuromuscular hyperactivity MAOI antidepressant or LINEZOLID IS COMBINED WITH SEROTONERGIC

Saline microscopy is the best way to diagnose what?

trichamonas vaginalis (saline microscopy = wet mount) Trichamonas presents with yellow-green vaginal discharge and vaginal burning

Holosytolic murmur that increased in intensity during inspiration...what heart path, what is increased intensity during inspiration called?

tricuspid regurgiation (Cavallo sign)...holosystolic murmur that increase in intesity during INSPIRATION

Given graph of flow rate vs. MAPB assuming a hematocrit of .5 what is the best estimate of filtration fraction @ 120 mmHg?

use GFR on graph = .1 @120 mmHg....FF = GFR/RPF [RPF = RBF * (1-Hematocrit)]

What does von williebrand factor bind to?

vWF binds as a promoter of platelet adhesion at sites of vascular injury by binding platelet glycoproteins to subendothelial collagen on injured blood vessel walls [also acts as a protective carrier for circulating factor VIII]

In a normal standard distribution where do 68% of values fall?

within 1 standard deviation from the mean

In a normal standard distribution where do 95% of values fall?

within 2 standard deviations of the mean

In a normal standard distribution where do 99.7% of values fall?

within 3 standard deviations from the mean


Related study sets

Macroeconomics 102 final study guide

View Set

Pharm Exam 3: Ch. 49 - Antidysrhythmic Drugs (Key Bank & Elsevier Q's)

View Set

Chapter 10 Food Selection and Preparation (Exam 2)

View Set

Chemistry Chapter 25 (Nuclear Radiation)

View Set

L2, R41: Backtesting and Simulation

View Set